Question #001. OA26: Second Amendment Masterclass, Part 2 A property owner agrees to sell one of his real estate parcels to a neighbor. He wrote up an agreement for sale with a sale price of $200,000 and signed the document. The neighbor took it and returned with a bank check for the $200,000 dollars demanding that the deed be tendered. The property owner then decided that the terms were not reasonable, and he returned the money and refused to tender the deed as required by the agreement. The neighbor sued for specific performance of the transaction. The property owner defended on the basis that the neighbor did not accept the agreement's terms and did not sign the document. Is the court likely to rule in favor of the neighbor? A) No, because the neighbor did not expressly state that he accepted the agreement. B) No, because the neighbor had to put his signature on the document to make it a binding contract. C) Yes, Because the neighbor manifested his assent by his conduct of tendering the full sale price, at which point a contract existed. D) Yes, Because the property owner created a binding contract when he prepared an agreement containing the agreed terms. Question #002. OA27: Abortion and Planned Parenthood v. Casey, Part 1 An inexperienced bicycle rider took her new bike for a test run. I already love this question. At an intersection, she could not stop for a red light and went into the intersection where she slipped off the bike. While attempting to remount it, a car coming into the intersection with a green light ran her over, causing severe injuries. The car driver was distracted by his cell phone and received a careless driving ticket from the police. The woman filed a claim for damages, asserting that the car driver was negligent per se. She claimed that a finding of negligence per se prevented the driver from asserting comparative negligence under a state statute. Will the trial court likely grant the woman's motion to preclude the man's attempted comparative negligence defense? A. Yes, because the driver had the last clear chance to avoid the accident, which abrogates the comparative negligence law. B. Yes, because negligence per se is a final judgment of total negligence against the driver and it cannot be modified or rebutted. C. No, because the common law doctrine of negligence per se does not abrogate the defendant's right to apportion fault under the comparative negligence statute. D. No, because she entered the intersection knowing it was a red light, thereby putting herself voluntarily and knowingly in danger and she assumed the risk. Question #003. OA28: Abortion and Planned Parenthood v. Casey, Part 2 A man sued a manufacturer of video and home theater equipment after his home theater started a raging fire, destroying the equipment and part of his home. He sued the manufacturer in federal court. At trial, the defendants filed a motion to preclude the report and testimony of the plaintiff's expert witness under Federal Rules of Evidence, Rule 702. The defendants claim bias, lack of scientific support, failure to conduct testing, divergence of opinion among experts, and that the expert was trained for large commercial fires. The expert had investigated over 500 fires, wrote a book that was used in courses nationwide, and taught a course in forensic fire investigations at a university. Her report ruled out all other possible causes. The court did a gatekeeping review, as required by Rule 702, and ruled that the expert was highly qualified and that she used accepted scientific methodologies in arriving at her opinion. The defendants appealed the ruling. Is the appellate court likely to go against the trial court ruling under these facts? A, yes, because the charge of bias against an expert witness requires a "trial within a trial", and that was not done. B, yes. Because whenever the expert does not back up her conclusion with experimental testing, an expert's opinion will be insufficiently reliable to be allowed. C, no, the court did the required gatekeeping inquiry and found that the witness was not only qualified and competent, but also used accepted methodologies. D, no, the court has absolute discretion in deciding the competency of the expert for purposes of giving expert trial testimony and evidence. Question #004. OA30: Little Baby Jesus in a Manger A homeowner buys a new leaf blower and is so impressed with its power that he gets carried away and blows large amounts of his work onto his next door neighbor's yard. In addition to leaves, he also blew large amounts of pest-infested plant debris onto the neighbor's property. The material has accumulated into large, unsightly piles and caused damage to the neighbor's healthy plants. He has to pay workers to haul the material away and incurs expenses to replace the infected perennials. The neighbor sues for the tort of trespass. The homeowner files a motion to dismiss because the neighbor cannot prove that the homeowner acted with intent to trespass or to do harm. Will the court grant my motion to dismiss? A. No, because the infected debris constituted an abnormally dangerous condition and the homeowner could be held strictly liable for the trespass. B. No, because trespass is based on an intentional act of entering the land or sending something onto the land and the actor does not have to intend harm to be liable for the tort. C. Yes, because the homeowner had no practical way of controlling where the material went and he acted without malice or ill will. D. Yes, because the homeowner expected the wind to carry the debris away and did not think that it would accumulate on the neighbor's property. Question #005. OA32: Phil Ivey’s Gambling Winnings (with guest Chris Kristofco) A creditor instituted a garnishment action against a woman who owed the creditor a back due debt. This was the first action taken, and the creditor did not have a judgment. It served the notice of garnishment on the garnishee, the woman's employer. The employer honored the notice by holding all of the woman's earnings in escrow, pending the creditor's lawsuit, and the entrance of a final judgment against the woman. The state where the defendant lived and worked had a provision for pre-judgment garnishment as long as the money was held and not distributed until the judgment was entered. The money would be paid to the woman if a judgment was not entered within the next 120 days. The woman sued the employer and the creditor, demanding the release of her earnings. Will the court order the garnishee to release the funds to the woman? A. No, because the state procedure for protecting the woman's money until a judgment is entered is a fair one that comports with procedural due process. B. No, because the state has an interest in protecting the rights of creditors to collect debts through the garnishment procedure. C. Yes, because the only party that has garnishment rights in the United States is the Internal Revenue Service. D. yes, because the interim freezing of wages without a chance to be heard violates procedural due process. Question #006. OA34: The "Fallout" Over Copyright Two people who have been cohabiting decide to get married. The man asks the woman to agree that all of his property that he now owns will remain in his name if they ever get divorced. She agrees to that, but demands a written document. However, they get married without having signed anything. Three months later, the husband leaves the wife for another woman. He sends her the notice to vacate the home due to their prenuptial agreement. When she doesn't move, the husband sues, asking for the enforcement of the oral prenuptial contract. Will the court likely enforce the husband's claim? A. Yes, because the consideration for the contract was the marriage and the consummation of the marriage made the contract complete and enforceable. B. Yes, because prenuptial agreements are given the benefit of the doubt when it comes to enforcement decisions. C. No, because a contract in consideration of marriage must be in writing and the prenuptial was therefore not legal so long as it remained oral. D. No, because the husband did not live up to the promise to put the agreement in writing and therefore he was guilty of fraud. Question #007. OA36: The Emoluments Clause (w/Seth Barrett Tillman) Part 2 A painter working for a construction company was working with a group of construction co-workers on a house when a section of the house fell down on the owner. The painter declared, quote, I knew they should have reinforced the standing wall better than that. It's all their fault. And it turned out he was referring to co-workers employed by the same construction company. The owner filed suit for damages against the construction business. He tried to put the statement from the painter into evidence, but the defendant objected. Will the court sustain the company's objection, that is, as the defendant, to the admission of the painter's statements? Okay, so this is a question about hearsay. Will the court sustain the defendant building construction company their objection to the admissibility of the painter's statements? A. Yes, because the painter is not qualified to make such statements regarding non-painting duties. B. Yes, because the painter is not a participant as a negligent party in the catastrophic mistake that occurred. C. No, because the painter works for the same company as the negligent workers and he made his statements within the scope and course of his employment. D. No, because the painter may testify as a subcontractor to what he observed while on the location. Question #008. OA38: FLSA and Exempt Employees, Part 2 Some homeless people started residing on a strip of land located under a bridge. The land was owned by a nearby chemical manufacturer. The squatters did not ask permission and the company did not feel it was an urgent problem. The squatters used a small lake on the property for bathing and drinking water. Within a few days, two squatters died from what was later revealed to be highly polluted water coming from the company's waste discharges. The company knew that it had a waste discharge problem. estates of the two decedents sued the company. Will they likely prevail on their tort claim despite the company's defense that the decedents were trespassers? A. Yes, because even though they trespassed, the owner had a duty to warn because it knew that they were in danger. B. Yes, because the owner was strictly liable for any injuries caused by the hazardous condition of the water in the lake. C. No, because the owner owes no duty to trespassers except if it acts with willful or wanton disregard. D. No, because an owner of land never has to worry about protecting the safety of trespassers. Question #009. OA40: Who is Neil Gorsuch, and How Scared Should You Be? Two cousins took title to real estate as joint tenants. The first cousin put up his interest in the joint tenancy as security for a $20,000 loan and signed a judgment note. It was later entered as a judgment against that cousin's interest in the real estate. The first cousin then died a year later. The second cousin defended on the basis that any lien on the first cousin's interest in the property was extinguished on the first cousin's death. Will the court enforce the lien of the mortgagee against the sale of the property by the second cousin? A. Yes, because the second cousin's assent to the judgment lien was presumed by law. B. Yes, because the joint tenancy was broken when the first cousin put the judgment lien on the property. C. No, because the judgment lien was never valid. A joint tenant cannot file a judgment note on just that joint tenant's interest. Or D. No, because when the first cousin died, the second cousin became the sole owner due to the right of survivorship, as per the joint tenancy, which was still fully intact. Question #010. OA42: Denny Hastert and the Limits of Contract Law A college student files a criminal case against his former roommate, claiming that the former roommate snuck in and removed his state-of-the-art home theater equipment from his house. The college student took the approved prosecution papers and filed them with a magistrate court. An ongoing police investigation determined, however, that the college student was a vengeful person, not averse to creating stories, and that he and the former roommate had been arguing over a number of issues. They also determined that he never had any quality sound or video. The police decide to drop the prosecution. The case is dismissed at the preliminary hearing. When the former roommate filed a civil case against the complainant for malicious prosecution, will the evidence of defendant's reputation for dishonesty be admissible? A. Yes, because dishonesty at any time in the past is always relevant to a malicious prosecution complaint. B. Yes, because even though character evidence is too collateral to be admitted for circumstantial evidence, it is admissible if it is directly at issue in the case. C. Yes, because reputation evidence, just like most character evidence, is virtually always admissible. D. Yes, because while most character evidence is not admissible, some is, and reputation evidence is always admissible. Question #011. OA44: All About Arbitration A patient who had surgery to repair a rupture in her spleen followed up with a full body MRI due to the excruciating pain that had developed. During the MRI, the technician saw at least two identifiable sponges that had been left in her body. The patient sued the hospital and the doctor for medical malpractice. She did not produce the printouts of the MRI. Instead, she attempted to testify as to what she observed on the MRI scan. The defense objected and insisted that she must produce the actual MRI printouts. What is the most likely ruling of the trial judge with respect to the objection? A. She does not have to produce the MRI scan because a layperson can testify to things of common knowledge, such as what a sponge looks like. B. She does have to produce the MRI scan because a layperson cannot testify as to personal observations. C. She does not have to produce the MRI scan because she can testify as to the content of her own medical records. D. She must produce the MRI records because it represents the best evidence under the best evidence rule. Question #012. OA46: What Could Donald Trump’s Tax Returns Tell Us? (With Guest Tony Di Fatta) - Part 2 A witness sees a murder in the park and is called to testify in the criminal case. The defense counsel knows that she was cited several times for illegally feeding the pigeons. Defense counsel tried to ask "You have a long time habit of feeding the pigeons, don't you? You have, in fact, been ticketed several times for feeding pigeons, haven't you? And yet you persisted in doing it." Will the trial court likely allow the witness to be questioned on these matters? A. No, because the question has no probative value regarding the credibility of the witness or the guilt of the defendant. B. No, because a witness can never be asked about personal prior bad acts. C. Yes, because the statement indicates that the witness's attention was always focused on feeding the pigeons. D. Yes, because feeding pigeons in the park and persisting in it after being ticketed may show an antisocial trait of the witness. Question #013. OA48: Three Cases You Care About - Planned Parenthood, Gay Florists, and Litigious Quacks A man is sitting on a beach chair peacefully and in a relaxed meditative state when a truck racing on the sand ran down a sunbathing elderly person. The man said out loud "Oh god, look what that truck driver did". The man driving that truck was racing and going about 100 miles an hour. Someone called 911. A woman who was sitting on the beach nearby but facing the other way heard the man's outcry. When the case comes to trial, will the court allow both the man and the woman to testify about the man's utterances? A. Yes, because both the woman and the man can testify because it is an excited utterance exception to the hearsay rule that goes to the material issue of the case. B. Yes, because this is not hearsay and it is not being entered for the truth of the statement by either witness so that they can both testify. C. No, neither of them can testify because the man was impaired due to being too relaxed from his meditation. My laughing is not an indication that this is an incorrect answer, but that is funny. And the woman cannot repeat the utterances of an impaired witness. Pretty clever. D. No, neither can testify because the utterances do not fit any of the exceptions to the hearsay rule. Question #014. OA50: Obama’s Fiduciary Rule (With Guest Ben Offit) A man with a long history of criminal violence who is a psychopathic personality often enjoys the act of mercilessly brutalizing innocent elderly people by physical torture that is heinous and shocking. He decides to torture the victim's father for no good reason. After the beating, the father is hospitalized and in intensive care. The victim was not present, but read about it in the newspaper and heard about it from the authorities. The victim suffered extreme emotional distress and required medical treatment. Can the victim bring a successful action against the perpetrator for the tort of intentional infliction of emotional distress? A. No, because the victim was not present at the time that the outrageous behavior occurred. B. Yes, because the activity was outrageous and shocking. C. No, because the perpetrator did not know that the father had immediate family members. D, Yes, because in torture cases, there is strict liability imposed on the perpetrator. Question #015. OA52: Thomas Knows Words! Thomas Has The Best Words! An elderly woman was mugged and robbed on a public street. At the time of the offense, the victim saw only her assailant's eyes. A hooded sweatshirt and bandanna concealed the rest of his face. A few days later, while the victim waited to be interviewed, she observed the defendant being escorted through the courthouse in jail clothing and restrained by handcuffs and leg irons. She identified him as the perpetrator shortly thereafter. Will the court allow that identification into evidence? A. No, because it is not reliable enough and there was too much suggestion to make a reasonably accurate identification. B. No, because prior statements of identification are considered hearsay and never allowed it to evidence. C. Yes, if the witness perceived the perpetrator by seeing his eyes and made the identification on that basis, it was reliable enough to be admitted. D. Yes, because the witness's recognition of someone is all that is needed. The reliability or credibility of it is subject to cross-examination. Question #016. OA54: Gerrymandering A business person sent his employee and administrative assistant to represent him at a conference where business deals are often negotiated. The principal gave the assistant all of the materials to set up a table with the principal's cards, brochures, promotional materials, price lists, and even some order forms with his logo on them. In addition, the assistant represented the third parties that she was there officially representing the principal and that she was authorized to execute contracts on his behalf. The assistant negotiated a deal for the business person with a third party which the business person refused to honor because he did not like the terms that were negotiated Will the third party likely prevail in enforcing the contract against the principal? A, Yes, because the furnishing of the indication of authority to act To the employee was sufficient proof to bind the principal. B. Yes, because the principal is bound to the employee's actions whenever he sends an employee to attend a convention on his behalf. C. No, because people dealing with an assistant at a convention have a duty to determine whether that person is in fact an authorized agent. D, No, because the technological revolution makes it easy these days for a potential customer to first confirm the existence of actual authority. Question #017. OA56: Jury Secrecy and Pena-Rodriguez v. Colorado A man went shopping to buy a used car. He found one on a used car lot that was offered, quote, as is. He looked at it thoroughly. The tag on the window indicated a special 30-point inspection was done on the car. The man put a $1,000 down payment down on the car. He took the car to an independent mechanic who told him that the car had been in an accident, sustained severe damage, including a cracked engine block. The mechanic believed that the dealer had to know about such extensive damage. The man tried to return the car and get his down payment back, but the dealer refused. Stuck with a lemon, the man sued the company under a common law fraud theory. The company counterclaimed for the balance of the purchase price. Will the man likely prevail in the litigation? A. Yes, the man will likely prevail in the litigation because the dealer withheld material information intentionally and knowingly misrepresenting the condition of the car. B. Yes, because the dealer has a duty to give every reason why an as-is car may go bad and list those reasons on the window. C. No, because the dealer cannot be held responsible for possible damage that the used car may have developed after years of use. D. No, because the dealer is absolutely protected when he sells a used car in as-is condition. Question #018. OA58: What Football Can Teach Us About Jury Nullification, Antitrust, and Donald Trump - Part 2 The United States Congress passed, and the president signed a law called the American Morality Leadership Act, which limits the amount of makeup that women can wear in public and precludes the wearing of short dresses. Men over 65 must wear only brown or gray attire, while those under 65 must wear button-down dress shirts with neckties, except for when they are exercising or working physically. The Act has other restrictions and specifications for dress and grooming mandates for men and women. A class action is filed to challenge the constitutionality of the Act. Which argument listed below more closely reflects the constitutional argument that is most likely to succeed in court? A. The law is unconstitutional because it violates the Commerce Clause. B. The law is unconstitutional because it denies all citizens the equal protection of the law. C. The law is unconstitutional because it violates procedural due process. D. The law is unconstitutional because it is not rationally related to a legitimate government interest. Question #019. OA60: Sex and Sexual Orientation Two companies, a construction company and a landscaping firm, got into a contract dispute over the nature of landscaping services that had to be provided to the construction company. The landscaping firm was incorporated in New York and did all of its business in that state. The construction company was an LLC that had been filed in New York with its members being domiciled in Pennsylvania, Vermont, and Virginia. The landscaping firm filed a breach of contract claim in federal court in New York claiming diversity jurisdiction within a mountain dispute that was in excess of $100,000. The construction firm filed a motion to dismiss claiming that the landscaper plaintiff did not have diversity jurisdiction. Will the court likely grant the motion to dismiss? A. Yes, there is clearly no diversity in that the construction company LLC and the landscaper corporation were both registered and filed in New York, making them citizens of the same state. B. Yes, because diversity must be between natural citizens and business entities are not entitled to diversity, but must find some other reason for federal jurisdiction. C. No, the motion to dismiss is premature and must wait until all discovery is completed before it will be considered. D. No, the plaintiff is a citizen of New York and the three members of the LLC are each from different states, which gives complete diversity of citizenship. Question #020. OA62: The Supreme Court’s Hall of Shame A regulation prevents persons with a drug arrest and conviction and or those undergoing current treatment for drug abuse from working in certain state-run facilities for the safety of the general public. One employee of the Department of the Treasury brought an action claiming a denial of equal protection and due process by taking away his right to a job. It is claimed that the prohibition is necessary to protect the public from potential problems that could arise. Which test of constitutional validity would likely be applied to this regulation? A. Strict scrutiny since drug users are a special class of persons often discriminated against by the public. B. Intermediate scrutiny, because like gender, this can be a sensitive area where discrimination is common. C. The rational basis test, because discrimination is generally favored with drug users or even those getting treatment. D. The rational basis test, because the regulation need only be related to a legitimate state interest to be valid. Question #021. OA64: How "Net Neutrality" Became "Selling the Internet" - A Choose-Your-Own Adventure, Part 1 The state constitution in one state guaranteed the right to marry to same-sex and opposite-sex couples alike. A political group got a proposition on the ballot and a majority of the people voted to eliminate the right of same-sex couples to marry. The proposition did not affect any of the other rights that same-sex couples may have possessed from existing state law. Persons affected sued the state government for unconstitutionally taking away the rights that they had and doing it for no legitimate purpose in violation of the due process clause of the 14th Amendment. Will the court grant the request of the lawsuit and declare the proposition void and reinstate the right of same-sex couples to marry? A. Yes, under the Due Process Clause, no compelling state interest justifies denying same-sex couples the fundamental right to marry. B. Yes, because under the Fourth Amendment, it would be an illegal interference against the right of privacy guaranteed to everyone. C. No, because the people voted to take a provision out of the existing Constitution, and when that is done by public vote, it cannot be altered. D. No, because the ballot proposition was properly placed on the ballot and all statutory procedures were followed, thus precluding the courts from taking adverse action. Question #022. OA66: Sanctuary Cities At a yard sale, a seller puts out an old etching that he inherited from his aunt. He labels it with a price tag of $10. A buyer comes by, fully aware that it is the work of a famous Italian Renaissance artist, and quietly offers the $10 for an etching that he knows to be worth at least $100,000. When the seller finds out that he made an awful mistake and that the buyer knew it was a precious work of art, he sued to get the $100,000 or for return of the etching. The buyer defended by alleging that the seller did not act reasonably to protect himself by knowing what he was selling. Under the restatement of contracts and the majority rule in case law, will the court grant the buyers motion to dismiss the seller's complaint? A. Yes, the seller should have had the items appraised before selling them. The seller always must know the value of what is being offered for sale. B. Yes, the seller cannot rescind a contract just because he later regrets it. Either party may end up with a superior deal depending on luck, knowledge, and a variety of factors. C. No, because in some unilateral mistakes, rescission is allowed if there had been an unconscionably unequal exchange of values and there is no substantial hardship to the other party. D. No, because a party always has a right to rescind a contract where the value of the product or services does not live up to his reasonable expectation. Question #023. OA68: Did Aaron Hernandez Cash In By Committing Suicide? (w/guest Chris Kristofco) A young woman joined a social media service that provided networking with other business professionals and entities nationwide. The woman discovered that the service was selling her personal profile information and the information pertaining to thousands of other members to third-party purchasers for tracking their Internet activities and buying habits. In her class action breach of contract action against the service, she alleged two theories of damages. First, she contended that she and the class members suffered, quote, embarrassment and humiliation from the disclosure. And second, that she and the others must be compensated for the market value of the information seized. Will the court likely recognize these allegations as sufficient to state a claim for breach of contract? A. Yes, because the service breached the duty of good faith inherent in every contract. B. Yes, because these are common types of damages that are authorized in breach of contract cases. C. No, because Internet laws protect absolutely all communications and downloads of this nature as a part of the service's guaranteed scope of free speech rights. D. No, because humiliation is not an item of damages in a breach of contract case, and the unauthorized collection of personal data does not establish an economic loss. Question #024. OA70: Donald Trump & Obstruction of Justice - Are We at the Peak of Yodel Mountain? a dance student worked very hard to strengthen her legs, including performing additional activities such as leaping and bounding up multiple flights of stairs in her apartment building. While she was performing her exercises, one of the janitors in the building warned her that "she was bound to get hurt if she keeps it up because there are problems with that floor." So janitor warns her you're bound to get hurt. After falling and breaking a leg due to the loose floorboard, she brought suit against the building owner. The building owner wants to testify that "The janitor told me I told her that running too fast is dangerous and she was bound to get hurt." Is the building owner's statement likely to be held admissible? A. Yes, because it is an exception to hearsay as a spontaneous declaration to an opponent party. B. Yes, because the building owner can testify and be questioned about what the janitor told him. C. No, because it is irrelevant and immaterial. D. No, because it is hearsay within hearsay, and there are no hearsay exceptions that apply. Question #025. OA72: Body Slamming Journalists PLUS Political vs. Racial Gerrymandering A vehicle operator was driving on a highway at night. His girlfriend was in the passenger seat. They were returning from a college night class. She was smoking marijuana, blowing it out the open window and not sharing it with the driver. The driver fell asleep, causing the car to veer off the highway and go into a ditch where it landed upside down. Both the driver and the passenger were seriously injured. She sued the operator for negligence. The operator filed a counterclaim asking for damages against the passenger by arguing that the passenger was herself negligent by smoking weed and causing him to fall asleep. Is the court likely to grant the passengers motion to dismiss the counterclaim? A. Yes, because the passenger was not actively participating in a joint enterprise and did not do enough to contribute to the cause of the accident. B. Yes, because a vehicle operator is always liable and a passenger cannot be made liable for the driver's actions under any circumstances. C. No, because the driver has properly alleged a joint enterprise situation where the passenger is liable for her share of the negligence in causing the accident. D. No, because the passenger was breaking the law, making her negligent per se. Question #026. OA74: Sippin’ #Covfefe With Trump’s Severed Head A purse snatcher ran up to a woman grabbed her purse and ran off there was a witness at the scene the witness worked with the police a few days later to try and create a composite sketch of the purse snatcher boy that is a tough word to say. This was used by the police to capture the purported purse stature. The witness died several days before the trial. When the witness's drawing was offered by the prosecution as evidence at trial, is the trial judge likely to accept it as admissible evidence? A. Yes, because the police took it and used it to find someone. Therefore, it must be an accurate representation of the perpetrator. The exhibit must be admitted. B. Yes, because it is in the nature of the written recollection of the decedent, at a time close to the crime, and therefore has testimonial reliability. C. No, because the defense has had no opportunity to cross-examine the witness under oath about the drawing prior to her death, and such sketches are inherently unreliable in any event. D. No, because it was drawn up days after the event and is in violation of the best evidence rule. Question #027. OA76: "I Hope" James Comey’s Senate Testimony Shows Obstruction of Justice A man was a passenger in a vehicle stopped at a red light. A drunk driver with a blood alcohol reading four times the legal limit rammed into the rear of the stopped vehicle without breaking and at a high speed. The passenger was severely and permanently injured. At trial, the drunk driver admitted liability so that the trial dealt with damages only. The drunk driver testified that he had no assets to pay a punitive damages award. He said that all he had was $100,000 in liability coverage. The court allowed the injured man's attorney to cross-examine the driver about his insurance coverage, and he elicited an admission that the driver knew that his company might have to pay the full award of the jury. The jury entered a verdict for $200,000 in compensatory damages and $500,000 in punitive damages. Defendant appeals, claiming that it was improper to question on the amount of insurance coverage or to speculate on whether the full amount may have to be paid. Will the appellate court likely affirm the award of punitive damages? A. No. The punitive damages award cannot be excessively high and cannot exceed three times the compensatory damages amount. B. No, because it is never proper to bring insurance information into the case against the defendant. C. Yes, because defendant opened the door to that kind of cross-examination by testifying that he had no assets to pay a punitive damage award. D. Yes, because the jury always has a right to be informed of the insurance coverage pertaining to each party in a tort case for personal injury damages. Question #028. OA78: Jeff Sessions, "Preemptive Executive Privilege," & More on Emoluments An owner of a pizza and Italian foods restaurant opens a new location on a street where another owner has a burger restaurant. It seems like business is slow for the pizza place owner and he eyes up his competitor, the burger man, as the reason. The pizza owner starts to make false statements about the burger restaurant and its inferior ingredients, which he says have been causing domain poisoning in some of the customers. When the burger joint owner loses customers and business income to the pizza place owner, can he sue for conversion? A. Yes, because the pizza owner interfered with his right of peaceful enjoyment and disrupted his business activities. B. Yes, conversion is an intentional tort and the pizza owner acted intentionally to convert and did convert part of the burger business. C. No, because conversion consists of appropriating another's real property. D. No, because there was no interference with his ownership or right of possession to his personal property. Question #029. OA80: Flashback Friday (featuring Health Care, The Slants, and Gerrymandering!) At a metro station in the city, there's always significant construction going on, including a rebuild of the elevator shaft between the platform and the ground level. There's a sign warning, "This elevator is not available at this time. Please do not ride it." A passenger sees the signs but tries to ride the elevator anyway and is injured when she falls into the shaft. When the passenger sues the city for negligence, the city states that she assumed the risk. Is that a valid and accurate defense under the circumstances? A. Yes, because she was fully informed and should have known that the elevator was unsafe based on the signs. B. Yes, because the signs make her contributorily negligent and a legal cause of her own injury. C. No, because the sign did not explain the danger and only told her that it was not available and therefore she did not know what she was encountering. D. No, because the city has strict liability when everyone is depending on the subway. Okay, okay. Question #030. OA82: Trinity Lutheran, Trump’s Executive Order & More (w/guest Andrew Seidel) A teenage male was on trial on charges of illegally entering an owner's property and stealing a car. The witness can only confirm for the prosecution that a car was stolen, the model and type of car, and that it was missing from the property on a date certain. The defense asks the witness, isn't it possible that my client did not steal the car, that someone else did it, and that the police are lying about the charges against him? The prosecution objected to the question. Will the court likely sustain the objection? A. Yes, because this is an objectionable question based on pure speculation. B. Yes, because it is not permissible to question the credibility of the police without substantial supporting evidence. C. No, questioning about the possibility of other perpetrators or about police misconduct is always permissible. D. No, because the witness may know something that he is not revealing, and for that reason a cross examiner is given wide latitude in questioning. Question #031. OA84: #CNNBlackmail, John Oliver’s lawsuit, and more on Maajid Nawaz The buyer asked a manufacturer to make him 100,000 widgets for $1 million. The buyer intended to retail them by mail order sales. The widgets had to have the buyer's logo imprinted on them. Manufacturer ordered the raw material and made a new widget mold that contained the buyer's logo. The buyer wanted quick delivery of at least 10,000 widgets right away. So the manufacturer rushed the order and sent 10,000 conforming widgets two days later. The buyer paid for the widgets but rejected, so the 10,000 widgets, but rejected delivery on the remaining 90,000 widgets. The manufacturer filed suit and claimed damages for breach of contract. The defendant buyer filed a cross motion to dismiss the complaint on the basis that the statute of frauds was violated in that it was a contract for over $500 that was unenforceable under state law. Will the court likely grant the motion to dismiss? A. Yes, the statute of frauds must be enforced when it involves goods over $500. B. Yes, the manufacturer did not protect its interests by agreeing to an oral contract, and it must pay for its bad business practices. C. No, because there is an exception for goods that are specially made for a buyer. D. No, the law will enforce the benefit of the bargain rule. Question #032. OA86: If Donald Trump, Jr. Commits Treason, Is It A Mini-Yodel? Two women lived in a one bedroom house with 20 cats. A neighbor who entered the house was appalled by their deplorable living conditions. The animal's excrement was deposited throughout the house. They were kept in tiny pens and some of the animals were sick. The visitor called the SPCA, a private nonprofit humane society. State law authorized humane societies to receive abused or neglected animals from animal control officers, care for them, and adopt them out. The SPCA took the animals designated by the officers and the officers then arrested the women for animal neglect. At a hearing, the prosecutor was unprepared and the judge dismissed the charges. The women then sued the officers and the SPCA under 42 USC 1983 for civil rights violations. The SPCA defendants filed a motion to dismiss based on the absence of state action on their part. Will the court likely grant the motion to dismiss? A. Yes, because there was nothing that was done by the Humane Society that involved police action. B. Yes, because the SPCA personnel did not at any time act "under color of law in their actions". C. No, because the SPCA was organized under a non-profit certificate of incorporation that was issued by the state. D. No, because the SPCA acted under the authority of state statute and cooperated with state authorities to perform the authorized functions. Question #033. OA88: Noah’s Ark & How Private Is The Stuff You Do On Your Computer? A small town police officer pulled over a driver for speeding. He believed that the driver was acting irritable and fidgy but had no articulable reason to think anything was wrong. He searched the car anyway and found two cartons of freshly canned peaches which were owned by the driver's neighbor and recorded stolen off her porch 24 hours earlier. Authorities charged him with theft under the state criminal code. His motion to suppress the evidence because of an unlawful search was denied. On appeal, will the appellate court likely reverse the lower court decision denying the motion to suppress? A. Yes, because the search was unconstitutional due to the officer having no reasonable suspicion that would justify searching the car. B. Yes, because when a car is pulled over for speeding, the officer must always obtain a search warrant prior to making a search. C. No, the stop and search were within the normal bounds of propriety for a speeding stop. D. No, because the driver being fidgety is enough for probable cause for a full search of the vehicle. Question #034. OA90: Pardon Me? Yes, Donald Trump Can Pardon Himself A college student attended a fraternity party. He noticed a young woman flirting with him in a sexually suggestive way. He took her home and ended up forcefully raping her against her will. She escaped after the rape and went directly to the authorities. The police filed charges of rape. The student defendant found several men willing to testify that the woman was promiscuous, that they had relations with her, and that she was very prolific in her sexual life. Will the trial court likely permit the witnesses to testify to the woman's sexual predisposition under the federal rules of evidence? A. Yes, because the rules permit evidence of a victim's sexual predisposition to test credibility. B. Yes, because the evidence will help the jury see that there may be two sides to the story and that the woman was thus the sexual predator instead of the man. C. No, because such sexual predisposition evidence is generally prohibited by the rules of evidence except under narrow exceptions. D. No, because the rules of evidence prohibit all evidence of a victim's prior sexual history. Question #035. OA92: The Unfortunate Application of Statutes of Limitation and Davino Watson A man who was emotionally upset due to marital problems voluntarily checked into a private medical facility for treatment. State law provided that if he tried to leave against medical advice, the hospital could hold him for 72 hours while trying to convince him to stay. During the stay, he told his medical providers that he was intensely angry at his wife, who was living with another man, and he wanted to harm her. Despite that knowledge, the hospital issued him an unrestricted weekend pass. When he got out, he shot and killed his wife. The man's children sued the hospital for negligence. The hospital filed a motion to dismiss, mainly because there was no privity with the decedent and no duty owed to her. Will the court grant the motion to dismiss? A. No, because the hospital is strictly liable for releasing patients who it knows or should know represent an unreasonable danger to third persons. B. No, because the special relationship and the control between doctor and an inpatient mandates due care by the doctor to prevent a foreseeable danger to others. C. Yes, because there is no duty to control the conduct of another person to prevent that person from harming others. D. Yes, because the intervening acts of third parties are unforeseeable and cannot give rise to liability. Question #036. OA94: Geoff Blackwell, Trump’s Anti-Trans Tweets & the Google Manifesto A man published a website wherein he billed himself as a crusader for justice and a hero with special powers. He wrote mainly about consumer issues while acting like a watchdog exposing wrongdoers. He wrote one article about a female preacher who had a local blog on the Internet. He wrote that she was "conspiring with the devil" to figure out how to, "lead people's minds astray", and to live a nice, "sheltered role of pure moral superiority at the expense of inferiors" like himself and his readers. He called the woman's fashion medieval and her hairstyle was like a bag of, "hot wires ready to electrocute anyone who dares come too close". The preacher was not amused and she filed a defamation action claiming libel per se and demanding damages along with an injunction against the website. The defendant filed a motion to dismiss. Will the court likely grant the motion to dismiss? A. Yes, because the defendant was expressing opinions more than factual assertions, and opinions in this context are constitutionally protected. B. Yes, because statements about religious beliefs or practices cannot be the subject of private defamation actions. C. No, because the preacher's allegations are based on serious factual distortions that will destroy her career if she doesn't get protection. D, No, because a defamation action cannot be subject to a motion to dismiss. The plaintiff is always entitled to present evidence to prove the tort. Question #037. OA96: Understanding Charlottesville A retailer agreed to purchase an orchard's total cherry production for a 12 month period. The contract included this provision, "terms, cash upon delivery, deliveries to be made at least twice per month". The deliveries went as scheduled. At the beginning of the fifth month, the orchard sent notice to the retailer that a load of cherries was ready to pick up. The truck driver, however, forgot to take the retailer's check with him. The orchard turned over the cherries but advised the retailer to pay within three days. The retailer sent the check by mail, but it arrived late by two days. The orchard refused to deliver any more cherries. The retailer sued the orchard for breach of the installment contract and demanded the difference in price between the cherries on the open market and what he would have paid under the contract. The orchard claimed that the delayed payment impaired the value of the contract and moved to dismiss. Will the court grant the orchard's motion to dismiss? A. Yes, the agreement called for cash at delivery, which was not done when the driver forgot to deliver the check. B. Yes, the failure to make an installment on time was a breach because time is of the essence in all installment agreements. C. No, there is always a grace period of 10 days for any single payment pertaining to an installment contract. D. No, the delay in one payment under these facts was not enough to impair the value of the contract under the uniform commercial code. Question #038. OA98: More Sovereign Citizen Madness! A witness in a murder case does not make the same statement faithfully, but rather he is given different versions of his observations at different times to different persons and investigators. The prosecution put the witness on the stand at trial to tell what he observed. The defendant's counsel impeached his testimony by bringing up prior inconsistent statements and accusing him of changing his story for trial. The prosecution then attempted to rehabilitate his credibility by referencing prior consistent statements. Prior consistent statements are not generally admissible because they are said to be repetitive, cumulative, and to unfairly bolster the witness's credibility. Will the court likely allow the prosecution to rehabilitate the witness using prior consistent statements under these facts? A. No, because the witness cannot be rehabilitated once a successful impeachment has occurred. B. No, because it would tend to confuse the jury with too much conflicting evidence. C. Yes, because it is being used to rehabilitate a witness whose credibility was attacked. D. Yes, because all repetitive prior statements are important to show the consistency of the witness's testimony. Question #039. OA100: Trump’s Trans Ban & Arpaio Pardon A foreign visitor was on trial for kidnapping a small child. The prosecutor stated that the visitor knew the child personally, which is why the child went with him, and that the perpetrator knew the child's parents had money. The prosecutor called a witness to testify that the perpetrator told the witness, I am looking forward to visiting with this child and his parents. They have a wonderfully luxurious house that I will enjoy staying in. The defense objected to the proposed testimony. Will the court likely sustain the objection? A. No, the statement is admissible to impeach the accused and establish that he is lying on cross if he takes the stand. B. No, the statement can come in as impeachment if the accused takes the stand and as a party admission to show the material facts of knowing the child and that the family had money. C. Yes, the prejudice of the statement will outweigh its probative value. D. Yes, the statement is irrelevant to the issue of guilt or innocence. Question #040. OA102: The Utah Nurse, DACA, & Disaster Relief The state charged the accused with the intentional murder of a former girlfriend. He admitted to killing her, but asserted that he lacked the intent to murder due to his suffering from a personality disorder complicated by two days of drinking alcohol heavily. The trial judge instructed the jury that, "the law presumes that a person intends the ordinary consequences of his voluntary acts". The judge did not tell them that they had a choice or could infer another conclusion. They were told only that the law presumed it. The accused was convicted of depraved heart murder and sentenced to 100 years in prison. On appeal, the appellant argued that the court took the fact-finding task of determining criminal intent out of the jury's hands by in effect ordering them to find intent. The jury could have felt that they were being told that they had no other choice. The State Supreme Court affirmed the conviction, but the United States Supreme Court granted certiorari. Will the court likely reverse the conviction? A. Yes, court will likely reverse the conviction because the wording of the instruction could have been viewed by jurors as a mandatory direction to find that there was intent to murder. B. Yes, because the wording in effect caused the defendant to be a witness against himself by having him intend everything that he did. C. No, because the defendant could raise an argument to rebut the permissible inference created by the jury instruction. D. No, because the instruction was only a presumption that the jury did not have to consider in its deliberations. So I think this is a really, really tough question. Question #041. OA104: Equifax, Class Actions, Sham Marriages & Redistricting! A detective found a footprint from a left foot shoe at a murder scene. The print was preserved appropriately as evidence. It had distinctive tread marks and an unusual wear pattern on the sole. It also had a V mark on the heel bottom that indicates the brand was a Victory shoe. The detective, armed with a proper search warrant, searched the suspect's apartment where he found a shoe to a right foot of the same size with a similar wear pattern and the same V mark as the shoe print found at the scene. The shoe for the left foot was not found, but the shoe for the right foot was seized from the suspect's closet and offered as evidence at trial. Is this admissible evidence? A. Yes, because it is direct evidence of guilt. B. Yes, because it is appropriate circumstantial evidence of guilt. C. No, because the shoe is irrelevant and will have no residue or other probative evidence relating to it. D. No, because footprint evidence is highly speculative unless both shoes and shoe prints are offered and authenticated. Question #042. OA 106: Elections Have Consequences! Donald Trump, Bernie Sanders & the DNC Lawsuit A man who was reportedly a renowned art thief, was on trial for stealing several Picasso's from a museum. The paintings had been recovered and were in the courtroom. The prosecution presented a professor of art history to authenticate each piece. The defense objected on the basis that the museum curator, where the paintings were housed, had to identify if these were in fact the paintings that were stolen. Assuming that others will testify to the fact of the burglary is the court likely to reject this witness? A. Yes, because artworks and other physical evidence must be authenticated by their official custodian. B. Yes, because an outside person may be able to partially authenticate the painting, but he can't say whether it was the one that was in the museum or that was stolen. C. No, granting the objection will simply waste the court's time if the court has to bring in the museum curator and go through hoops to authenticate the paintings. D. No, because the authentication can be accomplished in several different ways that will verify it is the genuine piece of art in question. Question #043. OA108: State-Sponsored Patriotism In the NFL & So Much More! The president and sole stockholder of a small business negotiated to sell that business to an individual investor. The parties met and signed a "letter agreement, which set forth the price down payment amount, financing terms, the assets to be included, and a host of other pertinent details". The final closing date was not stated, but the purchasers left a down payment of $20,000 with the seller. The letter stated that the terms comprised their essential agreement, but that a formal document would be drafted by the party's attorneys. Before that happened, the seller returned the check and refused to perform. The investor brought an equity action for specific performance to compel the sale. The equity court ruled that there was a complete and binding contract that must be specifically performed. Will the appellate court likely affirm the lower court's decision? A. No, because the closing date was left out and the attorneys were supposed to compose the final contract with all the remaining details put in. B. No because this type of letter is basically a gentleman's agreement that is not intended to be legally binding and C. Yes, because a letter of intent is legally binding if it sets forth the price, describes the business, and is signed by both parties. D. Yes, because where the letter contains all the essential terms required for a contract, it will be enforced as a contract. Question #044. OA110: Gun Control After Las Vegas & Two Trips To Yodel Mountain A female employee was fired as an executive assistant at the time when she was seven months pregnant. She brought a Title VII civil rights action in federal court for gender discrimination against the employer. At trial, a former co-worker of the woman was not allowed to testify that she heard the human relations manager saying in an informal meeting that the plaintiff and her predecessor were both fired due to what the manager described as, "pregnancy complications". The proposed witness had been a vice president of the company and a close advisor to the president. Without the benefit of that vital testimony, the jury entered a verdict for the defendant employer. On appeal, the plaintiff argued that the testimony should have been allowed and that it was not hearsay under the federal rules. Will the court likely grant the appeal and order a new trial? A. No, because the statement is clearly hearsay with no exception to the hearsay rule being applicable. B. No, because it was based on things the coworker heard from others, making it double hearsay that is never admissible. C. Yes, because that kind of testimony is reliable and not excludable as hearsay. D. Yes, because it was admissible under the spontaneous utterance exception to the hearsay rules. Question #045. OA112: Who’s Afraid of the FCC? The owner of a state licensed massage parlor sued the state for arbitrarily canceling its license and prior business permits to conduct a massage parlor at a given location in the city. Due to allegations of illegal activities in other massage parlors in the city, the owner's personal license and the business permits were revoked. The only procedure provided was a letter from the city declaring a new policy on massage parlors that had to be rapidly instituted for the welfare of the community. She sued the city and federal court, claiming that her business had been taken without just compensation in violation of the Fifth Amendment's takings clause. She also claimed a lack of procedural due process under the Fourteenth Amendment. The court granted a motion to dismiss based on her not having a property interest that was affected, including in the license and permits. She appealed the decision. Will the Federal Court of Appeals affirm the lower court's dismissal of her case? A. Yes, the Federal Court of Appeals will affirm the lower court's dismissal, saying that she has no case, because there is no right to earn a living under the substantive due process clause. B. Yes, because the state interest in protecting the public from illegal activities required that state action that was taken. C. No, because once the parlor had opened with all the proper permits, it could not be deprived of its licenses for any reason until they were up for renewal. D. No, because she had a property right in her license and permits, which were taken without any procedural due process. Question #046. OA114: Presidential Powers - Obamacare and the Travel Ban Two people who have been cohabiting decide to get married. The man asks the woman to agree that all of his property that he now owns will remain in his name if they ever get divorced. She agrees to that but demands a written document. However, they got married without having signed anything. Three months later, the husband leaves the wife for another woman. He sends her notice to vacate the home due to their prenuptial agreement. When she doesn't move, the husband sues, asking for enforcement of the oral prenuptial contract. Will the court likely enforce the husband's claim? A. Yes, because the consideration for the contract was the marriage and the consummation of the marriage made the contract complete and enforceable. B. Yes, because prenuptial agreements are given the benefit of the doubt when it comes to enforcement decisions. C. No, because a contract in consideration of marriage must be in writing and the prenuptial was therefore not legal as long as it remained oral. D. No, because husband did not live up to his promise to put the agreement in writing and therefore was guilty of fraud. Question #047. OA116: Hillary Clinton, Donald Trump & The Russians - Election Law (w/guest Beth Kingsley) A tenant's four-year-old daughter fell from a second floor outdoor stairway at their apartment building. The tenant sued the landlord for negligent construction and maintenance of the stairway, steps, and railing. The stairs were built on a dangerously steep incline, the steps were loose, and the railing was insufficient to keep the baby from falling over the side. The jury found that the landlord was negligent in the design and construction of the stairs and in failing to maintain the stairs, steps, and railing in a safe condition. The landlord appealed on the basis of immunity from suit and the failure of the tenant to assert any exception to landlord immunity. Under the modern rule, will the appellate court affirm the jury's verdict against the landlord? A. Yes, the landlord is strictly liable for the abnormally dangerous condition that he knowingly allowed to exist. B. Yes, the modern rule is that the landlord, like anyone else, has a duty to use due care to protect its tenants and others from injury caused by a foreseeable risk of harm. C. No, because the landlord still enjoys the common law protection of the doctrine of laissez-b'ouir, and residential property is generally still rented as is. D. No, because a landlord cannot be held liable for negligent design or maintenance even under the modern rule. Question #048. OA118: Indictment Monday & the View From Yodel Mountain Several persons together stole a painting from an art museum. One of them, who was the driver, was caught during the commission of another crime. He told the police, I didn't even get any money from that art museum. It all went to the first guy and the second guy. When the first guy and the second guy go on trial for the theft of the art painting, is the driver's statement admissible in court? A. Yes, as an admission by a party opponent. B. Yes, as a statement against the interests of the other defendants. C. No, because it was made while in custody and is therefore unreliable. D. No, because the statement was not made in the course of or in furtherance of the conspiracy and thus it is hearsay. Question #049. OA120: OA Shills For Monsanto! (w/guest Natalie Newell of "Science Moms") A furniture dealer sold furniture to a young couple with less than perfect credit. I like the commercial start of this. They signed a contract that said that if they purchased new items on the account, they would not own the old purchases until the new ones were paid in full. That provision was in fine print on the reverse side of the papers. When the husband lost his job, they had by that time paid for everything purchased on the account, except for one share bought a few weeks earlier. The store sued, trying to repossess all the furniture ever sold to the couple. Will the couple likely prevail on a defense of unconscionability? A. No, because they might be able to find another store to sell them furniture, which proves that there was no lack of bargaining power. B. No, because the store was nice enough to extend credit and the couple should be expected to pay for everything before they own any of it. That's the libertarian answer. C. Yes, because any time a seller puts terms in fine print, it is proof of bad faith and unconscionability. D. Yes, because under a combination of factors, makes it likely that the court will recognize unconscionability under these circumstances. Question #050. OA122: Moore is Less Cruiser and Flash were driving down Main Street in their distinctively painted, carefully restored old American automobiles, when Cruiser's vehicle bumped lightly into Flash's car, scraping away some paint. Cruiser and Flash both got out of their cars and examined the damage, after which Flash made some disparaging remarks about Cruiser's car. Cruiser slapped Flash in the face. Flash punched Cruiser in the nose. Cruiser seized Flash and a half Nelson and started to ram Flash's head into the side of Flash's car. At this moment, Flash's brother, Glitter, leaped from his car, pulled a knife, held it to Cruiser's throat saying, let him go or I'll cut you. Cruiser released flash, but subsequently sued glitter for assault. What is the result? A. Cruiser wins because Glitter threatened him with deadly force. B. Cruiser wins unless Ramming Flash's head into a car is considered a deadly attack. C. Glitter wins because Cruiser started the fight by slapping Flash. D. Glitter wins because he and Flash are relatives. Question #051. OA124: Happy Thanksgiving! A law of the state of Tentucky provides that all persons who have been residents of Tentucky for more than three years shall be entitled to free tuition at state-supported institutions of higher education. It further provides the persons who have resided in Tentucky for three years or less shall pay the non resident tuition rate, which is four thousand dollars per year. The book is written on golden plates. Sebastian was a student at Kentucky State University and a resident of Kentucky for a little more than two years. Sebastian was annoyed that he was required to pay tuition and filed a class action suit on behalf of himself and other university students with less than three years residency in federal district court for a declaratory judgment as to the constitutionality of the Kentucky statute. In eight months, when the case actually came to trial, Sebastian had been a resident of Kentucky for more than three years and was not required to pay tuition at the state university. However, Sebastian's case generated a great deal of interest and a number of amicus curiae briefs were filed with the court, some of them supporting and some of them opposing Sebastian's position. Should the state's motion to dismiss the case be granted? A. Yes, because Sebastian is now a three-year resident. B. Yes, because Sebastian lacks standing. C. No, because amicus curiae briefs have been filed. D. No, because there is a live controversy. Question #052. OA126: Mick Mulvaney & The Consumer Financial Protection Bureau Congress passed a law imposing a 50% excise tax on each pack of cigarettes manufactured for sale in the United States. An amendment was successfully added to the original bill requiring that all proceeds from the tax be used for anti-smoking educational programs in audio, video, and print media and elsewhere. The amendment also provided for the establishment of federal stop-smoking clinics funded through the excise tax. The various tobacco companies were required to pay the tax directly to the federal government. Puffham Tobacco Company filed suit in the appropriate federal court, praying that the tax be struck down as unconstitutional. The court is likely to rule: A. The tax is constitutional because the tax is severable from its purpose. B. The tax is constitutional because the broad provisions of the general welfare clause would condone it. C. The tax is unconstitutional because it does not provide equal time for the tobacco companies to present their side of the smoking controversy. D. The tax is unconstitutional because it abridges the First Amendment rights of tobacco manufacturers by forcing them to pay for messages they may not agree with. Question #053. OA128: Antitrust, Part Two During the course of his trial for assault, Danny placed professional sociologist Sato on the stand. Sato testified that he had scientifically pulled the community in which Danny lived and that Danny had a high reputation for being a peaceable man in a rather rough community. On cross-examination, the prosecutor asks Sato: "isn't it true, Mr. Sato, that last year you filed a false income tax return"? The defense immediately objected. Should the court require Sato to answer the question posed to him? A. Yes, but only if Sato has been convicted of filing a false tax return. B. Yes, because the question is relevant to the truthfulness and credibility of the witness. C. No, because specific instances of conduct are inadmissible. D. No, because the question does not go to a relevant character trait. Question #054. OA130: California on Fire Pluto went to Amy, an attorney, who agreed to file suit on his behalf on a contingent fee basis. Very common practice. Amy sent Pluto to see Dr. Sawbones, a physician, for the purpose of examining Pluto prior to trial and assessing the extent of his injuries. During the course of the examination, Pluto made some statements to Sawbones indicating that he was not completely free from negligence when the accident occurred. Duck seeks to call Sawbones to testify to the statements Pluto made to Sawbones. Amy objects. The objection by Amy should be: A. Overruled because Pluto made an admission. B. Overruled because a physician qualifies as an expert witness. C. Sustained because the attorney-client privilege applies. D. Sustained because the physician-patient privilege applies. Question #055. OA132: The Thomas Show! Can He Serve on the Federal Bench? Why is His High School Crazy? & More! Ted asked Steve if he could borrow Steve's boat for a day cruise. Steve agreed, but instructed Ted to stay inside the bay. Ted cruised around the bay, then, because the ocean was so calm outside the bay, crossed a mile of open ocean to the Blue Oyster Bar on the docks at Santa Patagonia Island. While Ted was docked, another boat passed the dock at an illegally excessive speed and its wake washed onto Steve's boat, causing water damage to the upholstery of about $150. Ted returned the boat directly to Steve without further incident. What, if any, recovery can Steve have against Ted resulting from Ted's use of Steve's boat? A. Steve can recover nothing because Ted was not responsible for the water damage to the upholstery. B. Steve can recover the cost of repairing the upholstery because it was damaged when Ted took the boat out of the bay. C. Steve can recover the value of the boat because Ted was in dominion and control of the boat when it was damaged. D. Steve can recover the value of the boat since Ted violated his instructions about staying in the bay. Question #056. OA134: Do Intergalactic Extraterrestrial Anchor Babies Use Cryptocurrency? One night after Devereaux had been in California for about six months, he missed his girlfriend in Texas so badly that he wanted to leave California immediately. Lacking cash for the trip, Devereaux approached his friend Sam for a loan. Sam declined. Devereaux then went to his neighbor, Hugh, and offered to buy Hugh's used stove for $50. Devereaux wrote Hugh a check for $50 for the stove. Devereaux then sold the stove to Sam for $50 and left for Texas. Having discovered that Devereaux's account was closed and that Sam had his stove, Hugh sued Sam for conversion. Which of the following facts, if true, would give Hugh the best chance of recovery against Sam? A. Devereaux knew his account was closed at the time he wrote the check to Hugh. B. Sam knew Devereaux had closed his bank account. C. Sam knew Devereaux had given Hugh a check not covered by sufficient funds. D. Sam intended to make a gift of the stove to his sister. Question #057. OA136: Chevron Deference Has Consequences - Particularly For Paul Manafort! Guy was hiking in the High Sierra in April, when a severe late season blizzard set in. He became lost in the snow and wandered for several hours trying to follow whimpering creek downstream. Fortunately, he came across a small seemingly unoccupied cabin, which he entered for shelter by breaking the lock and then fell asleep. Several hours later Basil, the owner of the cabin, roared up on his snowmobile and finding Guy in the Cabin demanded that Guy leave. When Guy refused, pointing out that the storm as at its peak, Basil dragged Guy out into the snow and then closed and locked the door. Utterly by chance, Guy managed to find another cabin, but suffered severe frostbite leading to the loss of several toes. If Guy sues Basil who will win? A) Basil, because Guy refused to leave when asked. B) Guy, because Basil did not behave as a reasonable person under the circumstances. C) Guy, because the blizzard was a life threatening situation. D) Basil, because Guy damaged his Cabin. Question #058. OA138: Pot, Gerrymandering, and Net Neutrality Like millions of other Americans, Bueller was anxious to purchase one of the new Mitsusushi two-way wrist videophones, the Walk and Talk Man. He went to the Electron Store, a chain retailer of consumer electronic devices, to purchase one. The Electron Store paid $1,500 to the factory for the Walk and Talk Man and charged its customers $2,500. Each Walk and Talk Man had to be ordered from the factory in Japan. Orders were so plentiful that it was taking up to eight weeks to receive the devices after ordering. An Electron Store salesperson filled out a pre-printed form, which had been drafted by the Electron Store's corporate legal department, in which Bueller agreed to make a $300 down payment and to pay the remaining $2,200 upon delivery of the Walk and Talk Man to him. Bueller, as is customary for consumers in these cases, signed the form without reading it. When Bueller's Walk and Talk Man arrived from Japan six weeks later, Bueller refused to accept it and demanded his $300 down payment back. The Electron store immediately sold that Walk and Talk Man for $2,500 to Calistoga, who had also ordered one from the factory on the same terms as Bueller. The Electron store then brought an action for breach of contract against Bueller, who counterclaimed for the return of his $300. What will be the likely outcome of this action? A. Judgment for Bueller in the amount of $300, because the purchase agreement was an unconscionable adhesion contract. B. Judgment for the Electron Store for the incidental costs of sale, since it resold the Walk and Talk Man at no loss to itself. C. Judgment for the Electron Store in the amount of $700, plus reasonable overhead and incidental costs, because under the circumstances, loss of profit is the only measure of damages which will make the Electron Store whole. D. Judgment for the Electron store in the amount of $2,200 because it could sell every Walk and Talk man it could obtain from the factory. Question #059. OA140: DACA and More! John and Mary were camping just south of a river in the National Forest when a forest fighter was ignited nearby by lightning. In order to avoid the flames, John swam across the river and untied a canoe from a no trespassing sign. One of many were stuck in the riverbank about every 100 feet. In his haste, John destroyed the sign in freeing the canoe. John paddled the canoe back across the river to get buried. John and Mary headed west, hoping to find a public road. Ogre, the owner of the property, was out practicing his bow hunting skills when he saw the couple and assumed they were intending to camp on his land. An accomplished woodsman, Ogre trailed them. Intending to frighten them so that they would leave and not return, Ogre shot an arrow so that it would pass in front of the two campers and "thwack!" into the tree just before they walked by. At the moment of release, however, another lightning strike occurred very close by, startling Ogre so that his arrow passed closer than he had intended, narrowly missing Mary and hitting and cutting John's hand. Can Ogre recover for the damage done to his no trespassing sign? A. No, because John was privileged to enter onto Ogre's land to escape the forest fire. B. No, because the damage to Ogre's property was de minimis. C. Yes, because John was a trespasser. D. Yes, because John destroyed the sign. Question #060. OA142: The Opioid Crisis - A (Mostly) Non-Partisan Friday John and Mary were camping just south of a river in the National Forest when a forest fighter was ignited nearby by lightning. In order to avoid the flames, John swam across the river and untied a canoe from a no trespassing sign. One of many were stuck in the riverbank about every 100 feet. In his haste, John destroyed the sign in freeing the canoe. John paddled the canoe back across the river to get buried. John and Mary headed west, hoping to find a public road. Ogre, the owner of the property, was out practicing his bow hunting skills when he saw the couple and assumed they were intending to camp on his land. An accomplished woodsman, Ogre trailed them. Intending to frighten them so that they would leave and not return, Ogre shot an arrow so that it would pass in front of the two campers and "thwack!" into the tree just before they walked by. At the moment of release, however, another lightning strike occurred very close by, startling Ogre so that his arrow passed closer than he had intended, narrowly missing Mary and hitting and cutting John's hand. Can John recover against Ogre for battery? A. No, because John was a trespasser. B. No, because the lightning was an intervening, superseding force. C. Yes, because Ogre intended to frighten him. D. Yes, because John was escaping from the forest fire. Question #061. OA144: Our Football-Free Superb Owl Edition John and Mary were camping just south of a river in the National Forest when a forest fighter was ignited nearby by lightning. In order to avoid the flames, John swam across the river and untied a canoe from a no trespassing sign. One of many were stuck in the riverbank about every 100 feet. In his haste, John destroyed the sign in freeing the canoe. John paddled the canoe back across the river to get buried. John and Mary headed west, hoping to find a public road. Ogre, the owner of the property, was out practicing his bow hunting skills when he saw the couple and assumed they were intending to camp on his land. An accomplished woodsman, Ogre trailed them. Intending to frighten them so that they would leave and not return, Ogre shot an arrow so that it would pass in front of the two campers and "thwack!" into the tree just before they walked by. At the moment of release, however, another lightning strike occurred very close by, startling Ogre so that his arrow passed closer than he had intended, narrowly missing Mary and hitting and cutting John's hand. Can Mary recover against Ogre for assault? A. No, because she was not touched by the arrow. B. No, because Ogre believed she was a trespasser. C. Yes, because Ogre used excessive force. D. Yes, since Mary must have seen the arrow pass in front of her. Question #062. OA146: Clearing the White Board! For many years, Schnauzer, a supplier of dog food, had accepted orders for dog food from Chow, the owner of a pet store, over the telephone. One day, Chow telephoned Schnauzer and ordered 840 pound bags of dog food, priced at $20 per bag, to be delivered to his store at the rate of 100 per month starting in January. On January 1, Schnauzer delivered 100 bags. With his check-in payment for the delivery, Chow sent Schnauzer a note saying, "$20 per bag is too high. I will not take the other 700 bags signed Chow". Schnauzer files suit alleging that Chow promised to buy 800 bags of dog food from him. In his defense, Chow pleads the statute of frauds. How should the court rule on this issue? A. For Schnauzer, because the note sent with Chao's check constitutes a writing sufficient to satisfy the statute of frauds. B. For Chao because the note sent with Chao's check was a repudiation rather than a memorandum of the oral agreement. C. For Chao because the note sent with Chao's check does not satisfy the Statute of Frauds. D. For Schnauzer because Chao received and accepted part of the goods. Question #063. OA148: The Parkland Massacre Penny was riding an escalator in Dirk's department store when suddenly the escalator sped up and then came to a very quick stop, throwing people against each other and then down the escalator. Penny tried to hold onto the railing but was unsuccessful. She was thrown violently to the left. Felt a horrible pain in her back and then tumbled down the escalator to the bottom. Officials of Dirk's department store promptly called their industrial health nurse who examined Penny and summoned an ambulance. When the ambulance arrived, paramedics placed Penny on the stretcher. And just as they picked up the stretcher to take her to the hospital, Penny heard Clara, a customer of the store, say "yesterday I heard Cora, another customer, tell Dirk that the escalator was acting funny, you know, speeding up and stopping". Penny was treated at the hospital, must undergo rehabilitation treatment for her badly injured back. She now sues Dirk's department store for her injuries and at trial calls Clara, that's the customer she overheard, to testify as to Cora's remark. The lawyer for Dirks objects. How should the court rule? A. Admissible as a statement against interest. B. Admissible as relevant evidence that Dirks was aware of the defect and did nothing to correct it. C. Inadmissible unless Cora cannot be located as a witness. D. Inadmissible as hearsay not within any exception. Question #064. OA150: Janus, The Angry Roman God Of Doorways (And Labor Law?) Dubba's old Basenji Reb didn't like the new puppy and would hide from him during the day so as not to have his (Reb's) naps interrupted by an impromptu game of chew Reb's ear. One warm evening, the dog Reb hid in the tool shed and was so deeply asleep that he didn't awaken when Dubba, the owner, shut and locked the shed for the night. When he, again woke about midnight and discovered that he was locked in, Reb began jumping against the shed door. The loud banging of the tool shed door aroused Dubba from sleep. Thinking that his house was being burglarized, Dubba grabbed his loaded shotgun and tiptoed onto the back patio from where he heard the loud banging. In the dark of the patio, Dubba did not notice that his young daughter had left her jacks scattered about on the floor. When his bare feet encountered the jacks, Dubba was soon hopping in pain and fell over a patio table, crashing heavily to the floor. Upon impact, the shotgun went off, both barrels blasting a hole in the patio screen. The pellet struck Dubba's neighbor Jimmy, who had also heard the thumping in the tool shed, and climbed over the fence which separated his yard from Dubba's, and Jimmy was killed instantly. Which of the following is Dubba's strongest defense to a charge of involuntary manslaughter as a result of Jimmy's death? A. Jimmy assumed the risk by trespassing on Dubba's property at night. B. Dubba's actions that night did not constitute criminal negligence. C. Jimmy's death did not result from a volitional act by Dubba. D. Dubba was privileged to kill in defense of his home. Question #065. OA152: Discrimination is for Dick’s? Daphne was a committed vegan, eating only plant-derived foods. One night, in order to dramatize the unhealthy nature of meat and to draw attention to the slaughter of innocent animals, Daphne went to the butcher's section of the supermarket, where she was employed as a produce manager, and sprinkled a nausea-inducing chemical on all the meat, fowl, and fish products. Despite being careful, Daphne accidentally put too much powder in one package of hamburger. When the woman who bought it served it to her family, her husband, who was unusually susceptible, died as a result of ingesting the chemical. In Daphne's jurisdiction, murder is defined as "the unlawful killing of a human being with malice aforethought. Such malice may be expressed or implied. It is expressed where there is manifested a deliberate intention unlawfully to take away the life of another person. It is implied when no considerable provocation appears or when the circumstances attending the killing show an abandoned and malignant heart". All murder which is perpetrated by willful, deliberate, or premeditated killing, or which is committed in the perpetration of or attempt to perpetrate arson, rape, robbery, or burglary, is murder of the first degree. All other kinds of murders are of the second degree. If prosecuted for the criminal homicide of the husband who ate the tainted meat, Daphne should be found guilty of: A. First degree murder since she willfully, deliberately, and with premeditation killed the victim. B. First degree murder since the homicide occurred during the perpetration of a felony. C. Second degree murder since the circumstances show that she acted with an abandoned and malignant heart and let the record show that the circumstances also show that whoever wrote these bar questions did not intend for them to be read aloud. D. Involuntary manslaughter since she did not intend to kill anyone. Question #066. OA154: Stormy Daniels is a Legal Genius When the 20-year incumbent in the State Senate district representing her city retired, Judith, a city council member, filed as a candidate for that seat. She had the support of all the relevant politicos and was considered a sure winner until Kimberly, a local television personality, entered the race. Kimberly and Judith had been friends and Judith felt betrayed by Kimberly's candidacy, believing that Kimberly was using her name recognition to rob Judith of a position she'd been working towards for 15 years. The campaign was consequently a bitter one. Judith doing everything she could to portray Kimberly as an intellectual lightweight who was more concerned about sound bites than issues. And Kimberly painting Judith as an old style politico with plenty of favors owed and sleazy connections to unsavory elements of the power structure. In an attempt to lower the hostility level, local community leaders organized a charity barbecue one weekend and invited both Judith and Kimberly to be celebrity chefs. The candidates were placed at barbecue grills on a raised dais before the large crowd where they were each supposed to cook a mess of ribs using their own special barbecue sauces. While preparing to cook, Judith was using a large chef's knife to trim the ribs she intended to barbecue. The slab of meat slipped from her grasp and fell to the floor. Judith swung rapidly towards the meat and uttered a sincere damn when she saw she could not save it from the filthy floor. Kimberly turned at the same moment and, seeing Judith crouched, holding a large knife apparently pointed at her, and appearing very angry, Kimberly believed that Judith was trying to kill her. Kimberly grabbed her own chef's knife and plunged it into Judith's neck. At Kimberly's trial for the murder of Judith, Kimberly should be found guilty of: A. No crime, because Kimberly was justified in killing Judith, whom she reasonably believed was about to kill her. B. No crime, because Kimberly was excused in killing Judith, whom she reasonably believed was about to kill her. C. Voluntary manslaughter, because Kimberly unreasonably but in good faith believed that Judith was about to kill her. D. Murder, because the bitterness between the candidates demonstrates that Kimberly killed Judith with malice aforethought. Question #067. OA156: Conor Lamb & Pennsylvania Recounts Nine gang members, including the gang leader, were indicted for the murder of a 10th gang member who had become an informant. The gang leader pleaded guilty. At the trial of the other eight gang members, the state's evidence showed the following: The gang leader had announced a party to celebrate the release of a gang member from jail, but the party was not what it seemed. The gang leader had learned that the released gang member had earned his freedom by informing the authorities about the gang's criminal activities. The gang leader decided to use the party to let other gang members see what happened to informants. He told no one about his plan. At the party, after everyone had consumed large amounts of alcohol, the gang leader announced that the released gang member was an informant and stabbed him with a knife in front of the others. The other eight gang members watched and did nothing, while the released gang member bled slowly to death. The jury found the eight gang members guilty of murder, and they appealed. Should the appellate court uphold the convictions? A. No, because the mere presence at the scene of a crime is insufficient to make a person an accomplice. B. No, because murder is a specific intent crime and there is insufficient evidence to show that the other gang members intended to kill. C. Yes, because the gang members made no effort to save the released gang member after he had been stabbed. D. Yes, because voluntary intoxication does not negate criminal responsibility. Question #068. OA158: Cambridge Analytica In January, a teacher contracted with a summer camp, possibly CampQuest.org, to serve as its head counselor at a salary of $10,000 for 10 weeks, from the 1st of June to the middle of August. In March, the camp notified the teacher that it had hired someone else's head counselor and that the teacher's services would not be needed. In April, the teacher spent $200 traveling to interview at the only other nearby summer camp for a position as its head counselor. The teacher was not chosen for the job. The teacher then took a position teaching in a local summer school at a salary of $6,000 for the same 10-week period. How much is the teacher entitled to recover as damages in a breach of contract action against the camp? A. $4,000. B. $4,200. C. $10,000. D. $10,200. Question #069. OA160: Schrodinger's Andrew A driver returning home from a long work shift at a factory fell asleep at the wheel and lost control of his car. As a result, his car collided with a police car being driven by an officer who was returning to the station after having responded to an emergency. The officer was injured in the accident and later sued the driver in negligence for her injuries. The driver has moved for summary judgment, arguing that the common law firefighters rule bars the suit. Should the court grant the motion? A. No, because the firefighters rule does not apply to police officers. B. No, because the police officer's injuries were not related to any special dangers of her job. C. Yes, because the accident would not have occurred but for the emergency. D. Yes, because the police officer was injured on the job. Question #070. OA162: Tariffs and Trade A lumber supplier agreed to sell to a furniture manufacturer all the lumber that the manufacturer required over a two-year period. The sales contract provided that the payment was due 60 days after delivery, but that a 3% discount would be allowed if the manufacturer paid within 10 days of delivery. Now, during the first year of the contract, the manufacturer regularly paid within the 10-day period and received the 3% discount. But 15 days after the supplier had made its most recent lumbered delivery to the manufacturer, the supplier had received no payment from the manufacturer. At the time, The supplier became aware of rumors from a credible source that the manufacturer's financial condition was precarious. The supplier called the manufacturer demanding assurances regarding the manufacturer's financial status. The manufacturer immediately mailed to the supplier its latest audited financial statements, as well as a satisfactory credit report prepared by the manufacturer's banker. The rumors proved to be false. Nevertheless, the supplier refused to resume deliveries. The manufacturer sued the supplier for breach of contract. Will the manufacturer likely prevail? That's very interesting. A. No, because the contract was unenforceable since the manufacturer had not committed to purchase a definite quantity of lumber. B. No, because the supplier had reasonable grounds for insecurity and was therefore entitled to cancel the contract and refuse to make any future deliveries. C. Yes, because the credit report and auditory statements provided adequate assurance of due performance under the contract. D. Yes, because the supplier was not entitled to condition resumption of deliveries on the receipt of financial status. Question #071. OA164: As American As Baseball, Hush Money, and… Segregated Schools? A toy manufacturer that has its headquarters and sole manufacturing plant in a particular state. It has developed a Martian toy that simulates the exploration of Mars by a remote-controlled vehicle. It accurately depicts the Martian landscape and the unmanned exploratory vehicle traversing it. The toy is of high quality, safe and durable, and has sold very well. Other toy manufacturers, all located outside of the state, have developed similar toys that are lower in price. These manufacturers have contracts to sell their Martian toys to outlets in the state. Although these toys are safe and durable, they depict the Martian landscape less realistically than the toys manufactured in the state. Nevertheless, because of the price difference, sales of these toys, that is the cheap out-of-state knockoffs, have severely cut into the sales of the Martian toys manufactured in the state. The state legislature has recently enacted a law "to protect the children of the state from faulty science and to protect in-state toy manufacturers from unfair competition". The law forbids the sale in the state of any toy that purports to represent extraterrestrial objects and does not satisfy specified scientific criteria. The Martian toy manufactured in the state satisfies all the criteria. None of the Martian toys of the competing manufacturers meet the requirement. Is that state law constitutional? A. No, because it abrogates the obligations of the contracts between the other toy manufacturers and the in-state outlets that have agreed to sell their Martian toys. B. No, because it imposes an undue burden on interstate commerce. C. Yes, because it deals with only a local matter, the sale of toys in stores located within the state. D. Yes, because the state's interest in protecting the state's children from faulty science justifies this burden on interstate commerce. Question #072. OA166: The Taint Team (& Also, Alex Jones) A landowner executed an instrument in the proper form of a deed purporting to convey his land to a friend. The landowner handed the instrument to the friend saying "This is yours, but please do not record it until after I'm dead. Otherwise, it will cause me no end of trouble with my relatives". Two days later, the landowner asked the friend to return the deed to him because he had decided that he should devise the land to the friend by will rather than by deed. The friend said that he would destroy the deed and so a day or so later falsely told the landowner that the deed had been destroyed. Six months ago, the landowner, who had never executed a will, died intestine, survived by a daughter as his sole heir. The day after the landowner's death, the friend recorded the deed from him. As soon as the daughter discovered this recording and the friend's claim to the land, she brought an appropriate action against the friend to quiet title to the land. For whom should the court hold? A. The daughter, Because the death of the landowner deprived the subsequent recording of any effect. B. The daughter because the friend was dishonest in reporting that he had destroyed the deed. C. The friend because the deed was delivered to him. D. The friend because the deed was recorded by him. Question #073. OA168: Michael Cohen Takes Five A landlord leased an apartment to a tenant by written lease for two years, ending on the last day of a recent month. The lease provided for $700 in monthly rent. The tenant occupied the apartment and paid the rent for the first 15 months of the lease term until he moved to another city to take a new job. Without consulting the landlord, the tenant moved a friend into the apartment and signed an informal writing transferring to the friend his "lease rights for the remaining nine months of the lease". The friend made the next four $700 rent payments to the landlord. But for the final five months of the lease term, no rent was paid by anyone, and the friend moved out with three months left on the term. A landlord was on an extended trip abroad and did not learn of the default and the vacancy until the end of the lease term. The landlord has sued the tenant and the friend jointly and severally for $3,500 for the last five months rent. What is the likely outcome of this lawsuit? A. Both the tenant and the friend are liable for the full $3,500 because the tenant is liable on privity of contract and the friend is liable on privity of estate as an assignee. B. The friend is liable for $1,400 on privity of estate, which lasted only until he vacated, and the tenant is liable for $2,100 on privity of contract and estate for the period after the friend vacated. C. The friend is liable for $3,500 on privity of estate and the tenant is not liable because of the landlord's failure to object to the friend's payment of rent relieved the tenant of liability. D. The tenant is liable for $3,500 on privity of contract and the friend is not liable because a sublessee does not have personal liability to the original landlord. Question #074. OA170: All Yodel, All the Time In a civil trial arising from a car accident at an intersection, the plaintiff testified on direct examination that he came to a full stop at the intersection. On cross-examination, the defendant's lawyer asked whether the plaintiff claimed to have been exercising due care at the time, which is the standard for negligence, and the plaintiff replied that he had been driving carefully. At a sidebar conference, the defendant's lawyer sought permission to ask the plaintiff about two accidents in the previous 12 months in which he had received traffic citations for failing to stop at stop signs. The plaintiff's lawyer has objected. Should the court allow defense counsel to ask the plaintiff about his two prior accidents? A. No, because improperly failing to stop on the other occasions does not bear on plaintiff's veracity and does not contradict his testimony in this case. B. No, because there is no indication that failing to stop on the other occasions led to convictions. C. Yes, because improperly failing to stop on the other occasions bears on plaintiff's credibility since he claims to have stopped in this case. D. Yes, because improperly failing to stop on the recent occasions tends to contradict the plaintiff's claim that he was driving carefully at the time that he collided with the defendant. Question #075. OA172: Private Prisons, Judge Ellis & More A landowner and a contractor entered into a written contract under which the contractor agreed to build a building and pave an adjacent sidewalk for the landowner for $200,000. Later, while construction was proceeding, the landowner and the contractor entered into an oral modification under which the contractor was not obligated to pave the sidewalk, but still would be entitled to the $200,000 upon completion of the building. The contractor completed the building. The landowner, after discussions with his landscaper, demanded that the contractor pave the adjacent sidewalk. The contractor refused. Has the contractor breached the contract? A. No, because the oral modification was in good faith and therefore enforceable. B. Yes, because a discharge of a contractual obligation must be in writing. C. Yes, because the parole evidence rule bars proof of the oral modification. D. Yes, because there was no consideration for the discharge of the contractor's duty to pave the sidewalk. Question #076. OA174: Is Michael Avenatti Fit To Practice Law In New York? A defendant was charged with burglary. At trial, a police officer testified that, after the defendant had been arrested and had agreed to answer questions, the officer had interrogated him with a stenographer present, but that the officer could not recall what the defendant had said. The prosecutor presented the officer with a photocopy of the stenographic transcript of the interrogation. After looking at it, the officer began to testify that he recalled that the defendant had admitted to being in the area of the burglary. The defendant has objected to the officer's testimony on the ground that it violates the "original document" rule (also known as the "best evidence" rule). Should the officer's testimony concerning the defendant's recorded confession be admitted? A. No, because a photocopy cannot be used without a showing that the original is unavailable. B. No, because the stenographer has not testified to the accuracy of the transcript. C. Yes, because a photocopy is a duplicate of the original. D. Yes, because the prosecutor is not attempting to prove the contents of the document. Question #077. OA176: It's Summer (Zervos) Time! A state legislature passed a statute providing that juries in criminal trials were to consist of 6 jurors, rather than 12, and that jury verdicts did not have to be unanimous but could be based on 5 votes out of 6 jurors. A defendant was tried for murder. Over his objection, he was tried by a jury composed of 6 jurors. The jurors found him guilty by a vote of 5 to 1, and over the defendant's objection, the court entered a judgment of conviction, which was affirmed on appeal by the state’s highest court. The defendant seeks to overturn his conviction in a habeas corpus proceeding in federal court, claiming that the trial court violated his constitutional rights by allowing both a non-unanimous jury verdict and a jury composed of fewer than 12 members. How is the federal court likely to rule in this action? A. It will set aside the conviction, because the jury was composed of fewer than 12 members. B. It will set aside the conviction, because the 6-person jury verdict was not unanimous. C. It will set aside the conviction, both because the jury was composed of fewer than 12 members and because the 6-person jury verdict was not unanimous. D. It will uphold the conviction. Question #078. OA178: Trump and the NFL In a civil trial for professional malpractice, the plaintiff sought to show that the defendant, an engineer, had designed the plaintiff's flour mill with inadequate power. The plaintiff called an expert witness who based his testimony solely on his own professional experience but also asserted, when asked, that the book *Smith on Milling Systems* was a reliable treatise in the field and consistent with his views. On cross-examination, the defendant asked the witness whether he and Smith were ever wrong. The witness answered, "Nobody's perfect." The defendant asked no further questions. The defendant later called his own expert witness and asked, "Do you accept the Smith book as reliable?" The witness said, "It once was, but it is now badly out of date." The plaintiff requested that the jury be allowed to examine the book and judge for itself the book's reliability. Should the court allow the jury to examine the book? A. No, because the jury may consider only passages read to it by counsel or by a witness. B. No, because the plaintiff's expert did not rely on the treatise in his testimony but on his own experience. C. Yes, because an expert has testified that the treatise is reliable. D. Yes, because the jury is the judge of the weight and credibility to be accorded both written and oral evidence. Question #079. OA180: Masterpiece Cakeshop A grantor executed an instrument in the proper form of a warranty deed purporting to convey attractive land to his church. The granting clause of the instrument ran to the church, "and its successors forever, so long as the premises are used for church purposes". The church took possession of the land and used it as a site of worship for many years. Subsequently, the church decided to relocate and entered into a valid written contract to sell the land to a buyer for a substantial price. The buyer wanted to use the land as a site for business activities and objected to the church's title. The contract contained no provision relating to the quality of the title the church was bound to convey. There is no applicable statute. When the buyer refused to close, the church sued the buyer for specific performance and properly joined the grand tour as a party. Is the church likely to prevail? A. No, because the grantor's interest prevents the church's title from being marketable. B. No, because the quoted provision is a valid restrictive covenant. C. Yes, because a charitable trust to support religion will attach to the proceeds of the sale. D. Yes, because the grantor cannot derogate from his warranty. Question #080. OA182: Paul Manafort is Going to Prison A car owner washed her car while it was parked on a public street, in violation of a local ordinance that prohibits the washing of vehicles on public streets during specified hours. The statute was enacted only to expedite the flow of automobile traffic. Due to sudden and unexpected cold weather, the car owner’s waste water formed a puddle that froze in a crosswalk. A Pedestrian slipped on the frozen puddle and broke her leg. The pedestrian sued the car owner to recover for her injury. At trial, the only evidence the pedestrian offered as to negligence was the car owner's admission that she had violated the ordinance. At the conclusion of the evidence, both parties moved for a directed verdict. How should the trial judge proceed? A. Deny both motions and submit the case to the jury, because, on the facts, the jury may infer that the car owner was negligent. B. Deny both motions and submit the case to the jury, because the jury may consider the ordinance violation as evidence that the car owner was negligent. C. Grant the car owner's motion, because the pedestrian has failed to offer adequate evidence that the car owner was negligent. D. Grant the pedestrian’s motion, because of the car owner’s admitted ordinance violation. Question #081. OA184: Families at the Border A state university adopted a new regulation prohibiting certain kinds of speech on campus. Students, staff, and faculty convicted by campus tribunals of violating the regulation were subject to penalties that included fines, suspensions, expulsions, and termination of employment. The regulation was widely unpopular, and there was a great deal of public anger directed toward the two tenured professors who had drafted and promoted it. The following year, the state legislature approved a severable provision in the appropriations bill for the university declaring that none of the university's funding could be used to pay the two professors, who were specifically named in the provision. In the past, the professor's salaries had always been paid from funds appropriated to the university by the legislature, and the university had no other funds that could be used to pay them. If the professors challenge the constitutionality of the appropriations provision, is the court likely to uphold the provision? A. No, because it amounts to the imposition by the legislature of a punishment without trial. B. No, because it is based on conduct the professors engaged in before it was enacted. C. Yes, because the Eleventh Amendment gives the state legislature plenary power to appropriate state funds in the manner that it deems most conducive to the welfare of its people. D. Yes, because the full faith and credit clause requires the court to enforce the provision strictly according to its terms. Question #082. OA187: Lowering the Lukumi Bar? Police officers received a tip that illegal drugs were being sold at a certain ground-floor apartment. They decided to stake out the apartment. The stakeout revealed that a significant number of people visited the apartment for short periods of time and then left. A man exited the apartment and started to walk briskly away. The officers grabbed the man and, when he struggled, wrestled him to the ground. They searched him and found a bag of heroin in one of his pockets. After discovering the heroin on the man, the officers decided to enter the apartment. They knocked on the door, which was opened by the woman who lived there. The officers asked if they could come inside, and the woman gave them permission to do so. Once inside, the officers observed several bags of heroin on the living room table. The woman has been charged with possession of the heroin found on the living room table. She has filed a pretrial motion to suppress the heroin on the ground that it was obtained by an illegal search and seizure. Should the woman’s motion be granted? A. No, because the tip together with the heroin found in the man’s pocket provided probable cause for the search. B. No, because the woman consented to the officers’ entry. C. Yes, because the officers’ decision to enter the apartment was the fruit of an illegal search of the man. D. Yes, because the officers did not inform the woman that she could refuse consent. Question #083. OA189: Supreme Court Justice Brett Kavanaugh A host pointed an unloaded revolver at her guest, threatening to shoot him. The guest knew that the revolver was not loaded, and that the ammunition for the revolver was stored in a locked basement closet, two stories below where the two were then standing. In an action brought by the guest against the host for assault, will the guest be likely to prevail? A. No, because the host did not intend to shoot her guest. B. No, because the host did not put her guest in apprehension of an imminent contact. C. Yes, because the ammunition was accessible to the host. D. Yes, because the host threatened her guest with a revolver. Question #084. OA191: Fact and Fiction About Brett Kavanaugh A defendant has pleaded not guilty to a federal bank robbery charge. The principal issue at trial is the identity of the robber. The prosecutor has called the defendant's wife to testify to the clothing that the defendant wore as he left their house on the day the bank was robbed, expecting her description to match that of eyewitnesses to the robbery. Both the defendant and his wife have objected to her testifying against the defendant. Should the wife be required to testify? A. No, because the defendant has a privilege to prevent his wife from testifying against him in a criminal case. B. No, because the wife has a privilege not to testify against her husband in a criminal case, C. Yes, because the spousal testimonial privilege does not apply in criminal cases. D. Yes, because the wife's viewing of the defendant’s clothing was not a confidential communication. Question #085. OA193: This Is Worse Than Watergate - PLUS Mandalay Bay Suing Victims? A landowner died, validly devising his land to his wife "for life or until remarriage, then to" their daughter. Shortly after the landowner’s death, his daughter executed an instrument in the proper form of a deed, purporting to convey the land to her friend. A year later, the daughter died intestate, with her mother, the original landowner’s wife, as her sole heir. The following month, the wife remarried. The wife then executed an instrument in the proper form of a deed, purporting to convey the land to her new husband as a wedding gift. Who now owns what interest in the land? A. The daughter's friend owns the fee simple. B. The wife owns the fee simple. C. The wife's new husband has a life estate in the land for the wife’s life, with the remainder in the daughter's friend. D. The wife’s new husband owns the fee simple. Question #086. OA195: Lordy, There Are Tapes! During negotiations to purchase a used car, a buyer asked a dealer whether the car had ever been in an accident. The dealer replied: "It is a fine car and has been thoroughly inspected and comes with a certificate of assured quality. Feel free to have the car inspected by your own mechanic." In actuality, the car had been in a major accident, and the dealer had repaired and repainted the car, successfully concealing evidence of the accident. The buyer declined to have the car inspected by his explaining that he would rely on the dealer's certificate of assured quality. At no time did the dealer disclose that the car had previously been in an accident, The parties then signed a contract of sale. After the car was delivered and paid for, the buyer learned that the car had been in a major accident. If the buyer sues the dealer to rescind the transaction, is the buyer likely to succeed? A. No, because the buyer had the opportunity to have the car inspected by his own mechanic and declined to do so. B. No, because the dealer did not affirmatively assert that the car had not been in an accident. C. Yes, because the contract was unconscionable. D. Yes, because the dealer's statement was intentionally misleading and the dealer concealed evidence of the accident. Question #087. OA197: Undetectable, Untraceable, 3-D Printed Guns A state constitution provides that in every criminal trial "the accused shall have the right to confront all witnesses against him face to face." A defendant was convicted in state court of child abuse based on testimony from a six-year-old child. The child testified while she was seated behind one-way glass, which allowed the defendant to see the child but did not allow the child to see the defendant. The defendant appealed to the state’s highest court, claiming that the inability of the child to see the defendant while she testified violated both the United States Constitution and the state constitution. Without addressing the federal constitutional issue, the state’s highest court reversed the defendant's conviction and ordered a new trial. The court held that "the constitution of this state is clear, and it requires that while testifying in a criminal trial, witness must be able to see the defendant." The state petitioned the United States Supreme Court for a writ of certiorari On which ground should the United States Supreme Court DENY the state's petition? A. A state may not seek appellate review in the United States Supreme Court of the reversal of a criminal conviction by its highest court. B. The decision of the state’s highest court was based on an adequate and independent state ground. C. The Sixth Amendment to the United States Constitution does not require that a witness against a criminal defendant be able to see the defendant while the witness testifies. D. The decision of the state’s highest court requires a new trial, and therefore it is not a final judgment. Question #088. OA199: Asbestos??!? (Or: Why Is This Man Smiling?) On January 5, a creditor loaned $1,000 to a debtor under a contract calling for the debtor to repay the loan at the rate of $100 per month payable on the first day of each month. On February 1, at the debtor's request, the creditor agreed to permit payment on February 5. On March 1, the debtor requested a similar time extension and the creditor replied, "Don’t bother me each month. Just change the date of payment to the fifth of the month. But you must now make the payments by cashier's check." The debtor said, "Okay," and made payments on March 5 and April 5 by cashier's check. On April 6, the creditor sold the loan contract to a bank but did not tell the bank about the agreement permitting payments on the fifth of the month. On April 6, the bank wrote to the debtor: "Your debt to [the creditor] has been assigned to us. We hereby inform you that all payments must be made on the first day of the month." Can the debtor justifiably insist that the payment date for the rest of the installments is the fifth of each month? A. No, because a contract modification is not binding on an assignee who had no knowledge of the modification. B. No, because although the creditor waived the condition of payment on the first of the month, the bank reinstated it. C. Yes, because although the creditor waived the condition of payment on the first of the month, the creditor could not assign to the bank his right to reinstate that condition. D. Yes, because the creditor could assign to the bank only those rights the creditor had in the contract at the time of the assignment. Question #089. A buyer entered into a written contract to purchase from a seller 1,000 sets of specially manufactured ball bearings of a nonstandard dimension for a price of $10 per set. The seller correctly calculated that it would cost $8 to manufacture each set, Delivery was scheduled for 60 days later. Fifty-five days later, after the seller had completed production of the 1,000 sets, the buyer abandoned the project that required the specially manufactured ball bearings and repudiated the contract with the seller. After notifying the buyer of his intention to resell, the seller sold the 1,000 sets of ball bearings to a salvage company for $2 per set. The seller then sued the buyer for damages. What damages should the court award to the seller? A. $2 per set, representing the difference between the cost of manufacture and the price the buyer agreed to pay. B. $6 per set, representing the difference between the cost of manufacture and the salvage price. C. $8 per set, representing the lost profits plus the unrecovered cost of manufacture. D. Nominal damages, because the seller failed to resell the goods by public auction. Question #090. OA203: Paul Manafort Convicted, Michael Cohen Pleads A construction company was digging a trench for a new sewer line in a street in a high-crime neighborhood. During the course of the construction, there had been many thefts of tools and equipment from the construction area. One night, the construction company’s employees neglected to place warning lights around the trench. A delivery truck drove into the trench and broke an axle. While the truck driver was looking for a telephone to call a tow truck, thieves broke into the truck and stole $350,000 worth of goods. The delivery company sued the construction company to recover for the $350,000 loss and for the damage to its truck. The construction company has stipulated that it was negligent in failing to place warning lights around the trench and admits liability for damage to the truck, but it denies liability for the loss of the goods. On cross-motions for summary judgment on the claim for the goods, how should the court rule? A. Deny both motions, because there is evidence to support a finding that the construction company should have realized that its negligence could create an opportunity for a third party to commit a crime. B. Grant the construction company’s motion, because no one could have foreseen that the failure to place warning lights could result in the loss of a cargo of valuable goods. C. Grant the construction company’s motion, because the criminal acts of third persons were a superseding cause of the loss. D. Grant the delivery company’s motion, because but for the construction company’s actions, the goods would not have been stolen. Question #091. OA205: More on Masterpiece, Younger & the Catholic Church (w/guest Andrew Seidel) Several public high school students asked the superintendent of their public school district whether the minister of a local church could deliver an interdenominational prayer at their graduation ceremony in the school auditorium, None of the students or their guests at graduation would be required to pray while the minister delivered the prayer. Would the minister’s delivery of such a prayer at the public high school graduation be constitutional? A. No, because it would be an unconstitutional establishment of religion. B. No, because it would deny attendees who are not members of the minister's denomination the right to freely exercise their religion. C. Yes, because none of the students or their guests would be required to pray at the graduation ceremony. D. Yes, because the idea for the prayer originated with the students and not with school officials. Question #092. OA207: Brett Kavanaugh's Confirmation Hearings At a defendant's trial for a gang-related murder, the prosecution introduced, as former testimony, a statement by a gang member who testified against the defendant at a preliminary hearing and has now invoked his privilege against self-incrimination. If the defendant now seeks to impeach the credibility of the gang member, which of the following is the court most likely to admit? A. Evidence that the gang member had three misdemeanor convictions for assault. B. Testimony by a psychologist that persons with the gang member's background have a tendency to fabricate, C. Testimony by a witness that, at the time the gang member testified, the gang member was challenging the defendant's leadership role in the gang. D. Testimony by a witness that the gang member is cocaine dealer. Question #093. OA209: Kavanaugh's Confirmation A defendant was charged with manslaughter. At the preliminary hearing, the magistrate dismissed the charge on the ground that the evidence was insufficient. The prosecutor then brought the case before a grand jury. After hearing the evidence presented by the prosecutor, the grand jury refused to return an indictment. The prosecutor waited a few months until a new grand jury had been impaneled and brought the case before that grand jury, which returned an indictment charging the defendant with manslaughter. The defendant has moved to dismiss the indictment on double jeopardy grounds. Should the motion be granted? A. No, because jeopardy had not attached before either grand jury was impaneled. B. No, because there has been no conviction or acquittal. C. Yes, because any proceeding after the preliminary hearing would violate double jeopardy. D. Yes, because bringing the case before the second grand jury violated the double jeopardy clause. Question #094. OA211: Manafort Flips (and more on Kavanaugh) Congress passed a statute directing the United States Forest Service, a federal agency, to issue regulations to control campfires on federal public lands and to establish a schedule of penalties for those who violate the new regulations. The statute provides that the Forest Service regulations should "reduce, to the maximum extent feasible, all potential hazards that arise from campfires on Forest Service lands." The Forest Service issued the regulations and the schedule of penalties directed by Congress. The regulations include a rule that provides for the doubling of the fine for any negligent or prohibited use of fire if the user is under the influence of alcohol or drugs. Which of the following arguments best supports a finding that the rule providing for the fines is constitutional? A. The executive branch of government, of which the Forest Service is part, has inherent rule-making authority over public lands. B. The rule was issued pursuant to a valid exercise of Congress's power to delegate rule-making authority to federal agencies. C. The rule is justified by a compelling government interest in safeguarding forest resources, D. The rule relates directly to law enforcement, which is an executive rather than a legislative function, and hence it does not need specific congressional authorization. Question #095. OA213: Rachel Mitchell to Cross-Examine Dr. Ford at Kavanaugh Hearings A defendant was charged with aggravated assault. At trial, the victim testified that the defendant beat her savagely, but she was not asked about anything said during the incident. The prosecutor then called a witness to testify that when the beating stopped, the victim screamed: "I'm dying-don’t let [the defendant] get away with it!" Is the testimony of the witness concerning the victim’s statement admissible? A. No, because it is hearsay not within any exception. B. No. because the victim was not asked about the statement. C. Yes, as a statement under belief of imminent death, even though the victim did not die. D. Yes, as an excited utterance. Question #096. OA215: Is Gamble v US the Real Reason Behind Kavanaugh? A bakery offered a chef a permanent full-time job as a pastry chef at a salary of $3,000 per month. The chef agreed to take the position and to begin work in two weeks. In her employment application, the chef had indicated that she was seeking Permanent job. One week after the chef was hired by the bakery, a hotel offered the chef a position as a restaurant manager at a salary of $3,500 a month. The chef accepted and promptly notified the bakery that she would not report for work at the bakery. Is the bakery likely to prevail in a lawsuit against the chef for breach of contract? A. No, because a contract for permanent employment would be interpreted to mean that the chef could leave at any time, B. No, because the position the chef took with the hotel was not substantially comparable to the one she had agreed to take with the bakery. C. Yes, because the chef's acceptance of a permanent position meant that she agreed to leave the bakery only after a reasonable time. D. Yes, because the chef's failure to give the bakery a chance to match the salary offered by the hotel breached the implied right of first refusal. Question #097. OA217: Can Ethics Complaints Take Down Kavanaugh? The personnel director of an investment company told a job applicant during his interview that the company was worth millions of dollars and that the company's portfolio would triple in the next several months. The applicant was very excited about the company's prospects and accepted an offer to work for the company. Two days later, the applicant read in the newspaper that the investment company had filed for bankruptcy reorganization. As a result of reading the news, the applicant suffered severe emotional distress but immediately found another comparable position. Is the applicant likely to prevail in an action for negligent misrepresentation? A. No, because the applicant did not suffer any physical injury or pecuniary loss. B. No, because the personnel director's statement was purely speculative. C. Yes, because the applicant relied on the personnel director's misrepresentations about the investment company. D. Yes, because the personnel director should have foreseen that his misrepresentations would cause the applicant to be distressed. Question #098. OA219: Harvard and Affirmative Action A city zoning ordinance requires that anyone who proposes to operate a group home obtain a special use permit from the city zoning board. The zoning ordinance defines a group home as a residence in which four or more unrelated adults reside. An individual applied for a special use permit to operate a group home for convicts during their transition from serving prison sentences to their release on parole. Although the proposed group home met all of the requirements for the special use permit, the zoning board denied the individual's application because of the nature of the proposed use. The individual sued the zoning board seeking declaratory and injunctive relief on constitutional grounds. Which of the following best states the appropriate burden of persuasion in this action? A. Because housing is a fundamental right, the zoning board must demonstrate that denial of the permit is necessary to serve a compelling state interest. B. Because the zoning board’s action has the effect of discriminating against a quasi-suspect class in regard to a basic subsistence right, the zoning board must demonstrate that the denial of the permit is substantially related to an important state interest. C. Because the zoning board’s action invidiously discriminates against a suspect class, the zoning board must demonstrate that denial of the permit is necessary to serve a compelling state interest D. Because the zoning board's action is in the nature of an economic or social welfare regulation, the individual seeking the permit must demonstrate that the denial of the permit is not rationally related to a legitimate state interest. Question #099. OA221: Elections Have Consequences, Florida Edition State troopers lawfully stopped a car driver on a turnpike for exceeding the speed limit by four miles per hour. One trooper approached the car to warn the driver to drive within the speed limit. The other trooper remained in the patrol car and ran a computer check of the car's license number. The computer check indicated that there was an outstanding warrant for the driver's arrest for unpaid traffic tickets. The troopers then arrested the driver based on the warrant, and they proceeded to search the driver. During the search, they discovered a package of heroin in one of the driver's pockets. Later, it was learned that the driver had paid the outstanding traffic tickets 10 days earlier and that the warrant had been quashed, but the clerk of the court had filed to update the computer, which continued to list the warrant as outstanding. The driver was charged with unlawful possession of heroin. Her attorney has filed a motion to suppress the use of the heroin as evidence Should the motion be granted? A. No, because the troopers could reasonably rely on the computer report and the search was incident to arrest. B. No, because troopers may lawfully search a driver incident to a valid traffic stop. C. Yes, because there was no arrest for the traffic violation and no lawful arrest could be made on the basis of the warrant. D. Yes, because there was no probable cause or reasonable suspicion to believe that the driver possessed drugs, Question #100. OA223: A Victory for Voting Rights in Pennsylvania! An investor purchased a tract of commercial land, financing a large part of the purchase price with a loan from a business partner that was secured by a mortgage. The investor made the installment payments on the mortgage regularly for several years. Then the investor persuaded a neighbor to buy the land, subject to the mortgage to his partner. They expressly agreed that the neighbor would not assume and agree to pay the investor's debt to the partner. The investor's mortgage to the partner contained a due-on-sale clause stating, "If Mortgagor transfers his or her interest without the written consent of Mortgagee first obtained, then at Mortgagee’s option the entire principal balance of the debt secured by this Mortgage shall become immediately due and payable." However, without seeking his partner's consent, the investor conveyed the land to the neighbor, the deed stating that it was "subject to a mortgage to [the partner]" and giving details and recording data related to the mortgage. The neighbor took possession of the land and made several mortgage payments, which the partner accepted. Now, however, neither the neighbor nor the investor has made the last three mortgage payments. The partner has sued the neighbor for the amount of the delinquent payments. In this action, for whom should the court render judgment? A. The neighbor, because she did not assume and agree to pay the investor's mortgage debt. B. The neighbor, because she is not in privity of estate with the partner. C. The partner, because the investor's deed to the neighbor violated the due-on-sale clause. D. The partner, because the neighbor is in privity of estate with the partner. Question #101. OA227: Brian Frosh Takes On Matthew Whitaker & More! A debtor owed a lender $1,500. The statute of limitations barred recovery on the claim. The debtor wrote to the lender, stating, "I promise to pay you $500 if you will extinguish the debt." The lender agreed. Is the debtor's promise to pay the lender $500 enforceable? A. No, because the debtor made no promise not to plead the statute of limitations as a defense. B. No, because there was no consideration for the debtor's promise. C. Yes, because the debtor's promise provided a benefit to the lender. D. Yes, because the debtor's promise to pay part of the barred antecedent debt needs no consideration to be enforceable. Question #102. OA229: Andrew Miller & the Appointments Clause A homeowner sued a plumber for damages resulting from the plumber’s allegedly faulty installation of water pipes in her basement, which caused flooding. At trial, the homeowner is prepared to testify that when she first detected the flooding, she turned off the water and called the plumber at his emergency number for help and that the plumber responded, "I'll come by tomorrow and redo the installation for free." Is the homeowner's testimony regarding the plumber’s response admissible? A. No, because the statement was an offer in compromise. B. No, because it is hearsay not within any exception. C. Yes, as a subsequent remedial measure. D. Yes, as evidence of the plumber's fault. Question #103. OA231: The End of the Beginning (for Trump) A purchaser bought land in the mountain foothills just outside a resort town and planned to build a housing development there. Soon thereafter, the county in which the land was located unexpectedly adopted a regulation that, for the first time, prohibited all construction in several foothill and mountain areas, including the area of the purchaser's property. The purpose of the county's regulation was "to conserve for future generations the unique natural wildlife and plant habitats" in the mountain areas. Since the adoption of the regulation, the purchaser has been unable to lease or sell the property at any price. Several Realtors have advised the purchaser that the property is now worthless. The purchaser has sued the county, claiming that the regulation has effected a taking of the purchaser’s property and that the county therefore owes the purchaser just compensation. Is the court likely to rule in favor of the purchaser? A. No, because the county did not take title to the property from the purchaser. B. No, because the regulation has not caused or authorized any uninvited physical invasion or intrusion onto the property. C. Yes, because the conservation objective of the county ordinance is not sufficiently compelling to justify the substantial diminution in the property value. D. Yes, because the effect of the county’s regulation is to deny the purchaser's investment-backed expectation and essentially all economically beneficial use of the property. Question #104. OA233: [REDACTED] & Wisconsin A woman who is a computer expert decided to dedicate herself to exposing persons who trafficked in child pornography. She posted a number of sexually oriented photographs on her website. The file for each photograph contained an embedded Trojan horse program (a program that would allow the woman to enter the computer of anyone who downloaded the photograph). A man downloaded one of those photographs onto his personal computer. Using the embedded program, the woman entered the man's computer and found a file containing a pornographic photograph of a child. She copied the file and turned it over to a federal law enforcement agency. A federal agent told her that a successful prosecution would require more than one photograph and offered her a monetary reward for additional photographs leading to the man’s conviction. The woman entered the man’s computer again, and this time she found hundreds of child pornography photographs, which she turned over to the federal agency. The man was charged with multiple counts of violating federal child pornography statutes. He has moved to suppress the photographs that the woman discovered on his computer. The motion is based on both the Fourth Amendment and a federal statute forbidding interception of electronic communication without permission. The parties have stipulated that the woman's conduct in downloading photographs from the man’s computer violated the interception statute. How should the court rule on the defendant’s motion to suppress? A. Deny the motion as to all photographs. B. Grant the motion as to all photographs, because the woman acted without probable cause. C. Grant the motion as to all photographs, because the woman violated the federal interception statute. D. Grant the motion only as to the second set of photographs. Question #105. OA235: Corporations Are People, My Friend... Criminal People A car dealer owed a bank $10,000, due on June 1. The dealer subsequently sold a car to a buyer at a price of $10,000, payable at $1,000 per month beginning on June 1. The dealer then telephoned the bank to ask whether the bank would accept payments of $1,000 per month for 10 months beginning June 1, without interest, in payment of the dealer's debt to the bank. The bank agreed to that arrangement, and the dealer then asked the buyer to make his car payments directly to the bank. When the buyer tendered the first payment to the bank, the bank refused the payment, asserting that it would accept payment only from the dealer. On June 2, the bank demanded that the dealer pay the debt in full immediately. The dealer refused to pay, and the bank sued the dealer to recover the $10,000. In this suit, which of the following arguments best supports the bank's claim for immediate payment? A. The agreement to extend the time for payment was not in writing. B. The dealer could not delegate its duty to pay to the buyer. C. The dealer gave no consideration for the agreement to extend the time of payment. D. The dealer's conduct was an attempted novation that the bank could reject. Question #106. OA237: Lowering the... Barr (Memo) A defendant was charged in federal court with selling a controlled substance (heroin) in interstate commerce. At trial, the prosecutor introduced evidence that the defendant had obtained the substance from a supplier in Kansas City and had delivered it in Chicago. The defendant denied that the substance in question was heroin, but he introduced no contrary evidence on the issue of transportation. Which of the following instructions regarding judicial notice may the judge legitimately give the jury? A. "If you find that the defendant obtained the substance in Kansas City and delivered it to Chicago, I instruct you to find that the substance was sold in an interstate transaction." B. "If you find that the defendant obtained the substance in Kansas City and delivered it to Chicago, then the burden of persuasion is on the defendant to establish that the transaction was not interstate." C. "If you find that the defendant obtained the substance in Kansas City and delivered it to Chicago, then you may, but you are not required to, find that the transaction was interstate in nature." D. "I instruct you that there is a presumption that the substance was sold in an interstate transaction, but the burden of persuasion on that issue is still on the government." Question #107. OA239: The Fourth Circuit's Puzzling Emoluments Ruling An associate professor in the pediatrics department of a local medical school was denied tenure. He asked a national education lobbying organization to represent him in his efforts to have the tenure decision reversed. In response to a letter from the organization on the professor's behalf, the dean of the medical school wrote to the organization explaining truthfully that the professor had been denied tenure because of reports that he had abused two of his former patients. Several months later, after a thorough investigation, the allegations were proven false, and the professor was granted tenure. He had remained working at the medical school at full pay during the tenure decision review process and thus suffered no pecuniary harm. In a suit for libel by the professor against the dean of the medical school, will the professor be likely to prevail? A. No, because the professor invited the libel. B. No, because the professor suffered no pecuniary loss. C. Yes, because the dean had a duty to investigate the rumor before repeating it. D. Yes, because the dean’s defamatory statement was in the form of a writing. Question #108. OA241: Is This The C-Hook That Could Send PG&E To Prison?? A man bought an antique car from a car dealer in State A. Under State A law, a person who buys from such a dealer acquires good title, even if the property was stolen from a previous owner. The man showed the car at an antique car show in State B.A woman recognized the car as having been stolen from her. Under State B law, a person whose property is stolen may reclaim it, even if the current possessor is an innocent purchaser. The woman sued the man in a State B court to reclaim the car, In his defense, the man claimed that he had good title under the law of State A. Nevertheless, the State B court applied State B law, and the woman prevailed. The man did not appeal. The sheriff had the car returned to State B and gave the woman possession of the car. Several months later, the woman moved to State A, bringing the car with her. The man brought a new suit against the woman in a State A court, claiming that the State B court in the prior suit should have applied the State A law, which protects innocent purchasers. The woman appeared and moved to dismiss the suit. What should the State A court do? A. Apply the federal law of sale of goods, because the car has moved in interstate commerce. B. Apply State A law, because the car is currently located in State A. C. Dismiss the suit, because the State A court must give full faith and credit to the State B judgment. D. Remove the case to federal court, because the car has moved in interstate commerce, and therefore the case raises a federal question. Question #109. OA243: Build That Wall! A seller and a purchaser signed a contract for the sale of a 60-year-old house. The contract required a warranty deed to be given at closing. The contract was silent regarding the condition of the house, and the purchaser did not ask. The purchaser received a warranty deed with all covenants of title at the closing and promptly recorded the deed. Approximately one month after the closing, the furnace in the house stopped working, the basement flooded, and the roof leaked so badly that the second floor could not be occupied. The seller, when told of the house's condition, was genuinely surprised. There is no applicable statute. The purchaser has sued the seller for damages. Will the purchaser likely be successful? A. Yes, because a conveyance of residential real property, a warranty of fitness is implied. B. Yes, based on the covenants of title contained in the deed the purchaser received. C. No, because the seller gave no warranty regarding the condition of the house. D. No, because of the doctrine of merger. Question #110. OA245: More on Barr and the Shutdown A dentist was anesthetizing a patient's jaw before pulling a tooth. Although the dentist used due care, the hypodermic needle broke off in the patient's gum tissue, causing injury. The needle broke because of a manufacturing defect that the dentist could not have detected, Is the patient likely to recover damages in an action against the dentist based on strict products liability and malpractice? A. No, on neither basis. B. Yes, based on malpractice, but not on strict products liability. C. Yes, based on strict products liability, but not on malpractice. D. Yes, on both bases. Question #111. OA247: Status of the Trans Ban A bottling company sent a purchase order to a wholesaler that stated, "Ship 100,000 empty plastic bottles at the posted price." Two days after receipt of this purchase order, the wholesaler shipped the bottles and the bottling company accepted delivery of them, A week after the bottles were delivered, the bottling company received the wholesaler’s acknowledgment form, which included a provision disclaiming consequential damages. After having used the wholesaler’s bottles in its bottling ‘operations for two months, the bottling company discovered a defect in the bottles that caused liquids to leak from them, The bottling company recalled 10,000 of the bottles that had been filled, incurring lost profits of $40,000, Assuming that all appropriate defenses are timely raised, will the bottling company likely succeed in recovering $40,000 in consequential damages from the wholesaler? A. No, because buyers are generally not entitled to recover consequential damages. B. No, because the bottling company’s acceptance of the goods also constituted an acceptance of the terms included in the wholesaler's acknowledgment. C. Yes, because the disclaimer of consequential damages is unconscionable. D. Yes, because the wholesaler’s acknowledgment did not alter the terms of an existing contract between the parties. Question #112. OA249: Overturning Roe v. Wade Starts Today A man, despondent and angry over losing his job, was contemplating suicide. With his revolver in his pocket, he went to a bar and drank until he was very intoxicated. He overheard a customer on the next barstool telling the bartender how it was necessary for companies to downsize in order to keep the economy strong. The man turned to the customer and said, "Why don’t you shut the hell up." The customer responded, "This is a free country, and I can say what I want," all the while shaking his finger at the man, The man became enraged, snatched his revolver from his pocket, and shot the customer, killing him. A state statute defines first-degree murder as "knowingly causing the death of another person after deliberation upon the matter." Deliberation is defined as "cool reflection for any length of time, no matter how brief." Second-degree murder is defined as "knowingly causing the death of another person." Manslaughter is defined as at common law. What crime did the man commit? A. Manslaughter, because there was a reasonable explanation for his becoming enraged. B. First-degree murder, because deliberation can take place in an instant. C. First-degree murder, because he contemplated taking a human life before becoming intoxicated. D. Second-degree murder, because he knowingly caused the customer's death without deliberation. Question #113. OA251: Gerrymandering in Maryland Heads Back to SCOTUS A state statute requires, without exception, that a woman under the age of 18 notify one of her parents at least 48 hours before having an abortion. A proper lawsuit has challenged the constitutionality of this state statute. In that suit, should the court uphold the constitutionality of the statute? A. No, because a 48-hour waiting period is excessively long, and therefore the statute imposes an undue burden on a woman’s right to procure an abortion. B. No, because the state law does not provide a bypass procedure that would allow a court to authorize a minor to obtain an abortion without prior parental notification under appropriate circumstances. C. Yes, because parents' rights to supervise their minor daughter's health care outweigh any individual right the daughter may have. D. Yes, because such parental notification and waiting-period requirements do not impose an undue burden on a minor's right to procure an abortion. Question #114. OA253: Religious Freedom and Domineque Ray A businessman owned a hotel, subject to a mortgage securing a debt he owed to a bank. The businessman later acquired a nearby parking garage, financing a part of the purchase price with a loan from a financing company, secured by a mortgage on the parking garage. Two years thereafter, the businessman defaulted on the loan owed to the bank, which caused the full amount of that loan to become immediately due and payable. The bank decided not to foreclose the mortgage on the hotel at that time, but instead properly sued for the full amount of the defaulted loan. The bank obtained and properly filed a judgment for that amount. A statute of the jurisdiction provides: "Any judgment properly filed shall, for 10 years from filing, be a lien on the real property then owned or subsequently acquired by any person against whom the judgment is rendered." There is no other applicable statute, except the statute providing for judicial foreclosure of mortgages, which places no restrictions on deficiency judgments. Shortly thereafter, the bank brought an appropriate action for judicial foreclosure of mortgage on the hotel and of its judgment lien on the parking garage. The financing company was joined as a party defendant, and appropriately sued for foreclosure of its mortgage on the parking garage, which was also in default. All procedures were properly followed, and the confirmed foreclosure sales resulted in the following: The net proceeds of the sale of the hotel to a third party were $200,000 less than the bank's mortgage balance. The net proceeds of the sale of the parking garage to a fourth party were $200,000 more than the financing company’s mortgage balance. How should the $200,000 surplus arising from the sale of the parking garage be distributed? A. It should be paid to the bank. B. It should be paid to the businessman. C. It should be paid to the financing company. D. It should be split equally between the bank and the financing company. Question #115. OA255: Wall of Emergency In a personal injury case, the plaintiff sued a retail store for injuries she sustained form a fall in the store. The plaintiff alleged that the store had negligently allowed its entryway to become slippery from snow tracked in from the sidewalk. Before the lawsuit was filed, when the plaintiff first threatened to sue, the store's manager said, "I know that there was slush on the marble entryway, but I think your four-inch-high heels were the real cause of your fall. So let's agree that we'll pay your medical bills, and you release us from any claims you might have." The plaintiff refused the offer. At trial, the plaintiff seeks to testify to the manager's statement that "there was slush on that marble entryway." Is the statement about the slush in the entryway admissible? A. No, because it is a statement made in the course of compromise negotiations. B. No, because the manager denied that the slippery condition was the cause of the plaintiff's fall. C. Yes, as a statement by an agent about a matter within the scope of his authority. D. Yes, because the rule excluding offers of compromise does not protect statements of fact made during compromise negotiations. Question #116. OA257.5 Michael Cohen Testifies, Part 2 In a civil action, a plaintiff sued a decedent's estate to recover damages for injuries she suffered in a collision between her car and one driven by the defendant. At trial, the plaintiff introduced undisputed evidence that the defendant's car had swerved across the centerline of the highway into oncoming traffic, where it had collided with the plaintiff's car. The defendant's estate introduced undisputed evidence that, before he swerved across the centerline, the defendant had suffered fatal heart attack, which he had no reason to foresee, and that, just prior to the heart attack, the defendant had been driving at a reasonable speed and in a reasonable manner. A statute makes it a traffic offense to cross the median of a highway. In this case, which party is likely to prevail? A. The defendant's estate, because its rebuttal evidence is undisputed, B. The defendant's estate, because the plaintiff has not established a prima facie case of liability. C. The plaintiff, because the accident was of a type that does not ordinarily happen in the absence of negligence on the actor's part. D. The plaintiff, because the decedent crossed the median in violation of the statute, Question #117. OA259: Your Guide to the Congressional Investigations A group of students at a state university's law school wished to debate the future of affirmative action in that state and at that law school. For this debate, they requested the use of a meeting room in the law school that is available on a first-come, first-served basis for extracurricular student use. Speakers presenting all sides of the issue were scheduled to participate. The law school administration denied the use of its meeting room for this purpose solely because it believed that "such debate, even if balanced, would have a negative effect on the morale of the law school community and might cause friction among the students that would disrupt the institution's educational mission." Is the law school’s denial of the use of its meeting room for this purpose constitutional? A. No, because the law school cannot demonstrate that is action was necessary to serve a compelling state interest. B. No, because the law school cannot demonstrate that its action was rationally related to a legitimate state interest. C. Yes, because the law school’s only concern was the adverse effect of such a discussion of affirmative action on the immediate audience and the mission of the institution. D. Yes, because the law students do not have a right to use a state-owned law school facility for a meeting that is not organized and sponsored by the law school itself. Question #118. OA261: Sentencing Paul Manafort A landowner lawfully subdivided his land into 10 large lots. The recorded subdivision plan imposed no restrictions on any of the 10 lots. Within two months after recording the plan, the landowner conveyed Lot 1 to a buyer, by a deed that contained no restrictions on the lot's use. There was then a lull in sales. Two years later, the real estate market in the state had generally improved, and during the next six months, the landowner sold and conveyed eight of the remaining nine lots. In each of the eight deeds of conveyance, the landowner included the following language: "It is a term and condition of this conveyance, which shall be covenant running with the land for the benefit of each of the 10 lots [with an appropriate reference to the recorded subdivision plan] that for 15 years from the date of recording of the plan, no use shall be made of the premises herein conveyed except for single-family residential purposes." The buyer of Lot 1 had actual knowledge of what the landowner had done. The landowner included the quoted language in part because the municipality had amended its zoning ordinance a year earlier to permit professional offices in any residential zone. Shortly after the landowner's most recent sale, when he owned only one unsold lot, the buyer of Lot 1 constructed a one-story house on Lot 1 and then conveyed Lot 1 to 8 doctor. The deed to the doctor contained no reference to any restriction on the use of Lot 1. The doctor applied for an appropriate certificate of occupancy to enable her to use a pat of the house on Lot 1 as a medical office. The landowner, on behalf of himself a the owner of the unsold lot, and on behalf of the other lot owners, sued to enjoin the doctor from carrying out her plans and to impose the quoted restriction on Lot 1. Who is likely to prevail? A. The doctor, because Lot 1 was conveyed without the restrictive covenant in the deed to the first buyer and the subsequent deed to the doctor. B. The doctor, because zoning ordinances override private restrictive covenants as a matter of public policy. C. The landowner, because the doctor, as a successor in interest to the first buyer, is estopped from denying that Lot 1 remains subject to the zoning ordinance as it existed when the landowner conveyed Lot 1 to the first buyer. D. The landowner, because with the first buyer's knowledge of the facts, Lot 1 became incorporated into a common scheme. Question #119. OA263: Nielsen v. Preap and Due Process Due Aliens A seller and a buyer have dealt with each other in hundreds of separate grain contracts over the last five years. In performing each contract, the seller delivered the grain to the buyer and, upon delivery, the buyer signed ice that showed an agreed-upon price for that delivery. Each invoice was silent in regard to any discount from the price for prompt payment. The custom of the grain trade is to allow a 2% discount from the invoice price for payment within 10 days of delivery. In all of their prior transactions and without objection from the seller the buyer took 15 days to pay and deducted 5% from the invoice price. The seller and the buyer recently entered into a contract for a single delivery of wheat at a price of $300,000. The same delivery procedure and invoice were used for this contract as had been used previously. The seller delivered the wheat and the buyer then signed the invoice. On the third day after delivery, the buyer received the following note from the seller: "Payment in full in accordance with signed invoice is due immediately. No discounts permitted. s/Seller." Which of the following statements concerning these facts is most accurate? A. The custom of the trade controls, and the buyer is entitled to take a 2% discount if he pays within 10 days. B. The parties' course of dealing controls, and the buyer is entitled to take a 5% discount if he pays within 15 days C. The seller's retraction of his prior waiver controls, and the buyer is entitled to no discount. D. The written contract controls, and the buyer is entitled to no discount because of the parole evidence rule. Question #120. OA265: The Investigation is Over, But the Investigations Continue (feat. Randall Eliason) A bus passenger was seated next to a woman whom he did not know. The woman stood to exit the bus, leaving a package on the seat. The passenger lightly tapped the woman on the back to get her attention and to inform her that she had forgotten the package. Because the woman had recently had back surgery, the tap was painful and caused her to twist and seriously injure her back. If the woman sues the passenger to recover for the back injury, will she be likely to prevail? A. No, because she is presumed to have consented to the ordinary contacts of daily life. B. No, because she was not put in apprehension by the touching. C. Yes, because the passenger intentionally touched her. D. Yes, because the passenger's intentional touching seriously injured her. Question #121. OA267: Originalism and the Eighth Amendment (Bucklew v. Precythe) The President issued an executive order in an effort to encourage U.S. citizens to use the metric (Celsius) system of temperatures. Section 1 of the executive order requires the U.S. Weather Bureau, a federal executive agency, to state temperatures only in Celsius in all weather reports. Section 2 of the executive order requires all privately owned federally licensed radio and television stations giving weather reports to report temperatures only in Celsius. No federal statute is applicable. Is the President's executive order constitutional? A. Section 1 is constitutional, but Section 2 is not. B. Section 2 is constitutional, but Section 1 is not. C. Sections 1 and 2 are constitutional. D. Sections 1 and 2 are unconstitutional. Question #122. OA269: JULIAN ASSANGE ARRESTED In a civil action for misrepresentation in the sale of real estate, the parties contested whether the defendant was licensed by the State Board of Realtors, a public agency established by statue to license real estate brokers. The defendant testified that she was licensed. On rebuttal, the plaintiff has offered a certification, bearing the seal of the secretary of the State Board of Realtors. The certification states that the secretary conducted a thorough search of the agency's records and all relevant databases, and that this search uncovered no record of license ever having been issued to the defendant. The certification is signed by the secretary. Is the certification that there is no record of a license having been issued to the defendant admissible? A. No, because it is hearsay not within any exception. B. No, because the writing was not property authenticated. C. Yes, for the limited purpose of impeaching the defendant. D. Yes, to prove the nonexistence of a public record. Question #123. OA273: Sears, Steve Mnuchin & "The Producers" A seller entered into written contract to sell a tract of land to an investor. The contract made no mention of the quality of title to be conveyed. The seller and the investor later completed the sale, and the seller delivered a warranty deed to the investor. Soon thereafter, the value of the land increased dramatically. The investor entered into a written contract to sell the land to a buyer. The contract between the investor and the buyer expressly provided that the investor would convey a marketable title. The buyer's attorney discovered that the tile of the land was not marketable and had not been marketable when the original seller had conveyed to the investor. The buyer refused to complete the sale. The investor sued the original seller for breach of contact, claiming damages from the seller's failure to convey marketable title, which resulted in the investor's loss of the sale to the subsequent buyer. Who is likely to prevail on this count? A. The investor, because the law implies in such a contract a covenant that the title will be marketable. B. The investor, because the original seller is liable for all reasonably foreseeable damages. C. The original seller, because her contract obligations as to title merged into the deed. D. The original seller, because she did not expressly agree to convey marketable title. Question #124. OA275: Yes, Bill Barr Perjured Himself A consumer became physically ill after drinking part of a bottle of soda that contained a large decomposed snail. The consumer sued the store from which she had bought the soda to recover damages for her injuries. The parties agreed that the snail had been put into the bottle during the bottling process, over which the store had no control. The parties also agreed that the snail would have been visible in the bottle before the consumer opened it. Will the consumer be likely to prevail in an action against the store? A. No, because the consumer could have seen the snail in the bottle before she drank out of it. B. No, because the store was not responsible for the bottling process. C. Yes, because the consumer was injured by a defective product sold to her by the store. D. Yes, because the store had exclusive control over the bottle before selling it to the consumer. Question #125. OA277: The Republican Civil War A defendant is on trial for attempted fraud. The state charges that the defendant switched a price tag from a cloth coat to a more expensive fur-trimmed coat and then presented the latter for purchase at the cash register. The defendant has testified in her own behalf that someone else must have switched the tag. On cross-examination, the prosecutor asks whether the defendant was convicted on two prior occasions of misdemeanor fraud of a retailer by the same means of switching the Price tag on a fur-trimmed coat. Is the question about the prior convictions proper either to impeach the defendant or to prove that the defendant committed the crime? A. It is not proper for either purpose. B. It is proper for both purposes. C. It is proper to impeach the defendant, but not to prove that the defendant committed the crime. D. It is proper to prove that the defendant committed the crime, but not to impeach the defendant. Question #126. OA279: Deutsche Wanna Loan? In a criminal trial, the evidence has shown that the defendant's neighbor entered the defendant's house with a knife and told the defendant that she was going to kill him. The defendant ran to his bedroom, picked up a gun, and told the neighbor to back off. The neighbor did not stop and stabbed the defendant in the arm. The defendant then shot the neighbor twice. The neighbor fell to the floor and lay quietly moaning. After a few seconds, the defendant fired a third shot into the neighbor. The case has gone to the jury, which has thus far found that the neighbor died instantly from the third shot and that the defendant was no longer in fear of being attacked by her when he fired the third shot. Of which of the following degrees of criminal homicide, if any, can the jury property convict the defendant? A. Attempted murder only. B. Manslaughter only. C. Murder or manslaughter. D. No degree of criminal homicide. Question #127. OA281: Follow the Money! (Analyzing Judge Mehta's Order) When a homeowner became ill, he properly executed a deed sufficient to convey his home to his nephew, who was then serving overseas in the military. Two persons signed as witnesses to qualify the deed for recording under an applicable statute. The homeowner handed the deed to his nephew's friend learned tat the homeowner's death was imminent. One day before the homeowner's death, the nephew's friend record the homeowner's death, the nephew's friend recorded the deed. The nephew returned home shortly after the homeowner's death, learned about the deed, and took possession of the home. The homeowner had died intestate, leaving a daughter as his sole heir. When she asserted ownership of the home, the nephew brought an appropriate action against her to determine title. The law of the jurisdiction requires only two witnesses for a will to be properly executed. If the court rules for the nephew and against the daughter, what will be the most likely explanation? A. The deed was delivered when the homeowner handed it to the nephew's friend. B. The delivery of the deed was accomplished by the recording of the deed. C. The homeowner's death consummated a valid gift causa mortis to the nephew. D. The homeowner's properly executed deed was effective as a testamentary document. Question #128. OA283: Mueller Speaks! (& Clarence Thomas Pens a Nonsensical Concurrence) A woman drove her car through the drive-through lane of a fast-food restaurant one afternoon. When she reached the microphone used to place orders, she said, "There's a man across the street with a rifle. He can see everything you do. If you do not do exactly what I tell you, he will shoot you. Put all the money from the register into a sack and give it to me when I drive up." The clerk did not see anyone across the street and was unsure whether anyone was there. However, unwilling to risk harm to himself, he put $500 in a paper bag and handed it to the woman when she drove up to the delivery window. The woman drove off with the money but was arrested a short time later. She had lied about there having been a man with a rifle and had acted alone. Of what crime or crimes can the woman be convicted? A. Embezzlement. B. Obtaining property by false pretenses. C. Robbery and larceny. D. Robbery or larceny. Question #129. OA285: Tulsi Gabbard & Michael Flynn A four-year-old child sustained serious injures when a playmate pushed him from between two parked cars into the street, where he was struck by a car. The child, by his representative, sued the driver of the car, the playmate's parents, and his own parents. At trial, the child's total damages were determined to be $100,000. The playmate’s parents were determined to be 20% at fault because they had failed to adequately supervise her. The driver was found to be 50% at fault. The child's own parents were determined to be 30% at fault for failure to adequately supervise him. The court has adopted the pure comparative negligence doctrine, with joint and several liability, in place of the common law rules relating to plant's fault. In addition, the common law doctrines relating to intra-family liability have been abrogated. What is the maximum amount, if anything, that the child's representative can recover from the driver? A. $30,000. B. $50,000. C. $100,000. D. Nothing. Question #130. OA287: Down the Hatch (Act)? The childhood home of a former U.S. president is part of a national park located in a city. The National Park Service entered into a contract with an independent antique collector to acquire items owned by residents of the city during the former residents' lifetime. According to the contract, the collector purchases items and then sells them to the Park Service at a price equal to the collector's cost plus a 10% commission. Purchases by antique collectors are ordinarily subject to the sales tax of the state in which the city is located. The collector has filed suit in state court to enjoin collection of the tax on these purchases for the Park Service, claiming that the sales tax is unconstitutional as applied to them. Should the state court issue the injunction? A. No, because as the purchaser of the antiques, the collector, rather than the federal government, is liable for the tax. B. No, because the suit is within the exclusive jurisdiction of the federal courts. C. Yes, because the federal government is contractually obligated to pay the amount of the sales tax when the government covers the collector's cost of the antiques. D. Yes, because under the supremacy clause, the federal program to acquire the antiques preempts the state sales tax on the purchase of these items. Question #131. OA289: #OpposeJustinWalker The defendant, a young doctor, was charged with falsely claiming deductions on her federal income tax return. At trial, a witness testified for the defendant that she has a reputation in the community for complete honesty. After a sidebar conference at which the prosecutor gave the judge a record showing that the defendant's medical school had disciplined her for altering her transcript, the prosecutor proposed to ask the witness on cross-examination: "Have you ever heard that the defendant falsified her medical school transcript?" Is the prosecutor's question proper? A. No, because it calls for hearsay not within any exception. B. No, because its minimal relevance on the issue of income tax fraud is substantially outweighed by the danger of unfair prejudice. C. Yes, because an affirmative answer will be probative of the defendant's character trait for dishonesty and, therefore, her guilt. D. Yes, because an affirmative answer will impeach the witness's credibility. Question #132. OA292: The End of Democracy A homeowner owned a large poisonous snake which had been defanged and was kept in a cage. A storm damaged the homeowner's house and the snake's cage, allowing the snake to escape. During the cleanup after the storm, a volunteer worker came across the snake. The worker tried to run away from the snake and fell, breaking his arm. In a suit by the worker against the homeowner based on strict liability in tort to recover for his injury, will the worker prevail? A. No, because the snake's escape was caused by a force of nature. B. No, because the worker should have anticipated an injury during his volunteer work. C. Yes, because the homeowner did not take adequate precautions to secure the snake. D. Yes, because the worker's injury was the result of his fear of the escaped snake. Question #133. OA294: How To Fix The Supreme Court! A buyer validly contracted in writing to buy improved land from a seller. The contract had no contingencies and was silent as to risk of loss if there was damage to, or destruction of, property improvements between contract and closing, and as to any duty to carry insurance. As soon as the parties signed the contract, the seller (who had already moved out) canceled her insurance covering the land. The buyer did not know this and did not obtain insurance. A few days later, three weeks before the agreed closing date the building on the land was struck by lightning and burned to the ground. There is no applicable statute. In an appropriate action, the buyer asserted the right to cancel the contract and to recover his earnest money. The seller said that because the risk of fire loss had passed to the buyer before the fire, the buyer must perform. If the seller prevails, what will be the most likely explanation? A. Once the parties signed the contract, only the buyer had an insurable interest and so could have protected against this loss. B. The buyer's constructive possession arising from the contract gave him the affirmative duty of protecting against loss by fire. C. The seller's cancellation of her casualty insurance caused the risk of loss to transfer to the buyer. D. Upon execution of the contract, the buyer became the equitable owner of the land under the doctrine of equitable conversion. Question #134. OA296: Understanding the Jeffrey Epstein Indictment A customer asked to see an expensive watch in a jewelry store. In conversation with the store clerk, the customer falsely claimed to be the son of the mayor. When handed the watch, the customer asked if he could put it on, walk around a bit so he could see how it felt on his wrist, and then briefly step outside to observe it in natural light. The clerk agreed, saying, "I know I can trust someone like you with the merchandise." The customer walked out of the store wearing the watch and did not return. A week later, the clerk was at a gathering when she spotted the customer wearing the watch. She told him that he must ether pay for the watch or give it back. He hissed, "You'll be sorry if you mess with me." Intimidated, the clerk backed off. The following list of crimes is in descending order of seriousness. What is the most serious crime the customer committed? A. Robbery. B. Larceny. C. False pretenses. D. Embezzlement. Question #135. OA298: Hope Hicks & Weaponized Ticks In a civil action for breach of an oral contract, the defendant denied having entered into a contract with the plaintiff, although he admitted that he had discussed doing so. Which of the following standards of admissibility should the court apply to evidence proffered as relevant to prove whether a contract was formed? A. Whether a reasonable juror would find the evidence determinative of whether the contract was or was not formed. B. Whether the evidence has any tendency to make the fact of contract formation more or less probable than without the evidence. C. Whether the evidence is sufficient to prove, absent contrary evidence, that the contract was or was not formed. D. Whether the evidence makes it more likely than not that a contract was or was not formed. Question #136. OA300: Mueller Testifies! A mother whose adult son was a law school graduate contracted with a tutor to give the son a bar exam preparation course. "If my son passes the bar exam," the mother explained to the tutor, "he has been promised a job with a law firm that will pay $55,000 a year." The tutor agreed to do the work for $5,000, although the going rate was $6,000. Before the instruction was to begin and before any payment was made, the tutor repudiated the contract. Although the mother or the son reasonably could have employed, for $6,000, an equally qualified instructor to replace the tutor, neither did so. The son failed the bar exam, and the law firm refused to employ him. It can be shown that had the son received the tutor’s instruction, he would have passed the bar exam. If the mother and the son join as plaintiffs and sue the tutor for breach of contract, how much, if anything, are they entitled to recover? A. $1,000, because all other damages could have been avoided by employing another equally qualified instructor. B. $55,000, because damages of that amount were within the contemplation of the parties at the time they contracted. C. Nominal damages only, because the mother was not injured by the breach and the tutor made no promise to the son. D. Nothing, because neither the mother nor the son took steps to avoid the consequences of the tutor's breach. Question #137. OA302: #DemocracyRIP A federal statute requires the National Bureau of Standards to establish minimum quality standards for all beer sold in the United States. The statute also provides that public hearings must precede adoption of the standards, and that once they are adopted, the standards will be subject to judicial review. While the proposed standards have not yet been announced, several Bureau officials have publicly expressed opinions indicating a belief that pasteurized beer is safer than unpasteurized beer. However, these officials have not stated whether they intend to include a pasteurization requirement in the standards. A brewery that produces unpasteurized beer is concerned that, after the appropriate proceedings, the Bureau may adopt quality standards that will prohibit the sale of unpasteurized beer. The brewery has sued in federal district court to enjoin the Bureau from adopting standards that would prohibit the sale of unpasteurized beer. How should the district court proceed with the suit? A. Determine whether the Bureau could reasonably believe that pasteurization is the safest process by which to brew beer and, if so, refuse to issue the injunction against the Bureau. B. Determine whether the process used by the brewery is as safe as pasteurization and, if so, issue the injunction against the Bureau. C. Refuse to adjudicate the merits of the suit at this time and stay the action until the Bureau has actually issued beer-quality standards. D. Refuse to adjudicate the merits of the suit and dismiss it, because it docs not involve a justiciable case or controversy. Question #138. OA304: Chelsea Manning & More A landowner entered into a single contract with a builder to have three different structures built on separate pieces of property owned by the landowner, Each structure was distinct from the other two, and the parties agreed on a specific price for each. After completing the first structure in accordance with the terms of the contract, the builder demanded payment of the specified price for that structure. At the same time, the builder told the landowner that the builder was "tired of the construction business" and would not even begin the other two structures. The landowner refused to pay anything to the builder. Is the builder likely to prevail in a suit for the agreed-upon price of the first structure? A. No, because substantial performance is a constructive condition to the landowner’s duty to pay at the contract rate. B. No, because the builder's cessation of performance without legal excuse was a willful breach of the contract. C. Yes, because the contract is divisible, and the landowner will be required to bring a separate claim for the builder's failure to complete the other two structures, D. Yes, because the contract is divisible, but the landowner will be able to deduct any recoverable damages caused by the builder's failure to complete the contract. Question #139. OA305: Live from New York! At a civil trial for slander, the plaintiff showed that defendant had called her a thief. In defense, the defendant called a witness to testify "I've been the plaintiff's neighbor for many years and people in the community generally have said she's a thief". Is the testimony concerning the plaintiff's reputation admissible? A. No, because character is an essential element of the offense and must be proven by specific instances of conduct. B. Yes, to prove that the plaintiff is a thief and to reduce or refute the damages claimed. C. Yes, to prove the plaintiff is a thief but not on damages. D. Yes, to reduce or refute the damages claimed, but not to prove that the plaintiff is a thief. Question #140. OA306: From Gavin Grimm to Jeffrey Epstein An uncle was the record title holder of a vacant tract of land. He often told friends that he would leave the land to his nephew in his will. The nephew knew of these conversations. Prior to the uncle's death, the nephew conveyed the land by warranty deed to a woman for $10,000. She did not conduct a title search of the land before she accepted the deed from the nephew. She promptly and properly recorded her deed. Last month, the uncle died, leaving the land to the nephew in his duly probated will. Both the nephew and the woman now claim ownership of the land. The nephew has offered to return the $10,000 to the woman. Who has title to the land? A. The nephew, because at the time of the deed to the woman, the uncle was the owner of record. B. The nephew, because the woman did not conduct a title search. C. The woman, because of the doctrine of estoppel by deed. D. The woman, because she recorded her deed prior to the uncle's death. Question #141. OA308: Faithless Electors A customer fell and injured himself when he slipped on a banana peel while shopping at a grocery store. The banana peel was fresh and unblemished except for a mark made by the heel of the customer's shoe. In an action brought by the customer against the store, these are the only facts in evidence. Should the trial judge permit the case to go to the jury? A. No, because the customer had an obligation to watch where he stepped. B. No, because there is not a reasonable basis for inferring that the store knew or should have known of the banana peel. C. Yes, because it is more likely than not that the peel came from a banana offered for sale by the grocer, D. Yes, because the store could foresee that a customer might slip on a banana peel. Question #142. OA310: Citizenship and the Military and... A law student rented a furnished apartment. His landlord began to solicit his advice about her legal affairs, but he refused to provide it. The landlord then demanded that he vacate the apartment immediately. The landlord also began engaging in a pattern of harassment, calling the student at home every evening and entering his apartment without his consent during times when he was at school. During these unauthorized visits, she removed the handles from the bathroom and kitchen faucets, making the faucets unusable. But she did not touch any personal property belonging to the student. The lease has a year to run, and the student is still living in the apartment. The student has sued the landlord for trespass. Is he likely to prevail? A. No, because he has no standing to sue for trespass. B. No, because the landlord caused no damage to his property. C. Yes, for compensatory damages only. D. Yes, for injunctive relief, compensatory damages, and punitive damages. Question #143. OA312: Gerrymandering in North Carolina On May 1, an uncle mailed a letter to his adult nephew that stated: "I am thinking of selling my pickup truck, which you have seen and ridden in. I would consider taking $7,000 for it." On May 3, the nephew mailed the following response: "I will buy your pickup for $7,000 cash." The uncle received this letter on May 5 and on May 6 mailed a note that stated: "It’s a deal." On May 7, before the nephew had received the letter of May 6, he phoned his uncle to report that he no longer wanted to buy the pickup truck because his driver's license had been suspended. Which of the following statements concerning this exchange is accurate? A. There was a contract as of May 3. B. There was a contract as of May 5. C. There was a contract as of May 6. D. There is no contract. Question #144. OA314: The Supreme Court and Trump's Asylum Rules A buyer and a seller entered into a written contract for the sale of an identified parcel of land. The contract expressly provided that the buyer was to pay $150,000 cash for the land at the time of the closing but did not state the closing date. The parties had not agreed on the closing date because the buyer was not sure at the time the contract was signed how she would raise the cash. Fifteen days after the contract was signed, the seller learned that he could sell the land to a third party for $200,000. The seller asked the buyer if she would agree to rescind the contract. The buyer refused. The seller then told her that he would not complete the transaction, contending that the contract was unenforceable under the statute of frauds because an essential element (time for performance) was not agreed upon by the parties and was not expressly stated in the written agreement. The seller sold the land to the third party. The buyer brought an appropriate action against the seller for breach of contract. For which party is the court likely to find? A. The buyer, because of the doctrine of unjust enrichment. B. The buyer, because the court will infer that performance within a reasonable time was intended. C. The seller, because the contract is unenforceable under the statute of frauds. D. The seller, because time of performance is presumed to be of the essence. Question #145. OA316: Unsealing Mueller's Grand Jury Testimony & Other Yodel Mountain Madness Four men are being tried for conspiracy to commit a series of bank robberies. Nine successful bank robberies took place during the period of the charged conspiracy. Because the robbers wore masks and gloves and stole the bank surveillance tapes, no witnesses have been able to directly identify the robbers. Some circumstantial evidence ties each of the men to the overall conspiracy. During cross-examination, a prosecution witness testified that one of the men was in jail on other charges during the last six robberies. That man’s lawyer has moved for a judgment of acquittal at the close of the government's case. Should the motion be granted? A. No, because a conspirator is not required to agree to all of the objectives of the conspiracy. B. No, because a conspirator need not be present at the commission of each crime conspired upon. C. Yes, provided that the man has complied with the rule requiring pretrial notice of alibi. D. Yes, regardless of compliance with the alibi rule, because the government is bound by exculpatory evidence elicited during its case-in-chief. Question #146. OA318: Quid Pro Quo Burger A seller and a buyer entered into a contract obligating the seller to convey title to a parcel of land to the buyer in exchange for $100,000. The agreement provided that the buyer's obligation to purchase the parcel was expressly conditioned upon the buyer's obtaining a loan at an interest rate no higher than 10%. The buyer was unable to do so but did obtain a loan at an interest rate of 10.5% and timely tendered the purchase price. Because the value of the land had increased since the time of contracting, the seller refused to perform. The buyer sued the seller. Is the buyer likely to prevail? A. No, because an express condition will be excused only to avoid forfeiture. B. No, because the contract called for a loan at an interest rate not to exceed 10% and it could not be modified without the consent of the seller. C. Yes, because the buyer detrimentally changed position in reliance on the seller's promise to convey. D. Yes, because the buyer's obtaining a loan at an interest rate no higher than 10% was not a condition to the seller's duty to perform. Question #147. OA320: The (Idiotic) Hearsay Defense A bright 12-year-old child attended a day-care center after school. The day-care center was located near a man-made duck pond on the property of a corporation. During the winter, the pond was used for ice-skating when conditions were suitable. At a time when the pond was obviously only partially frozen, the child sneaked away from the center's property and walked out onto the ice over the pond. The ice gave way, and the child fell into the cold water, He suffered shock and would have drowned had he not been rescued by a passerby. At the time of the incident, the pond was clearly marked with numerous signs that stated, "THIN ICE-KEEP OFF." When the child sneaked away from the day-care center, the center was staffed with a reasonable number of qualified employees, and the employees were exercising reasonable care to ensure that the children in their charge did not leave the premises. There had not been a previous instance of a child coming onto the corporation's property from the day-care center. The jurisdiction follows a rule of pure comparative negligence. In a suit brought on the child's behalf against the corporation and based only on the facts above, who is likely to prevail? A. The child, because the corporation owes a duty to keep its premises free of dangerous conditions. B. The child, because the pond was an attractive nuisance. C. The corporation, because the danger of thin ice may reasonably be expected to be understood by a 12-year-old child. D. The corporation, because the day-care center had a duty to keep the child off the ice. Question #148. OA322: Blackouts, Taxes & House Rules A bright 12-year-old child attended a day-care center after school. The center was located near a man-made duck pond on the property of a corporation. During the winter, the pond was used for ice-skating when conditions were suitable, At a time when the pond was obviously only partially frozen, the child sneaked away from the center's property and walked out onto the ice over the pond. The ice gave way, and the child fell into the cold water. He suffered shock and would have drowned had he not been rescued by a passerby. At the time of the incident, the pond was clearly marked with numerous signs that stated, "THIN ICE-KEEP OFF." When the child sneaked away from the day-care center, the center was staffed with a reasonable number of qualified employees, and the employees were exercising reasonable care to ensure that the children in their charge did not leave the premises. There had not been a previous instance of a child coming onto the corporation's property from the day-care center. The jurisdiction follows a rule of pure comparative negligence. In a suit brought on the child's behalf against the day-care center and based only on the facts above, who is likely to prevail? A. The child, because he left the center while he was under the center's care. B. The child, because the day-care center is located near a pond. C. The day-care center, because it was not negligent. D. The day-care center, because the child was a trespasser. Question #149. OA324: Trump's 9 Crimes and Misdemeanors A man intensely disliked his neighbors. One night, intending to frighten them, he spray-painted their house with racial epithets and threats to kill them. The man was arrested and prosecuted under a state law providing that "any person who threatens violence against another person with the intent to cause that person to fear for his or her life or safety may be imprisoned for up to five years." In defense, the man claimed that he did not intend to kill his neighbors, but only to scare them so that they would move away. Can the man constitutionally be convicted under this law? A. No, because he was only communicating his views and had not commenced any overt action against the neighbors. B. Yes, because he was engaged in trespass when he painted the words on his neighbors’ house. C. Yes, because his communication was a threat by which he intended to intimidate his neighbors. D. Yes, because his communication was racially motivated and thus violated the protections of the Thirteenth Amendment. Question #150. OA326: When the SCIF Hit the Fan An innkeeper, who had no previous experience in the motel or commercial laundry business and who knew nothing about the trade usages of either business, bought a motel and signed an agreement with a laundry company for the motel's laundry services. The agreement was for a term of one year and provided for "daily service at $500 a week." From their conversations during negotiation, the laundry company owner knew that the innkeeper expected laundry services seven days a week. When the laundry company refused to pick up the motel’s laundry on two successive Sundays and indicated that it would never do so, the innkeeper canceled the agreement. The laundry company sued the innkeeper for breach of contract, At trial, clear evidence was introduced to show that in the commercial laundry business "daily service" did not include service on Sundays. Is the laundry company likely to succeed in its action? A. No, because the laundry company knew the meaning the innkeeper attached to "daily service," and therefore the innkeeper’s meaning will control. B. No, because the parties attached materially different meanings to "daily service," and therefore no contract was formed. C. Yes, because the parol evidence rule will not permit the innkeeper to prove the meaning he attached to "daily service." D. Yes, because the trade usage will control the interpretation of "daily service." Question #151. OA328: The Impeachment Inquiry Resolution! (H.R. 660) A landowner orally gave his neighbor permission to share the use of a private road on the landowner's land so that the neighbor could have more convenient access to the neighbor's land. Only the landowner maintained the road. After the neighbor had used the road on a daily basis for three years, the landowner conveyed his land to a grantee, who immediately notified the neighbor that the neighbor was not to use the road. The neighbor sued the grantee, seeking a declaration that the neighbor had a right to continue to use the road. Who is likely to prevail? A. The grantee, because an oral license is invalid, B. The grantee, because the neighbor had a license that the grantee could terminate at any time. C. The neighbor, because the grantee is estopped from terminating the neighbor's use of the road. D. The neighbor. because the neighbor's use of the road was open and notorious when the grantee purchased the land. Question #152. OA330: The Impeachment Inquiry Explainer (& Pre-Embryos in Connecticut) A carpenter contracted with a homeowner to remodel the homeowner’s home for $10,000, the contract price to be paid on completion of the work. On May 29, relying on the expectation that he would finish the work and have the homeowner's payment on June 1, the carpenter contracted to buy a car under the following terms: "$10,000 in cash, if payment is made on June 1; if payment is made thereafter, the price is $12,000." The carpenter completed the work according to specifications on June 1 and demanded payment from the homeowner on that date. The homeowner, without any excuse, refused to pay. As a result, the carpenter became very excited, suffered a minor heart attack, and incurred related medical expenses of $4,000. The reasonable value of the carpenter's services in remodeling the homeowner's home was $13,000. In an action by the carpenter against the homeowner, which of the following should be the carpenter's measure of recovery? A. $10,000, the contract price. B. $14,000, the contract price plus $4,000 for the medical expenses incurred because the homeowner refused to pay. C. $12,000, the contract price plus $2,000, the bargain that was lost because the carpenter could not pay cash for the car on June 1. D. $13,000, the amount the homeowner was enriched by the carpenter's services. Question #153. OA332: Your Two New Best Friends, Bill Taylor and George Kent A plaintiff sued her employer, alleging that poor working conditions had caused her to develop a stomach ulcer. At trial, the plaintiff's medical expert testified to the cause of the plaintiff's ulcer and stated that his opinion was based in part on information in a letter the plaintiffs personal doctor had written to the plaintiffs employer, explaining why the plaintiff had missed work. When offered to prove the cause of the plaintiff's condition, is the letter from the plaintiff's doctor admissible? A. No, because it is hearsay not within any exception. B. No, because the plaintiff's doctor is not shown to be unavailable. C. Yes, because it was relied upon by the plaintiff's medical expert. D. Yes, under the business records exception to the hearsay rule. Question #154. OA335: This Week in Impeachment A homeowner was using a six-foot stepladder to clean the furnace in his home. The homeowner broke his arm when he slipped and fell from the ladder. The furnace had no warnings or instructions on how it was to be cleaned. In a suit by the homeowner against the manufacturer of the furnace to recover for his injury, is the homeowner likely to prevail? A. No, because the danger of falling from a ladder is obvious. B. No, because the homeowner should have hired a professional to clean the furnace. C. Yes, because the furnace did not have a ladder attached to it for cleaning purposes. D. Yes, because the lack of warnings or instructions for how to clean the furnace made the furnace defective. Question #155. OA337: How to Talk to Your (Republican) Family About Impeachment A landlord and a tenant orally agreed to a commercial tenancy for a term of six months beginning on July 1. Rent was to be paid by the first day of each month, and the tenant paid the first month’s rent at the time of the agreement. When the tenant arrived at the leased premises on July 1, the tenant learned that the previous tenant had not vacated the premises at the end of her lease term on May 31 and did not intend to vacate. The tenant then successfully sued the previous tenant for possession. The tenant did not inform the landlord of the eviction action until after the tenant received possession. The tenant then sued the landlord, claiming damages for that portion of the lease period during which the tenant was not in possession. If the court finds for the landlord, what will be the most likely explanation? A. By suing the previous tenant for possession, the tenant elected that remedy in lieu of a suit against the landlord. B. The landlord had delivered the legal right of possession to the tenant. C. The tenant failed to timely vacate as required to sue for constructive eviction. D. The tenant had not notified the landlord before bringing the eviction action. Question #156. OA339: Who is Jonathan Turley, Anyway? A plaintiff sued a defendant in federal court for injuries arising out of an accident involving the parties. The plaintiff alleged and presented evidence at trial demonstrating that her injuries had left her legs permanently paralyzed. The jury found in favor of the plaintiff and awarded her $5 million in damages. Two months after the court entered judgment, the defendant was given a videotape made that day showing the plaintiff jogging with her doctor. What is the best way for the defendant to seek relief from the judgment? A. Move for a new trial or in the alternative for remittitur to reduce the award in light of the shortened duration of the plaintiff's injuries B. Move for relief from the judgment on the ground that the judgment was based on the jury's mistaken belief that the plaintiff's injuries would be permanent. C. Move for relief from the judgment on the ground that the plaintiff committed a fraud in obtaining damages for permanent injuries. D. Move for relief from the judgment on the ground that there is newly discovered evidence that the plaintiff's injuries were not permanent Question #157. OA341: Articles of Impeachment (& Espionza) Before putting her home up for sale, a homeowner painted the living room ceiling to conceal major water damage caused by a leaking roof that had not yet been repaired. On the first day the home was offered for sale, the homeowner gave a buyer a personal tour. The homeowner made no statements at all regarding the water damage or the roof. Without discovering the water damage or the leaking roof and without consulting a lawyer, the buyer immediately agreed in writing to buy the home for $200,000. Before the closing date, the buyer discovered the water damage and the leaking roof. The cost of repair was estimated at $22,000. The buyer has refused to go through with the purchase. If the homeowner sues the buyer for breach of contract, is the homeowner likely to prevail? A. No, because no contract was formed since the buyer did not have a real opportunity to understand the essential terms of the contract. B. No, because the homeowner concealed evidence of the water damage and of the leaking roof. C. Yes, because the homeowner made no affirmative statements of fact about the water damage or the leaking roof. D. Yes, because the buyer acted unreasonably by failing to employ an inspector to conduct an independent inspection of the home. Question #158. OA343: The End of the ACA? (Also: Some Stuff About Impeachment) A defendant is being tried for the murder of a woman who disappeared 10 years ago and has not been heard from since. Her body has never been found. The prosecutor has presented strong circumstantial evidence that she was murdered by the defendant. To help establish the fact of her death, the prosecutor has requested that the judge give the following instruction, based on a recognized presumption in the jurisdiction: "A person missing and not heard from in the last seven years shall be presumed to be deceased." Is the instruction proper? A. No, because the fact that someone has not been heard from in seven years does not necessarily lead to a conclusion that the person is dead. B. No, because mandatory presumptions are not allowed against a criminal defendant on an element of the charged crime. C. Yes, because it expresses a rational conclusion that the jury should be required to accept. D. Yes, because the defendant has a chance to rebut the presumption by offering evidence that the woman is alive or has been heard from in the last seven years. Question #159. OA345: How John Roberts Saved Christmas (Or: Everything You Need to Know About Nixon v. US) An executive of an accounting firm was fired and told to immediately leave the building where she worked. The executive went home, but she returned that night to retrieve personal items from her office. When she discovered that her key no longer opened a door to the building, she forced the door open and went to her former office. To avoid attracting attention, she did not turn on any lights. In the dark, she knew that she was taking some items that were not hers; she planned to sort these out later and return them. Upon arriving home, she found that she had taken a record book and some financial papers that belonged to the firm. After thinking it over and becoming angrier over being fired, she burned the book and papers in her fireplace. The jurisdiction has expanded the crime of burglary to include all buildings. What crime(s) has the executive committed? A. Burglary and larceny. B. Burglary, but not larceny. C. Larceny, but not burglary. D. Neither larceny nor burglary. Question #160. OA347: Pennhurst and the Voter Purge in Georgia A wholesaler contracted in a signed writing to sell to a bakery 10,000 pounds of flour each week for 10 weeks, the flour to be delivered to the bakery on Mondays and payment to be made on Wednesday’ of each week. The bakery did all of its weekly bread baking on Tuesdays. On Monday morning of the first week, the wholesaler tendered delivery of 8,000 pounds of flour to the bakery, and the bakery accepted it on the wholesaler’s assurance that the remaining 2,000 pounds would be delivered later that evening, which it was, The bakery paid for both deliveries on Wednesday. On Monday of the second week, the wholesaler tendered delivery of 5,000 pounds of flour to the bakery and said that the remaining 5,000 pounds could not be delivered on Monday but would be delivered by Wednesday. The bakery rejected the tender. Was the bakery legally justified in rejecting the tender of the 5,000 pounds of flour? A. Yes, because the bakery was legally entitled to reject any tender that did not conform perfectly to the contract. B. Yes, because the tender was a substantial impairment of that installment and could not be cured. C. No, because the tender was not a substantial impairment of the entire contract, and the wholesaler had given assurance of a cure, D. No, because by accepting the first 8,000 pounds on Monday of the first week, the bakery had waived the condition of perfect tender and had not reinstated it. Question #161. OA349: Bolton Will Testify; Iran, Soleimani & So Much More Congress enacted a statute prohibiting discrimination in the rental of residential property anywhere in the United States on the basis of sexual orientation or preference by any person or entity, public or private. Which of the following provisions provides the strongest basis for Congress's authority to enact this statute? A. The enforcement clause of the Fourteenth Amendment B. The privileges and immunities clause of Article IV. C. The commerce clause of Article I, Section 8 D. The general welfare clause of Article I, Section 8. Question #162. OA351: Who's the Next Justin Amash? Your Guide to Impeachment, Part XVIII (Feat. Lev Parnas) A woman and her sister took a trip to the Caribbean. When they passed through U.S. Customs inspection upon their return, the customs officials found packages of cocaine stitched inside separate carry-on bags belonging to each of them. They were arrested. Upon separate questioning by customs officers, the woman broke down and cried, "I told my sister there were too many officers at this airport." The sister did not give a statement. The woman and her sister were indicted for conspiracy to import cocaine. They were tried separately. At the woman's trial, after the government introduced the above evidence and rested its case, her lawyer moved for a judgment of acquittal on grounds of insufficient evidence. Should the court grant the motion? A. No, because the evidence shows that both the woman and her sister agreed to import cocaine. B. No, because the evidence shows that both the woman and her sister possessed cocaine. C. Yes, because the evidence shows only that the woman and her sister committed separate crimes of cocaine possession. D. Yes, because the evidence shows that the woman effectively withdrew from the conspiracy when she cooperated by giving a statement. Question #163. OA353: Duplicity and Impeachment Several defendants, senior executives of a corporation, were charged with securities fraud, The government called as a witness another executive of the corporation, who had not been charged and who had been given immunity from prosecution, to authenticate handwritten notes that she had made after meetings of the corporation's management team at which the alleged fraud was discussed. The witness testified that she had prepared the notes on her own initiative to help her remember what had happened at the meetings. After this testimony, the government offered the notes into evidence to establish what had happened at the meetings. Should the witness's notes be admitted? A. No, because the notes are hearsay not within any exception. B. No, because the witness's immunity agreement with the government makes her notes untrustworthy and thus substantially more prejudicial than probative. C. Yes, because they are business records. D. Yes, because they are past recollections recorded. Question #164. OA355: Honest Answers to Impeachment Questions A college student was asleep in his bed in a college dormitory when his roommate, in a drunken fury, entered their room intending to attack the student with an ice pick while he slept. Fortunately, the phone rang and awakened the student. The roommate retreated quickly and threw the ice pick under his own bed in the same room. The next day, the student heard from friends about the roommate's murderous plans and later found the ice pick under the roommate's bed. Even though the college expelled his roommate, the student remained extremely upset and afraid to sleep. In a suit against the roommate for assault, will the student prevail? A. No, because the roommate did not touch the student. B. No, because the student was not awake when the roommate entered the room and was unaware until later that the roommate was intending to attack him. C. Yes, because it was reasonable for the student to feel afraid of sleeping in his room afterward. D. Yes, because the roommate intended to inflict serious harm. Question #165. OA358: Can Trump Block New Yorkers From Global Entry? (No.) Before the close of evidence in a federal negligence trial, the defendant submitted a proposed jury instruction on contributory negligence. Before instructing the jury, the judge informed the parties of the instructions she would give, which did not include the defendant's proposed contributory negligence instruction but did include the court's own instruction on contributory negligence. Neither party objected, ether then or after the judge had given the instructions. The jury returned a verdict for the plaintiff, and the judge entered judgment on the verdict. The defendant would like to appeal the verdict on the ground that the judge should have instructed the jury using the defendant's proposed instruction on contributory negligence. Has the defendant preserved the issue for appeal? A No, because the defendant failed to object after the judge gave the instructions to the jury. B. No, because the defendant failed to object after the judge informed the parties of the instructions she would give. C. Yes, because the defendant submitted a proposed instruction on contributory negligence. D. Yes, because the judge's failure to give the defendant's contributory negligence instruction amounted to a ruling on the instruction. Question #166. OA360: The Tuesday Afternoon Massacre A plaintiff sued a defendant, alleging that she was seriously injured when the defendant ran a red light and struck her while she was walking in a crosswalk. During the defendant's case, a witness testified that the plaintiff had told him that she was "barely touched" by the defendant's car. On cross-examination, should the court allow the plaintiff to elicit from the witness the fact that he is an adjuster for the defendant's insurance company? A. No, because testimony about liability insurance is barred by the rules of evidence. B. No, because the reference to insurance raises a collateral issue. C. Yes, for both substantive and impeachment purposes. D. Yes, for impeachment purposes only. Question #167. OA362: The Pardon Power (Or: Blagojevich, Milken, and Trump, Oh My!) An engineer entered into a written contract with an owner to serve in the essential position of on-site supervisor for construction of an office building. The day after signing the contract, the engineer was injured while bicycling and was rendered physically incapable of performing as the on-site supervisor. The engineer offered to serve as an off-site consultant for the same pay as originally agreed to by the parties. Is the owner likely to prevail in an action against the engineer for damages resulting from his failure to perform under the contract? A. No, because the engineer offered a reasonable substitute by offering to serve as an off-site consultant. B. No, because the engineer's physical ability to perform as on-site supervisor was a basic assumption of the contract. C. Yes, because the engineer breached the contract by disappointing the owner’s expectations. D. Yes, because the engineer's duty to perform was personal and absolute, Question #168. OA364: Will The Supreme Court Shield Trump's Taxes? (No.) A state law made it a criminal offense for any state employ to "knowingly provide educational services or extend welfare benefits" to a foreign national who was in the United States in violation of U.S. immigration laws. The principal of a public elementary school was prosecuted under the law for enrolling and providing education to several foreign nationals he knew to be in the country illegally. All of these actions took place before the new law was adopted, No federal statute applied to the principal's actions. What constitutional provision would be most helpful to the principal's defense? A The due process clause of the Fourteenth Amendment. B. The equal protection clause of the Fourteenth Amendment. C. The ex post facto clause of Article I, Section 10. D. The privileges or immunities clause of the Fourteenth Amendment. Question #169. OA366: Your Guide to the Coronavirus! A store owner whose jewelry store had recently been robbed was shown by a police detective a photograph of the defendant, who previously had committed other similar crimes. The store owner examined the photograph and then asked the detective whether the police believed that the man pictured was the robber. After the detective said, "We're pretty sure," the store owner stated that the man in the photograph was the one who had robbed her. The defendant was indicted for the robbery. His counsel moved to suppress any trial testimony by the store owner identifying the defendant as the robber. Should the court grant the motion and suppress the store owner's trial testimony identifying the defendant as the robber? A. No, because suppression of in-court testimony is not a proper remedy, even though the out-of-court identification was improper. B. No, because the out-of-court identification was not improper. C. Yes, because the improper out-of-court identification has necessarily tainted any in-court identification D. Yes, unless the prosecution demonstrates that the in-court identification is reliable. Question #170. OA368: Your Guide to the Coronavirus, Part 2 A man owned land. He took out a loan that was secured by a mortgage on the land. The man had personal liability on the loan. The man sold the land to a purchaser, who expressly assumed the mortgage debt. The purchaser failed to make two installment payments. To prevent default and foreclosure, the man paid the overdue installments. After making the payments, the man sued the purchaser for reimbursement. Will the man be likely to prevail? A. No, because the man did not initiate foreclosure proceedings before seeking reimbursement. B. No, because the man was personally liable on the loan. C. Yes, because the law of suretyship permits the man to seek reimbursement. D. Yes, because the man's transfer of the land discharged him from any personal liability on the loan. Question #171. OA372: The CARES Act, COVID-19, and Your $1,200 Check An experienced rancher contracted to harvest his neighbor's wheat crop for $1,000 "when the crop [was] ripe." In early September, the neighbor told the rancher that the crop was ripe. The rancher delayed because he had other customers to attend to. The neighbor was concerned that the delay might cause the crop to be lost, for hailstorms were common in that part of the country in the fall. In fact, in early October, before the crop was harvested, it was destroyed by a hailstorm. Is the rancher liable for the loss? A. No, because no time for performance was established in the contract. B. No, because the neighbor failed to tell the rancher that the crop might be destroyed by a hailstorm. C. Yes, because at the time the contract was made, the rancher had reason to foresee the loss as a probable result of is breach. D. Yes, because a party who undertakes a contractual obligation is liable for all the consequences that flow from his breach. Question #172. OA374: The Light at the End of the Tunnel A plaintiff filed a tort action in state court but then failed to prosecute the action. The defendant moved to dismiss the action, and the court granted the motion in an order that stated: "The defendant's motion to dismiss is granted, and this action is dismissed with prejudice." The court accordingly entered judgment for the defendant The plaintiff then filed the same claim against the defendant in federal court, invoking diversity jurisdiction. The defendant has asserted the defense of res judicata (claim preclusion) in its answer. Should the federal court give preclusive effect to the state-court judgment? A. No, because the judgment was entered by a state court, not a federal court. B. No, because the state court did not rule on the merits in its dismissal. C. Yes, because a dismissal with prejudice operates as a judgment on the merits. D. Yes, because a judgment for failure to prosecute operates as a judgment on the merits under the Federal Rules of Civil Procedure. Question #173. OA376: Texas, Wisconsin & Washington (feat. Andrew Seidel) A pedestrian domiciled in State A was crossing a street in State B when he was hit by a car driven by a citizen of a foreign country. Both the pedestrian and the driver suffered injuries. The pedestrian filed a negligence action against the driver in a federal district court in State B, seeking $100,000 in damages. The "river believes that the pedestrian was crossing the street illegally and is therefore responsible for the accident. The driver seeks an attorney's advice on how best to respond to the action. Assume that State B is a contributory negligence state. How should the attorney advise the driver to respond? A. File an answer raising the affirmative defense of contributory negligence and asserting a counterclaim for negligence, seeking damages for the driver's injuries. B. File an answer raising the affirmative defense of contributory negligence and move for judgment on the pleadings. C. Move to dismiss for lack of personal jurisdiction, because the driver is not a citizen of State B. D. Move to dismiss for lack of subject-matter jurisdiction, because the driver is not a US. citizen. Question #174. OA378: Trump's Stupid Threat to "Adjourn" the Senate (A Primer on Appointments) A niece had worked in her aunt's bookstore for many years. The bookstore business, which was housed in a building that the aunt leased, was independently appraised at $200,000. The aunt decided to retire. She wrote to the niece, expressing her affection for the niece and offering to sell her the bookstore business for $125,000 if the landlord would agree to a transfer of the lease. The letter also specified when the aunt would transfer the business. The niece wrote back accepting her aunt's offer. In a phone call to the niece, the aunt stated that the landlord had approved the transfer of the lease and that she would now ask her attorney to draft a written contract so that there would be a record of the terms. Before the attorney had finished drafting the document, the aunt changed her mind about selling the business and informed the niece of her decision. In an action for breach of contract brought by the niece against her aunt, is the niece likely to prevail? A. No, because the motivation for the transfer of the business was the aunt's affection for her niece, not the price. B. No, because the promised consideration was inadequate in light of the market value of the business. C. Yes, because the condition concerning the landlord's assent to the transfer of the lease was beyond the control of either party. D. Yes, because the document being drafted by the attorney was merely a record of an agreement already made, not a condition to it. Question #175. OA380: This Week at the Supreme Court A man and his friend were watching a televised football game at the man’s home. Upset by a penalty called by the referee, the friend threw a bottle of beer at the man’s television, breaking the screen. Enraged, the man picked up a nearby hammer and hit the friend on the head with it. The friend died from the blow. The crimes below are listed in descending order of seriousness. Ina jurisdiction that follows common law principles, what is the most serious crime of which the man could properly be convicted? A. Murder. B. Voluntary manslaughter. C. Involuntary manslaughter. D. Assault. Question #176. OA382: Bernie Sanders Was Right! (The DNC & Payday Lenders) A protester entered an IRS office during business hours. He denounced the income tax and set fire to pages from his copy of the Internal Revenue Code. The fire was extinguished before it caused any other damage. The protester was arrested and charged with, violating a state law that prohibited igniting a fire in a public building. He claimed that his prosecution was unconstitutional under the First Amendment. May the protester constitutionally be convicted? A. because he was exercising his right to freedom of speech by burning a copy of the code. B. No, because the copy of the code belonged to him, and thus burning it did not infringe upon a legitimate government interest. C. Yes, because the burning of the code was conduct rather than speech. D. Yes, because the state law is narrowly drawn to further a substantial government interest in prohibiting the noncommunicative aspects of the act in question. Question #177. OA384: Bridgegate Apparently Was Fine A gas company built a large refining facility that conformed to zoning requirements on land near a landowner’s property. The landowner had his own home and a mini-golf business on his property. In a nuisance action against the gas company, the landowner established that the refinery emitted fumes that made many people feel quite sick when they were outside on his property for longer than a few minutes. The landowner’s mini-golf business had greatly declined as a consequence, and the value of his property had gone down markedly. Is the landowner likely to prevail? A. No, because the landowner has offered no evidence demonstrating that the gas company was negligent. B. No, because the refinery conforms to the zoning requirements. C. Yes, because the refinery has substantially and unreasonably interfered with the landowner’s use and enjoyment of his property. D. Yes, because the value of the landowner’s property has declined. Question #178. OA386: The Opening Arguments Amicus Brief! A defendant was charged with attempted murder. At the preliminary hearing, the presiding judge heard the testimony of four prosecution witnesses and found that the prosecution had failed to establish probable cause that the defendant had committed any offense. Accordingly, he dismissed the charge. The prosecutor then called the same four witnesses before a grand jury. The grand jury indicted the same defendant for attempted murder. The defendant has moved to quash the indictment on the ground of double jeopardy. How should the court proceed? A. Grant the motion, because the dismissal of the first charge on the merits, whether corrector incorrect, bars any further prosecution. B. Grant the motion, unless the prosecution has evidence that was not presented in the first case. C. Deny the motion, because the defendant has not yet been in jeopardy of conviction on the attempted murder charge. D. Deny the motion, because the protection of the double jeopardy clause does not come into play until there has been a conviction or an acquittal further prosecution. Question #179. OA388: Reinstating the Rule of Law A plaintiff sued his insurance company for the proceeds of a casualty insurance policy covering his 60-foot yacht, claiming that the yacht was destroyed by an accidental fire. The company has denied liability, claiming that the plaintiff hired his friend to set the fire. In the hospital the day after the fire, the friend, who had been badly burned in the fire, said to his wife, in the presence of an attending nurse, "I was paid to set the fire." Two weeks later, the friend died of an infection resulting from the burns, At trial, the insurance company calls the wife to testify to the friend’s statement. Is the wife’s testimony admissible over the plaintiff's objection? A. No, because the marital privilege survives the communicating spouse's death. B. No, because the statement was made after the conspiracy ended. C. Yes, because it is a statement against interest, D. Yes, because it is a statement by a coconspirator. Question #180. OA390: Trump's War on Twitter (A Deep Dive on Section 230) A fire that started in the defendant's warehouse spread to the plaintiff's adjacent warehouse. The defendant did not intentionally start the fire, and the plaintiff can produce no evidence as to how the fire started. However, the defendant had failed to install a sprinkler system, which was required by a criminal statute. The plaintiff can produce evidence that had the sprinkler system been installed, it could have extinguished the fire before it spread. In an action by the plaintiff against the defendant to recover for the fire damage, is it possible for the plaintiff to prevail? A. No, because the statute provides only for criminal penalties. B. No, because there is no evidence that the defendant negligently caused the fire to start. C. Yes, because a landowner is strictly liable for harm to others caused by the spread of fire from his premises under the doctrine of Rylands v. Fletcher. D. Yes, because the plaintiff was harmed as a result of the defendant's violation of a statute that was meant to protect against this type of occurrence. Question #181. OA392: In the Aftermath of George Floyd A defendant was charged with the crime of defrauding the federal agency where he worked as an accountant. At trial, the court allowed the defendant to call his supervisor at the large corporation where he had previously worked, who testified about the defendant's good reputation in the community for honesty. Over objection, the defendant then sought to elicit testimony from his former supervisor that on several occasions the corporation had, without incident, entrusted him with large sums of money. Should the testimony be admitted? A. No, because the testimony is extrinsic evidence on a collateral matter. B. No, because good character cannot be proved by specific instances of conduct unless character is an essential element of the charge or defense. C. Yes, because it is evidence of a pertinent character trait offered by an accused. D. Yes, because it is relevant to whether the defendant was likely to have taken money as charged in this case. Question #182. OA394: The Amicus Show! A landowner borrowed $100,000 from a lender and executed a valid mortgage on a commercial tract of land to secure the debt. The lender promptly recorded the mortgage. A year later, the landowner conveyed the same tract to a developer by a deed that expressly stated that the conveyance was subject to the mortgage to the lender and that the grantee expressly assumed and agreed to pay the mortgage obligation as part of the consideration for the purchase. The mortgage was properly described in the deed, and the deed was properly executed by the landowner; however, because there was no provision or place in the deed for the developer to sign, he did not do so. The developer promptly recorded the deed. The developer made the monthly mortgage payments of principal and interest for six payments but then stopped payments and defaulted on the mortgage obligation. The lender properly instituted foreclosure procedures in accordance with the governing law: After the foreclosure sale, there was a $10,000 deficiency due to the lender. Both the landowner and the developer had sufficient assets to pay the deficiency. There is no applicable statute in the jurisdiction other than the statute relating to foreclosure proceedings. At the appropriate stage of the foreclosure action, which party will the court decide is responsible for payment of the deficiency? A. The developer, because he accepted delivery of the deed from the landowner and in so doing accepted the terms and conditions of the deed. B. The developer, because he is estopped by his having made six monthly payments to the lender. C. The landowner, because the developer was not a signatory to the deed. D. The landowner, because he was the maker of the note and the mortgage, and at most the developer is liable only as a guarantor of the landowner’s obligation. Question #183. OA396: Happy Juneteenth from the Supreme Court! A private university is owned and operated by a religious organization. The university is accredited by the department of education of the state in which it is located. This accreditation certifies that the university meets prescribed educational standards. Because it is accredited, the university qualifies for state funding for certain of its operating expenses. Under this funding program, 25 percent of the university's total operating budget comes from state funds. A professor at the university was a part-time columnist for the local newspaper. In one of her published columns, the professor argued that "religion has become a negative force in society." The university subsequently discharged the professor, giving as its sole reason for the dismissal her authorship and publication of this column, The professor sued the university, claiming only that her discharge violated her constitutional right to freedom of speech. The university moved to dismiss the professor's lawsuit on the ground that the U.S. Constitution does not provide the professor with a cause of action in this case. Should the court grant the university's motion to dismiss? A. Yes, because the First and Fourteenth Amendments protect the right of the university to employ only individuals who share and communicate its views. B. Yes, because the action of the university in discharging the professor is not attributable to the state for purposes of the Fourteenth Amendment. C. No, because the accreditation and partial funding of the university by the state are sufficient to justify the conclusion that the state was an active participant in the discharge of the professor. D. No, because the U.S. Constitution provides a cause of action against any state-accredited institution that restricts freedom of speech as a condition of employment. Question #184. OA398: The SCOTUS Asylum Ruling, Explained Seven years ago, a man, his sister, and his cousin became equal owners, as tenants in common, of a house. Until a year ago, the man lived in the house alone. The sister and the cousin are longtime residents of another state. One year ago, the man moved to an apartment and rented the house to a tenant for three years under a lease that the man and the tenant both signed. The tenant has since paid the rent each month to the man. Recently, the sister and the cousin learned about the rental. They brought an appropriate action against the tenant to have the lease declared void and to have the tenant evicted, The tenant raised all available defenses. What will the court likely decide? A. The lease is void, and the tenant is evicted. B. The lease is valid, and the tenant retains exclusive occupancy rights for the balance of the term. C. The lease is valid, but the tenant is evicted because one-third of the lease term has expired and the man had only a one-third interest to transfer. D. The lease is valid, and the tenant is not evicted but must share possession with the sister and the cousin. Question #185. OA400: No, John Roberts Is Not Your Friend A schizophrenic patient who was institutionalized in a psychiatric facility pushed a nurse down a stairwell at the facility. The nurse, a paid employee of the facility who was trained to care for schizophrenic patients, was injured. The patient is an indigent whose care is paid for by the government, The jurisdiction generally follows the rule that a person with a mental deficiency is held to the standard of a reasonable person. In a negligence action brought by the nurse against the patient, the patient's lawyer will argue that the patient should not be held responsible for the nurse’s injury. Which of the following facts will be LEAST helpful to the patient's lawyer's argument? A. The nurse was a professional caregiver. B. The nurse was trained to care for patients with schizophrenia. C. At the time she pushed the nurse, the patient thought she was being attacked by an elephant. D. The patient is an indigent whose care is paid for by the government. Question #186. OA402: Good News from SCOTUS! A shop owner domiciled in State A sued a distributor in a federal district court in State A for breach of contract. The shop owner sought $100,000 in damages for allegedly defective goods that the distributor had provided under the contract. The distributor is incorporated in State B, with its principal place of business in State C. The distributor brought in as a third-party defendant the wholesaler that had provided the goods to the distributor, alleging that the wholesaler had a duty to indemnify the distributor for any damages recovered by the shop owner. The wholesaler is incorporated in State B, with its principal place of business in State A. The wholesaler has asserted a $60,000 counterclaim against the distributor for payment for the goods at issue, and the distributor has moved to dismiss the counterclaim for lack of subject-matter jurisdiction. Should the motion to dismiss be granted? A No, because the wholesaler’s and the distributor's principal places of business are diverse. B. No, because there is supplemental jurisdiction over the wholesaler’s counterclaim. C. Yes, because there is no diversity of citizenship between the distributor and the wholesaler. D. Yes, because there is no diversity of citizenship between the shop owner and the wholesaler. Question #187. OA404: Rule of Law Not Found A man decided to give his farm to his nephew. The man took a deed to his attorney and told the attorney to deliver the deed to the nephew upon the man’s death, The man also told the attorney to return the deed to him if he asked. None of these instructions to the attorney were in writing, and the deed was not recorded. The man then e-mailed the nephew informing him of the arrangement. Shortly thereafter, the nephew died testate. In his will, he devised the farm to his daughter. Several years later, the man died intestate, survived by two sons. The nephew's daughter immediately claimed ownership of the farm and demanded that the attorney deliver the deed to her. Must the attorney deliver the deed to the daughter? A. No, because a gratuitous death escrow is void unless supported by a written contract. B. No, because the man never placed the deed beyond his control. C. Yes, because the death of the nephew rendered the gratuitous death escrow irrevocable by the man. D. Yes, because the deed to the nephew was legally delivered when the man took it to his attorney. Question #188. OA408: What Happens When The CARES Act Expires Tomorrow? A plaintiff sued for injuries arising from a car accident, claiming a back injury. At trial, she wishes to testify that before the accident she had never had any problems with her back. Is the plaintiff's proposed testimony admissible? A. No, because the plaintiff has not been qualified as an expert. B. No, because the plaintiff's pain could have been caused by factors arising after the accident, such as an injury at work. C. Yes, because it is probative evidence of the plaintiff's injury. D. Yes, because the testimony of parties is not subject to the lay opinion rule. Question #189. OA410: NY AG Files to Dissolve the NRA! A farmer kept antiques in an uninhabited farmhouse on his property. The farmhouse had been broken into several times in the past, and some of the farmer’s goods had been stolen. Instead of posting "No Trespassing" signs, the farmer decided to install an alarm system to deter intruders. While the farmer was in the farmhouse installing the alarm system, he heard a window open in the adjoining room. The farmer crept very quietly to the door of the room, threw the door open, and found an intruder, a young child. The farmer immediately struck the child, a 10-year-old girl, very hard in the face, breaking her nose. In an action on behalf of the child against the farmer to recover for the injury to her nose, is the child likely to prevail? A. No, because the farmer did not use deadly force. B. No, because the farmer had probable cause to believe that the child was a thief. C. Yes, because the farmer should have posted a "No Trespassing" sign. D. Yes, because the farmer used excessive force. Question #190. OA414: Steve Bannon Arrested! A state law that restricted abortion was challenged in state court as a violation of the due process clause of the Fourteenth Amendment to the U.S. Constitution and as a violation of a similar due process provision of the state constitution. The ease made its way to the state's highest court, which riled that the law violated the due process provisions of both the U.S. and the state constitutions. If petitioned to do so, may the U.S. Supreme Court exercise jurisdiction to review the state court decision? A. No, because the state court's decision in this case rests on adequate and independent state law grounds. B. No, because the U.S. Supreme Court has appellate jurisdiction only over state court decisions that determine the constitutionality of federal laws. C. Yes, because the U.S. Supreme Court has appellate jurisdiction over any ruling of a state's highest court based on an interpretation of federal law. D. Yes, because the U.S, Supreme Court has appellate jurisdiction over decisions that find state laws in violation of the federal Constitution. Question #191. OA416: The Kenosha Terrorist The attorney for a plaintiff in an action filed in federal district court served the defendant with the summons, the complaint, and 25 interrogatories asking questions about the defendant's contentions in the case. The interrogatories stated that they were to be answered within 30 days after service. The defendant is likely to succeed in obtaining a protective order on which of the following grounds? A Interrogatories are proper only to discover facts, not contentions. B. Interrogatories may not be served until an answer to the complaint is filed. C. Interrogatories may not be served until the parties have conferred to arrange for initial disclosures and prepare a discovery plan. D. The interrogatories exceed the number permitted without permission from the court or an agreement between the parties. Question #192. OA418: D.C. Circuit Panel's Multiple Bad Court Thingies A businesswoman owned two adjoining tracts of land, one that was improved with a commercial rental building and another that was vacant and abutted a river. Twenty years ago, the businesswoman conveyed the vacant tract to a grantee by a warranty deed that the businesswoman signed but the grantee did not. The deed contained a covenant by the grantee as owner of the vacant tract that neither he nor his heirs or assigns would "erect any building" on the vacant tract, in order to preserve the view of the river from the commercial building on the improved tract. The grantee intended to use the vacant tract as a nature preserve. The grantee promptly and property recorded the deed. Last year, the businesswoman conveyed the improved tract to a businessman. A month later, the grantee died, devising all of his property including the vacant land, to his cousin. Six weeks ago, the cousin began construction of a building on the vacant tract. The businessman objected and sued to enjoin construction of the building. Who is likely to prevail? A. The businessman, because the commercial building was constructed before the cousin began his construction project. B. The businessman, because the cousin is bound by the covenant made by the grantee. C. The cousin, because an equitable servitude does not survive the death of the promisor. D. The cousin, because the grantee did not sign the deed. Question #193. OA420: DoJ Defending Trump in Carroll Case? An actor straight out of drama school and an agent entered into a one-year written contract that described the services the agent would provide. Because he was eager for work, the actor agreed, in the contract, to pay the agent 15% of his yearly earnings. At the end of the year, the actor was so pleased with his many roles that he gave the agent 20% of his earnings. After the first contract had expired, the actor and the agent decided to continue working together. They photocopied their old contract, changed the date, and signed it. At the end of the year, a dispute arose as to what percentage of earnings the actor owed. It is a trade practice in the acting profession for actors to pay their agents 10% of their yearly earnings, payable at the end of the year. What percentage of the actor's earnings is a court most likely to award the agent? A. 20%, because course of dealing is given greater weight than trade usage. B. 15%, because it was an express term of the contract. C. 10%, because trade usage is the applicable default rule. D. Nothing, because the contract is too indefinite. Question #194. OA422: Road to Sedition aka Bill Barr, Evil POS A man and a woman, both purse snatchers, independently entered a grocery store, each with the intent to steal purses from unsuspecting shoppers. The man approached a victim, grabbed the strap of the purse that was hanging from her shoulder, and pulled. The victim screamed and tried to hold on to the purse, but the man knocked her down, causing her to lose her grip on the purse. The man then escaped with the purse. During the commotion that arose when the man's victim screamed, the woman approached the shopping cart of a shopper who had turned toward the noise and wandered away from her cart. The woman reached into the now-unattended shopping cart, grabbed the shopper's purse, and quickly walked away, also escaping with a purse. Both purse snatchers were apprehended a short while later. Which of the two purse snatchers could properly be convicted of robbery? A. Both the man and the woman. B. Only the man. C. Only the woman. D. Neither the man nor the woman. Question #195. OA424: No Charges for Breonna's Killer A mining company that operated a copper mine in a remote location kept dynamite in a storage facility at the mine. The storage facility was designed and operated in conformity with state-of the-art safety standards. In the jurisdiction, the storage of dynamite is deemed an abnormally dangerous activity. Dynamite that was stored in the mining company’s storage facility and that had been manufactured by an explosives manufacturer exploded due to an unknown cause. The explosion injured a state employee who was at the mine performing a safety audit. The employee brought an action in strict liability against the mining company. What would be the mining company’s best defense? A. The mine was in a remote location. B. The mining company did not manufacture the dynamite. C. The state employee assumed the risk of injury inherent in the job. D. The storage facility conformed to state-of-the-art safety standards. Question #196. OA426: Breonna Updates; Trump's Outrageous Tax Returns A recently enacted federal statute requires the President to make each appointment of a United States ambassador to a foreign country from a list of three individuals. The list is to be compiled by the Senate Foreign Relations Committee and approved by the full Senate in advance of the appointment. The statute also provides that Senate confirmation of the appointment is deemed to occur automatically 30 days after the time the President names an appointee from the list, unless the full Senate determines otherwise within the 30-day period. Is this statute constitutional? A. No, because the statute violates the constitutional requirements for appointment of principal officers of the United States. B. No, because the statute impermissibly restricts the plenary foreign affairs powers of the President. C. Yes, because the statute is consistent with the constitutional requirement that the presidential appointment of ambassadors be with the advice and consent of the Senate. D. Yes, because the statute is a necessary and proper measure in furtherance of Congress's power to regulate commerce with foreign states. Question #197. OA428: Govt. Caught Lying for Flynn A plaintiff, who had been injured in an automobile collision, sued for damages. The defendant denied negligence and denied that the plaintiff's injuries were severe, At trial, the plaintiff has offered in evidence a color photograph of himself made from a videotape taken by a television news crew at the scene of the collision. The plaintiff has demonstrated that the videotape has since been routinely reused by the television station and that the footage of the plaintiff was erased. The photograph shows the plaintiff moments after the collision, with his bloodied head protruding at a grotesque angle through the broken windshield of his car. Should the photograph be admitted over the defendant's objection? A. No, because the plaintiff has failed to establish that a duplicate could not be found. B. No, because the plaintiff has failed to produce the original videotape or a duplicate. C. Yes, because it tends to prove a controverted fact. D. Yes, because a photograph that establishes a disputed fact cannot be excluded as prejudicial. Question #198. OA430: Amy Coney Barrett Is Terrible Under a state law, a drug company that makes a false factual claim about a prescription drug is strictly liable in tort to any user of the rug. In an advertisement promoting sales of a particular drug, a drug company claimed that the drug was safe for children. Suit was filed against the company on behalf of a child who allegedly was harmed as a result of taking the drug. At the time the child took the drug, the available medical studies supported the company’s claim that the drug was safe for children, but later research proved that the drug actually was harmful to children. The company has moved to dismiss the suit on First Amendment grounds. Should the court grant the motion? A. No, because false or misleading commercial speech is not constitutionally protected. B. No, because the drug business is subject to extensive health and safety regulation. C. Yes, because liability cannot be imposed for false statements without a showing of actual malice. D. Yes, because the company's claims about the drug were a matter of public concern. Question #199. OA432: Trump Accidentally Declassifies Russia Docs?! A retailer brought a federal diversity action against a wholesaler, alleging breach of contract and fraudulent misrepresentation. After the parties presented their evidence at trial, the court instructed the jury on the law. Neither party filed a motion for judgment as a matter of law before the case went to the jury. The jury found for the retailer on both claims. After the court entered judgment on the verdict, the wholesaler moved for a new trial and for judgment as a matter of law, arguing that the evidence was insufficient to support the jury verdict on either claim. The court acknowledged that there had been problems with some of the evidence, but it denied the motions. The wholesaler appealed, challenging the sufficiency of the evidence. Should the appellate court consider the wholesalers challenge? A. No, because a determination of the sufficiency of the evidence is solely within the jury's province. B. No, because the wholesaler did not raise the sufficiency-of-the-evidence issue in a motion for judgment as a matter of law before the case went to the jury. C. Yes, because the challenge was raised and ruled on by the trial court before the wholesaler filed the appeal. D. Yes, because, as the trial court acknowledged, the wholesaler has strong arguments on the challenge. Question #200. OA434: Kavanaugh's Crooked Concurrence A bank agreed to lend a merchant $10,000 for one year at 8% interest. The loan proceeds were to be disbursed within two weeks. The merchant intended to use the loan proceeds to purchase a specific shipment of carpets for resale at an expected profit of $5,000 but said nothing about these plans to the bank. The bank failed to disburse the proceeds and refused to assure the merchant that it would do so. The merchant was able to secure a loan from another lender at 10% interest for one year. However, by the time the merchant started the application process for a substitute loan, it was too late to pursue the opportunity to buy the shipment of carpets. In an action against the bank for breach of contract, which of the following amounts is the merchant likely to recover? A. Nothing, because damages for lost opportunities are not recoverable. B. Nothing, because the parties failed to tacitly agree that the merchant would be entitled to damages in the event of a breach by the bank. C. The difference in cost over time between a loan at 10% and a loan at 8%. D. $5,000, the merchant's foreseeable loss. Question #201. OA436: It's A Runoff! A rancher and his neighbor were involved in a boundary dispute. In order to resolve their differences, each drove his truck to an open pasture area on his land where the two properties were separated by a fence. The rancher was accompanied by four friends, and the neighbor was alone. The neighbor got out of his truck and walked toward the fence. The rancher got out but simply stood by his truck. When the neighbor came over the fence, the rancher shot him, inflicting serious injury. In a battery action brought by the neighbor against the rancher, the rancher testified that he actually thought his neighbor was armed, although he could point to nothing that would have reasonably justified this belief. Is the neighbor likely to prevail? A. No, because the rancher was standing on his own property and had no obligation to retreat. B. No, because the rancher suspected that the neighbor was armed. C. Yes, because deadly force is never appropriate in a property dispute. D. Yes, because it was unreasonable for the rancher to consider the use of a gun necessary for self-defense, Question #202. OA438: This Coup Shall Pass A cyclist sued a defendant corporation for injuries sustained when she was hit by a truck owned by the defendant and driven by its employee, who was making deliveries for the defendant. The day after the accident, the employee visited the cyclist in the hospital and said, "I'm sorry for what I did." At trial, the employee has testified that he exercised due care. Why is the cyclist’s testimony relating what the defendant's employee said a the hospital admissible to prove negligence? A. It is a prior inconsistent statement. B. It is a statement against interest. C. It is a statement by a party-opponent’s agent. D. It is a statement of then-existing state of mind. Question #203. OA440: Rudy's Literal Meltdown Police, who had probable cause to arrest a man for a series of armed robberies, obtained a warrant to arrest him, At 6 am, they surreptitiously entered the man’s house and, with guns drawn, went to the man’s bedroom, where they awakened him. Started, the man asked, "What's going on?" and an officer replied, "We've got you now." Another officer immediately asked the man if he had committed a particular robbery, and the man said that he had. The police then informed him that he was under arrest and ordered him to get dressed. Charged with robbery, the man has moved to suppress the use of his statement as evidence What is the man’s best argument for granting his motion? A. The police did not give him the required Miranda warnings. B. The statement was not voluntary. C. He was not informed that he was under arrest until after he made the statement. D. The police did not have a search warrant authorizing entry into the house. Question #204. OA444: Going Out of Business Pardon Sale A passenger in a car sustained a serious back injury after the vehicle she was riding in was rear-ended by a negligent driver. The passenger sued both the man who was driving the vehicle that she was in as well as the driver of the vehicle that rear-ended their car. The jury found that the passenger's damages totaled $100,000. The man was found to be 20% at fault. The driver of the other vehicle was found to be 70% at fault. The passenger was found to be 10% at fault for not wearing a seatbelt. The jurisdiction has adopted the pure comparative negligence doctrine with joint-and-several liability. What is the maximum amount, if anything, that the passenger can recover from the man? A. $100,000. B. $90,000. C. $20,000. D. Nothing. Question #205. OA446: Republicans Obliterate the Constitution in Texas v. Pennsylvania A man was flipping through a manufacturer's catalogue one day and noticed that the manufacturer produces deck chairs. Interested in a set of four for his house, the man called the manufacturer and spoke with the salesperson. The parties orally agreed that the man would purchase four standard chairs for $600. The man said he might be able to come pick them up, and the salesperson replied that they would "make the necessary arrangements with a shipping company." Nothing else regarding delivery was mentioned. The manufacturer immediately began work on the chairs. The next day, the man found different chairs that he liked better from another source. He sent an email to the manufacturer, stating the following: "I am no longer interested in purchasing the four chairs from you that we agreed upon yesterday. I am sorry for the inconvenience." He signed his name at the bottom of the email. While the manufacturer saw that he had an email from the man, the e-mail was not opened before the chairs were completed. The manufacturer arranged with a local shipping company to have the chairs shipped to the man’s billing address, delivered the chairs to the shipping company, and sent the man a new email stating that the chairs were on the way. The email from the man was then read, but it was too late as the chairs had already been shipped. Unfortunately the chairs were damaged while in the shipping company’s possession. When they were delivered to the man, he refused to pay for them. The manufacturer brought a claim for breach of contract. Who is likely to prevail? A. The manufacturer, because the manufacturer relied on the man’s promise to buy the chairs. B. The manufacturer, because the risk of loss had shifted to the man. C. The man, because there is no enforceable contract. D. The man, because the chairs were damaged in transit. Question #206. OA448: Secret Expert Witness "Spyder" is Dude Named Josh A restaurant owner sued a city official who condemned her building, arguing that the official wanted the land for his own personal profit. The restaurant owner filed a request for a preliminary injunction to prevent the destruction of her building until the case could be resolved at trial. The court granted this request. The city official asserted a claim of immunity, arguing that he did not have to stand trial. He also filed a motion to dismiss for failure to state a claim upon which relief could be granted. The court denied both of the city official's requests. The official wanted to immediately appeal all three decisions before trial and sought the advice of his attorney. Which of the following is the best and most accurate piece of advice the attorney could give? A. None of these decisions are appealable until the case is completed to the end. B. All three of these decisions are immediately appealable as they are all either of grave importance or could resolve the case then and there. C. Only the granting of the preliminary injunction is immediately appealable as an interlocutory order. D. Both the granting of the preliminary injunction and the denial of immunity are immediately appealable. Question #207. OA450: A Christmas Deep-Dive into the COVID Bill A man was arrested as the main suspect in a murder investigation. He was brought into an interrogation room, but was not read his Miranda rights. A police officer asked the man, "Where were you on the night of May 2?" The man proceeded to confess to the murder. He also told the officer where to find the murder weapon. The officer immediately left the room, and he then realized that he accidentally forgot to read the man his Miranda rights. He reported his mistake to the chief, who decided to place the man in a holding cell while they searched for the weapon. The weapon was recovered. Two days later, the man was brought back into the interrogation room where he was questioned by a different officer. Prior to questioning, this officer properly read the man his Miranda rights. The man stated that he understood his rights and repeated his confession, although he left out any discussion of the murder weapon. At his trial, the man admitted that both of his statements were voluntarily made, but argued that they should each be suppressed, along with the weapon, all of which were fruit of a Miranda violation. Which, if any, pieces of evidence should be suppressed? A. Both of the confessions and the weapon should be suppressed. B. The first confession and the weapon should be suppressed, but the second confession should be admitted. C. The first confession should be suppressed, but the weapon and the second confession should be admitted. D. Both of the confessions should be suppressed, but the weapon should be admitted. Question #208. OA452: Another "Expert" Witness Unmasked; Bernie's Filibuster Explained A factory is located on the edge of a town. The factory is a major producer of plastics, which it supplies to manufacturers across the country. With the development of new technology and products, there is a great demand for a new type of sturdier plastic. In order for the factory to produce this plastic, the factory began using chemical reactions that released a toxic gas into the air. After the factory began production of this plastic, hundreds of citizens in the town started to complain about the gas. It brings with it an unpleasant odor and also causes sickness. This gas has been proven to cause many significant illnesses and thus is jeopardizing the health of the community. A citizen of the town sues the factory for public nuisance. Will the citizen recover? A. Yes, if the use or enjoyment of the citizen’s property has been unreasonably interfered with. B. Yes, if the citizen has suffered some sort of damages or injuries that are unique to those suffered by the rest of the town. C. No, if the factory has not acted unreasonably and has taken all feasible precautions to limit the gas. D. No, if it is deemed that the factory is not acting intentionally to spread the gas across the town. Question #209. OA454: GEORGIA!!! (oh and a fascist coup) An explosives manufacturer contracted with a fireworks company in a signed writing for the sale of 100 red, white, and blue-themed fireworks, 50 of them to be delivered on June 1 and the remaining 50 to be delivered on July 1. The agreement did not specify where the delivery would take place or when payment was due. On June 1, the explosives manufacturer delivered 49 fireworks and explained that one of the fireworks unexpectedly suffered water damage but that a replacement would be delivered within two days. Which of the following states the rights and duties of the fireworks company? A. The fireworks company is entitled to accept any of the 49 fireworks, reject the rest, and cancel the contract for the remaining fireworks due on July 1. B. The fireworks company is entitled to accept any number of the 49 fireworks and reject the rest, but it may not cancel the contract as to any fireworks due on July 1. C. The fireworks company must accept the 49 fireworks, but it may cancel the rest of the contract for the remaining fireworks due on July 1. D. The fireworks company must accept the 49 sets and is not entitled to cancel the rest of the contract. Question #210. OA456: Insurrections Have Consequences A state law provides funding for public students using a complex formula. One of the major components of the formula takes into account property values surrounding the schools. This leads to a disparity in student funding as schools located in areas where property values are high have substantially more funding than schools located in areas where property values are not high. A group of students from a school located in an area with low property values challenges this law on the basis that it violates the Equal Protection Clause of the Fourteenth Amendment. Who will prevail? A. The state, because the state can show that the law is necessary to achieve a compelling government interest. B. The state, because the students will not be able to show that the law is not rationally related to any legitimate government interest. C. The students, because the students will be able to show that the law is not substantially related to an important government interest. D. The students, because the state will not be able to show that the law is rationally related to a legitimate government interest. Question #211. OA458: Biden's Amazing First Day A woman sued a man for physical damages based on a federal statutory claim. The incident occurred within State A, but the woman brought suit in federal court. State A recognizes all of the privileges recognized at common law. At trial, the man called the woman's treating physician to testify as to statements made in confidence to him by the woman in furtherance of medical treatment. The woman objects and claims that the physician-patient privilege applies to her statements. Which law should the court apply to the claim of privilege? A. The state law, and the court should therefore recognize the claim of privilege. B. The federal law, and the court should therefore recognize the claim of privilege. C. The state law, and the court should therefore reject the claim of privilege. D. The federal law, and the court should therefore reject the claim of privilege. Question #212. OA460: Did the Courts Really "Refuse to Hear Trump's Evidence" of Voter Fraud? (No.) A landlord commercially leased his building to a woman who intended to open an ice cream shop. The woman installed an extensive amount of equipment in the building, which was permitted by the lease agreement, and she began making ice cream. She stored her ice cream in a freezer. However, the building's cooling system broke and her ice cream all melted. She asked the landlord to fix the cooling system, but he refused. She had to temporarily close her store while searching for a mechanic to fix the cooling system. While the building was unusable for the purpose of storing ice cream, the woman remained on the premises to work on new marketing projects. Angry at the landlord for not fixing the cooling system, which has still not been fixed, the woman refused to make the next rent payment. The landlord began eviction proceedings, and in response, the woman sued the landlord for breach. Assuming the lease agreement is silent on this matter, who will prevail? A. The landlord, because the woman had no legal reason for withholding rent payments. B. The landlord, because the woman surrendered the property when she did not pay rent. C. The woman, because the landlord breached the implied warranty of habitability. D. The woman, because the landlord breached the covenant of quiet enjoyment by constructively evicting her prior to the withholding of rent. Question #213. OA462: Rittenhouse Team Commits Perjury; DC Statehood and Impeachment! A woman was walking across the street when she was struck by a vehicle traveling at a high rate of speed. As she was laying on the pavement dying, she told a bystander who was at her side, "I know I am dying. I want to tell you before I die that I saw my neighbor steal the Madonna painting from the museum." The woman then died. In a civil suit by the museum against the neighbor for conversion, is the woman's statement to the bystander admissible? A. Yes, as an excited utterance. B. Yes, as a dying declaration. C. No, because it would violate the neighbor's Sixth Amendment right of confrontation. D. No, it is inadmissible hearsay. Question #214. OA464: Impeachment 2 Secret of the Ooze A father lived with his son, who was an alcoholic. When drunk, the son often became violent and physically abused his father. As a result, the father always lived in fear. One night, the father heard his son on the front stoop making loud obscene remarks. The father was certain that his son was drunk and was terrified that he would be physically beaten again. In his fear, he bolted the front door and took out a revolver. When the son discovered that the door was bolted, he kicked it down. As the son burst through the front door, his father shot him four times in the chest, killing him. In fact, the son was not under the influence of alcohol or any drug and did not intend to harm his father. At trial, the father presented the above facts and asked the judge to instruct the jury on self-defense. How should the judge instruct the jury with respect to self-defense? A. Give the self-defense instruction, because it expresses the defense’s theory of the case. B. Give the self-defense instruction, because the evidence is sufficient to raise the defense. C. Deny the self-defense instruction, because the father was not in imminent danger from his son. D. Deny the self-defense instruction, because the father used excessive force. Question #215. OA466: Opening Arguments Blocked in Australia?! A man sued a railroad for personal injuries suffered when his car was struck by a train at an unguarded crossing. A major issue is whether the train sounded its whistle before arriving at the crossing. The railroad has offered the testimony of a resident who has lived near the crossing for 15 years. Although she was not present on the occasion in question, she will testify that, whenever she is home, the train always sounds its whistle before arriving at the crossing. Is the resident's testimony admissible? A. No, due to the resident's lack of personal knowledge regarding the incident in question. B. No, because habit evidence is limited to the conduct of persons, not businesses. C. Yes, as evidence of a routine practice. D. Yes, as a summary of her present sense impressions. Question #216. OA468: SCOTUS News! Trump's Taxes, Affirmative Action, and Title X To keep its public school expenditures under control in a time of increasing costs, a state passed a law providing that children who have not lived in the state for at least one year cannot attend public schools in the state. Which of the following statements about this law is most accurate as a matter of constitutional law? A. The one-year residence requirement is valid because it does not affect any fundamental right or suspect class. B. State durational residence requirements that are established for publicly funded services are constitutional because they relate to government operations reserved exclusively to the states by the Tenth Amendment. C. Because publicly funded education is a fundamental constitutional right, a state may not deny it to any class of persons who reside in that state. D. State durational residence requirements established for this kind of publicly funded service solely for the purpose of reducing state expenditures violate the equal protection clause of the Fourteenth Amendment. Question #217. OA470: Can Harris Just Overrule the Parliamentarian? A man has four German shepherd dogs that he has trained for guard duty and that he holds for breeding purposes. The man has "Beware of Dogs" signs clearly posted around a fenced-in yard where he keeps the dogs. The man's nextdoor neighbor frequently walks past the man’s house and knows about the dogs’ ferocity. One summer day, the neighbor entered the man’s fenced-in yard to retrieve a snow shovel that the man had borrowed during the past winter. The neighbor was attacked by one of the dogs and was severely injured. In a suit against the man, is the neighbor likely to prevail? A. No, because the neighbor knew that the man had dangerous dogs in the yard. B. No, because the neighbor was trespassing when he entered the man’s property. C. Yes, because the neighbor was an invitee for the purpose of retrieving the shovel. D. Yes, because the man was engaged in an abnormally dangerous activity. Question #218. OA472: The American Rescue Plan - You Made This Happen! A woman from State A filed an action against a retailer in a state court in State B. The complaint alleged that the retailer had not delivered $100,000 worth of goods for which the woman had paid. Twenty days after being served, the retailer, which is incorporated in State C and has its principal place of business in State B, filed a notice of removal in a federal district court in State B. Was the action properly removed? A. No, because the notice of removal was not timely filed. B. No, because the retailer is a citizen of State B. C. Yes, because the parties are citizens of different states and more than $75,000 is in controversy. D. Yes, because the retailer is a citizen of both State B and State C. Question #219. OA474: FBI "Faked" Kavanaugh Investigation?! A man owned a house where he lived with his family. The man was convicted of selling large quantities of an illegal drug from his house. Acting under a state law authorizing the destruction of buildings that are used for illegal activity, the city destroyed the man’s house. The man’s family then rented an apartment and demanded that the city pay the rent for that temporary residence. The family relied on a state law providing that any person who was dispossessed of his or her place of residence because of the actions of city officials was entitled to replacement housing at the city’s expense until permanent substitute housing could be found. When the city refused to pay the rent for the apartment, the man’s family sued the city in a state trial court claiming a right to such payment under both the state law and the due process clause of the Fourteenth Amendment to the United States Constitution. The highest state court ruled for the family. Although the court decided that the family had no right to payment under the state law, it held that the Fourteenth Amendment entitled the family to payment of the rent for the temporary apartment. In its opinion, the highest state court indicated that in several of its decisions it had found cities liable for compensation in similar situations on the basis of the due process Clause of the state constitution. But the highest state court declined to base its holding on the state constitution because that issue had not been properly raised in the case. The city then filed a petition for a writ of certiorari in the United States Supreme Court. Does the Court have jurisdiction to review the merits of this case? A. Yes, because the highest state court based its decision wholly on federal law grounds. B. Yes, because the federal and state law issues in this case are so intertwined that a resolution of the federal law issues is necessary to facilitate a proper determination of the state law issues. C. No, because the decision of the highest state court renders the case moot. D. No, because independent state law grounds could have been used to justify the result in this case. Question #220. OA476: Sidney Powell's Defense Is Even Worse Than You Think A sporting goods shop contracted with a publisher to buy, for sale in its store, 1,200 posters featuring a professional golfer. During production, the image of the golfer was inadvertently reversed and the right-handed golfer appeared to be left-handed. When the posters were delivered on the date provided in the contract, the sporting goods shop noticed the discrepancy, which had no provable significant impact on the effectiveness of the poster. In the opinion of the shop management, however, the posters did not look as good as they had in the catalog from which the shop had ordered them. Is the sporting goods shop legally entitled to reject the posters? A. No, because the nonconformity does not materially alter the value of the posters to the sporting goods shop. B. No, because the publisher must be given an opportunity to cure the nonconformity before the sporting goods shop can reject the posters. C. Yes, because the posters do not conform to the contract. D. Yes, because the publisher has breached an implied warranty of fitness for a particular purpose. Question #221. OA478: Why Biden's Judicial Picks Are Hardball A young man suggested to his friend that they steal a large screen TV from a neighbor's house. The friend was angry with the young man and decided to use the opportunity to get even with him by having him arrested. So the friend said he would help and that night he drove the young man to the neighbor's house. The young man broke in while the friend remained outside. The friend called the police on his cell phone and then drove away. Police officers arrived at the scene just as the young man was carrying the TV out the back door. The jurisdiction defines crimes as at common law. Of what crime, if any, can the friend properly be convicted? A. No crime. B. Conspiracy. C. Burglary. D. Conspiracy and larceny. Question #222. OA480: Why Both Sides of the Chauvin Trial Cite Graham v. Connor A 50-year-old nurse who had been fired from his job at a hospital told his attorney, "I was fired because of my age, and I want to sue the hospital." Based on this information, the attorney filed an age discrimination complaint against the hospital in federal court. As it turned out, the hospital had hired a 52-year-old man as the nurse’s replacement, a fact that rendered an age discrimination claim unavailable. The hospital responded to the complaint by filing a motion for sanctions against the nurse’s attorney. Is the court likely to grant the hospital's motion? A. No, because sanctions are not proper against the attorney of a represented party. B. No, because the hospital failed to give the attorney the chance to withdraw the complaint in advance of filing the motion with the court. C. Yes, because the nurse’s attorney failed to conduct a reasonable pre-filing inquiry. D. Yes, because the nurse’s complaint contained legal contentions that were not warranted by existing law based on the facts in this case. Question #223. OA482: Justice for Daunte Wright A daughter was appointed guardian of her elderly father following an adjudication of his mental incompetence. The father had experienced periods of dementia during which he did not fully understand what he was doing. The father later contracted to purchase an automobile at a fair price from a seller who was unaware of the guardianship. At the time of the purchase, the father was lucid and fully understood the nature and purpose of the transaction. What is the legal status of the transaction? A. The contract is enforceable, because a reasonable per- son in the situation of the seller would have thought that the father had the capacity to make the contract. B. The contract is enforceable, because it was made on fair terms and the seller had no knowledge of the father’s guardianship. C. The contract is void, because the father was under guardianship at the time it was made. D. The contract is voidable at the option of the father. Question #224. OA484: Chauvin Guilty! What To Expect Next Two sisters own a single tract of land as tenants in common, each holding a one-half interest. The younger sister entered into a three-year written lease with a tenant; the lease described by metes and bounds a specified portion of the land, which consisted of about 40% of the total tract. The tenant went into sole possession of the leased portion of the land. The older sister has sued both the younger sister and the tenant to establish the older sister's right to possession of the leased portion of the land. Who is likely to prevail? A. The older sister, because the younger sister cannot unilaterally partition the land without the older sister's consent. B. The older sister, because the younger sister may not lease her undivided interest in the land without the older sister's consent. C. The younger sister and the tenant, because the older sister has been excluded only from the specified portion of the land subject to the lease, which makes up less than one-half of the land's total area. D. The younger sister and the tenant, because the younger sister's lease to the tenant was necessarily for less than a fee simple interest. Question #225. OA486: Apportionment Might NOT Be Final A state law prohibits any barbershop licensed by the state from displaying posters in support of any current candidate for public office or displaying or distributing any campaign literature in support of such a candidate. No other kinds of posters or literature are subject to this prohibition, nor are any other types of commercial establishments in the state subject to similar prohibitions. Is this law constitutional? A. No, because it treats barbershops differently from other commercial establishments. B. No, because it imposes a restriction on the content or subject matter of speech in the absence of any evidence that such a restriction is necessary to serve a compelling state interest. C. Yes, because it leaves political candidates free to communicate their campaign messages to voters by other means. D. Yes, because the operation of a licensed barbershop is a privilege and therefore is subject to any reasonable restriction imposed by the state. Question #226. OA488: A New Trial for Chauvin? A defendant was prosecuted for mail fraud. At trial, the defendant moved to have all witnesses excluded from the courtroom, and the court granted the motion. The government named the investigating FBI agent as its designated representative. Upon learning that the agent would be giving testimony during the trial, the defendant moved that the agent also be excluded from the courtroom. Should the defendant’s motion be granted? A. No, provided that the government can show that the agent’s presence is essential to the presentation of its case. B. No, because the government has a right to have its designated representative remain in the courtroom throughout the trial. C. Yes, because the agent's testimony might be influenced by the testimony of other witnesses. D. Yes, because the defendant has a right to exclude all persons who may be called as government witnesses. Question #227. OA490: Chauvin Sentencing - Four Aggravating Factors A man entered a tavern in an obviously intoxicated condition, was refused service, and was ordered to leave and escorted out. Just after leaving the tavern, the man staggered across the road toward a liquor store. As he was crossing the road, the man was struck by a car and severely injured. The man sued the tavern for his personal injuries. At trial, the evidence established the facts as set out above. At the close of the evidence, both parties moved for judgment as a matter of law. How should the court rule on these motions? A. Deny both motions and submit the case to the jury, because reasonable jurors could conclude that the accident was foreseeable. B. Deny both motions and submit the case to the jury, because a tavern is a place of public accommodation. C. Grant the tavern’s motion, because there is no evidence that the tavern breached a duty to the man. D. Grant the man’s motion, because it is undisputed that by being ejected from the tavern, the man was put at risk. Question #228. OA492: Court Locks Mississippi Citizens Out of Amending Their Own Constitution An attempt was made to hijack a commercial airliner while it was in flight from San Francisco to New Orleans. Within minutes, however, the hijacker was seized, and the plane proceeded to its destination. Upon the plane’s arrival, television stations broadcast pictures of the passengers as they disembarked. Among the passengers pictured on television was a businessman who was supposed to be in Chicago on company business. The disclosure that the businessman was in New Orleans and not in Chicago at the time resulted in the loss of his position with his company and great humiliation and embarrassment for him. If the businessman asserts a claim against the television stations for broadcasting his picture as he disembarked, is he likely to prevail? A. Yes, because the businessman’s location was revealed against his wishes. B. Yes, because publication of the television pictures caused the businessman pecuniary loss. C. No, because the humiliation and embarrassment did not result in physical harm to the businessman. D. No, because the scene shown on television was newsworthy. Question #229. OA494: Florida's Pointless, Unconstitutional Social Media Law A protester brought an action in federal court against a police officer, alleging that the officer's use of force in arresting the protester violated the protester’s federal civil rights. During the jury trial, eyewitnesses gave conflicting testimony on the arrest. At the close of evidence, the protester moved for judgment as a matter of law, which the court denied. The court instructed the jury that the protester’s burden of proof was clear and convincing evidence, rather than the correct burden of preponderance of the evidence. The jury returned a verdict for the officer, and the court entered judgment accordingly. What is the protester’s best option for challenging the judgment? A. Seek a new trial, because the jury instruction affected the protester’s substantial rights. B. Seek a new trial, because the verdict was against the clear weight of the evidence. C. Seek judgment as a matter of law, because the jury did not have legally sufficient evidence to find for the officer. D. Seek judgment as a matter of law, because the jury’s findings were clearly erroneous. Question #230. OA496: Stanford Law Forgets How Law Works A company contracted with a builder to construct a new corporate headquarters for a fixed price of $100 million. At the time of the contract, structural steel was widely available and was included in the contract as a $6 million item. Before work began on the project, tornado damage shut down the production facility of the biggest structural steel supplier in the country, and the price of structural steel increased by 20% as a result. The builder informed the company of the steel price increase, and the parties then orally agreed to increase the project price to $101 million. The builder proceeded with construction and delivered the project on time. The company paid the builder $100 million but refused to pay the additional $1 million. If the builder sues the company for $1 million, is the builder likely to prevail? A. No, because the modification was never reduced to a writing signed by the party to be charged. B. No, because there was no consideration for the modification of the contract. C. Yes, because the company’s promise was supported by consideration. D. Yes, because the modification was fair and equitable in view of the unanticipated increase in the price of structural steel. Question #231. OA498: The Garland DoJ Coverage is Completely Wrong At a defendant's trial for extortion, the prosecutor called a witness expecting her to testify that she had heard the defendant threaten a man with physical harm unless the man made payoffs to the defendant. The witness denied ever having heard the defendant make such threats, even though she had testified to that effect before the grand jury. The prosecutor now seeks to admit the witness’s grand jury testimony. How should the court rule with regard to the grand jury testimony? A. Admit the testimony, because it contains a statement by a party-opponent. B. Admit the testimony, both for impeachment and for substantive use, because the witness made the inconsistent statement under oath at a formal proceeding. C. Admit the testimony under the former testimony exception to the hearsay rule. D. Exclude the testimony for substantive use, because it is a testimonial statement. Question #232. OA500: Eminent Domain! A man arranged to have custom-made wooden shutters installed on the windows of his home. The contractor who installed the shutters did so by drilling screws and brackets into the exterior window frames and the shutters. The man later agreed to sell the home to a buyer. The sales agreement did not mention the shutters, the buyer did not inquire about them, and the buyer did not conduct a walkthrough inspection of the home before the closing. The man conveyed the home to the buyer by warranty deed. After the sale closed, the buyer noticed that the shutters and brackets had been removed from the home and that the window frames had been repaired and repainted. The buyer demanded that the man return the shutters and pay the cost of reinstallation, claiming that the shutters had been conveyed to him with the sale of the home. When the man refused, the buyer sued. Is the buyer likely to prevail? A. No, because the sales agreement did not mention the shutters. B. No, because the window frames had been repaired and repainted after removal of the shutters. C. Yes, because the shutters had become fixtures. D. Yes, because the man gave the buyer a warranty deed and the absence of the shutters violated a covenant of the deed. Question #233. OA502: Why Critical Race Theory is Indispensable The president of a pharmaceutical firm received a report from his testing bureau that a manufactured lot of the firm’s anti-cancer prescription medication was well below strength. Concerned about being able to fulfill contractual commitments, the president instructed his staff to deliver the defective lot. A cancer patient who had been maintained on the drug died shortly after beginning to take the defective pills. Medical evidence established that the patient would have lived longer had the drug been at full strength, but would have died before long in any event. The president was convicted of murder. On appeal, he argues that his conviction should be reversed. Should the conviction be reversed? A. No, because the intentional delivery of adulterated or mislabeled drugs gives rise to strict criminal liability. B. No, because the jury could have found that the president’s conduct was sufficiently reckless to constitute murder. C. Yes, because distribution of the defective lot was only a regulatory offense. D. Yes, because the cancer, not the president’s conduct, was the proximate cause of death of the patient. Question #234. OA504: Cosby Released in Nonsense Ruling An entrepreneur from State A decided to sell hot sauce to the public, labeling it "Best Hot Sauce." A company incorporated in State B and headquartered in State C sued the entrepreneur in federal court in State C. The complaint sought $50,000 in damages and alleged that the entrepreneur's use of the name "Best Hot Sauce" infringed the company’s federal trademark. The entrepreneur filed an answer denying the allegations, and the parties began discovery. Six months later, the entrepreneur moved to dismiss for lack of subject-matter jurisdiction. Should the court grant the entrepreneur's motion? A. No, because the company’s claim arises under federal law. B. No, because the entrepreneur waived the right to challenge subject-matter jurisdiction by not raising the issue initially by motion or in the answer. C. Yes, because although the claim arises under federal law, the amount in controversy is not satisfied. D. Yes, because although there is diversity, the amount in controversy is not satisfied. Question #235. OA506: Trump's Desperate, Harebrained Lawsuit Is Garbage and Deserves Sanctions An investor from State A filed an action against his State B stockbroker in federal court in State A. The summons and complaint were served at the stockbroker’s office in State B, where the process server handed the documents to the stockbroker’s administrative assistant. The stockbroker has answered the complaint, asserting the defense of improper service of process. Assume that both states’ requirements for service of process are identical to the requirements of the Federal Rules of Civil Procedure. Is the court likely to dismiss the action for improper service of process? A. No, because service was made on a person of suitable age found at the stockbroker's place of employment. B. No, because the stockbroker waived her claim for improper service of process by asserting it in her answer. C. Yes, because an individual defendant may not be served by delivering process to a third party found at the defendant's place of employment. D. Yes, because the process of State A courts is not effective in State B. Question #236. OA508: Texas Democrats Flee the State to Stop Republican Voter Suppression Bill A truck driver from State A and a bus driver from State B were involved in a collision in State B that injured the truck driver. The truck driver filed a federal diversity action in State B based on negligence, seeking $100,000 in damages from the bus driver. What law of negligence should the court apply? A. The court should apply the federal common law of negligence. B. The court should apply the negligence law of State A, the truck driver’s state of citizenship. C. The court should consider the negligence law of both State A and State B and apply the law that the court believes most appropriately governs negligence in this action. D. The court should determine which state’s negligence law a state court in State B would apply and apply that law in this action. Question #237. OA510: Liberty University Punished Women Who Reported Sexual Assaults, According to Complaint A patent holder brought a patent infringement action in federal court against a licensee of the patent. The patent holder believed that a jury would be more sympathetic to his claims than a judge, and asked his lawyer to obtain a jury trial. What should the lawyer do to secure the patent holder's right to a jury trial? A. File and serve a complaint that includes a jury trial demand. B. File and serve a jury trial demand at the close of discovery. C. File and serve a jury trial demand within 30 days after the close of the pleadings. D. Make a jury trial demand at the initial pretrial conference. Question #238. OA512: Actions Continue to Have Consequences! A consumer from State A filed a $100,000 products liability action in federal court against a manufacturer incorporated and with its principal place of business in State B. The consumer claimed that a flaw in the manufacturer's product had resulted in severe injuries to the consumer. In its answer, the manufacturer asserted a third-party complaint against the product designer, also incorporated and with its principal place of business in State B. Believing that the consumer had sued the wrong defendant, the manufacturer claimed both that the designer was solely responsible for the flaw that had led to the consumer's injuries and that the manufacturer was not at fault. The designer is aware that the manufacturer did not follow all of the designer's specifications when making the product. Which of the following arguments is most likely to achieve the designer's goal of dismissal of the third-party complaint? A. The court does not have subject-matter jurisdiction over the third-party complaint, because both the manufacturer and the designer are citizens of State B. B. The manufacturer failed to obtain the court’s leave to file the third-party complaint. C. The manufacturer's failure to follow the designer's specifications caused the flaw that resulted in the consumer's injuries. D. The manufacturer's third-party complaint failed to state a proper third-party claim. Question #239. OA514: I Love It When A Plan Comes Together A wholesaler brought a federal diversity action against a large pharmaceutical company for breach of contract. During jury selection, one potential juror stated that five years earlier he had been an employee of the company and still owned several hundred shares of its stock. In response to questioning from the judge, the potential juror stated that he could fairly consider the evidence in the case. The wholesaler’s attorney has asked the judge to strike the potential juror for cause. Should the judge strike the potential juror for cause? A. No, because the potential juror said that he could fairly consider the evidence in the case. B. No, because the wholesaler’s attorney could use a peremptory challenge to strike the potential juror. C. Yes, because other potential jurors still remain available for the jury panel. D. Yes, because the potential juror is presumed to be biased because of his relationship to the company. Question #240. OA516: We Turned Down Ads for a Scam Company Called CrowdHealth. Other Shows May Not. Send Them This! A man brought a federal diversity action against his insurance company, alleging that the company had breached its duty under his insurance policy by refusing to pay for his medical expenses resulting from a mountain-biking accident. At the jury trial, the man presented evidence that he had paid all premiums on the insurance policy and that the policy covered personal-injury-related medical expenses arising from accidents. After he rested his case, the company presented evidence that a provision of the policy excluded payment for injury-related expenses resulting from an insured’s "unduly frisky" behavior. The company also presented a witness who testified that the accident had occurred in an area where posted signs warmed bikers not to enter. The man did not cross-examine the witness. After resting its case, the company moved for judgment as a matter of law. Should the court grant the motion? A. No, because a motion for judgment as a matter of law must first be made at the close of the plaintiffs case-in-chief. B. No, because whether the man’s behavior was unduly risky is a question of fact for the jury to resolve. C. Yes, because the company’s uncontradicted evidence of the man’s unduly risky behavior means that no reasonable jury could find that the policy covers his injuries. D. Yes, because the man waived his right to rebut the company’s evidence by not addressing the "unduly risky" policy provision in his case-in-chief. Question #241. OA518: Terpsichore Tries Her Hand at Frivolous Lawsuits A motorcyclist was involved in a collision with a truck. The motorcyclist sued the truck driver in state court for damage to the motorcycle. The jury returned a verdict for the truck driver, and the court entered judgment. The motorcyclist then sued the company that employed the driver and owned the truck in federal court for personal-injury damages, and the company moved to dismiss based on the state-court judgment. If the court grants the company’s motion, what is the likely explanation? A. Claim preclusion (res judicata) bars the motorcyclist’s action against the company. B. Issue preclusion (collateral estoppel) establishes the company’s lack of negligence. C. The motorcyclist violated the doctrine of election of remedies. D. The state-court judgment is the law of the case. Question #242. OA520: Kraken Lawyers Hit with Sanctions! YASSSSSSSSSSSSSSSSSSSS A student at a private university sued the university in federal court for negligence after he fell from scaffolding in a university-owned theater building. At trial, after briefing from both parties, the court permitted the jury to hear testimony that there had been several previous accidents in the same building. The jury found for the student, and the university appealed. One of the university's arguments on appeal is that the testimony about the previous accidents should have been excluded as irrelevant and highly prejudicial. Which standard of review applies to this argument? A. Abuse of discretion. B. Clearly erroneous. C. De novo. D. Harmless error. Question #243. OA522: Roe v. Wade is Dead. A local bartender owns Blackacre, which is an undeveloped piece of property next to his bar. On March 1, the bartender conveyed Blackacre to an investor who paid value and wanted to build an apartment complex on the property. The investor did not immediately record his interest. On April 1, the bartender sold the still undeveloped Blackacre to his son. The son did not know about the conveyance to the investor. The son immediately recorded the deed. On May 1, the investor recorded the deed and had a construction company start digging on the site. On June 1, the son sold Blackacre to his friend. On July 1, the bartender died. His will left Blackacre to his brother, who immediately recorded the deed. On August 1, the friend recorded his deed. The jurisdiction has a race-notice recording statute. In an action to quiet title, who will have priority rights to Blackacre? A. The investor B. The son C. The friend D. The brother Question #244. OA524: Laurence Tribe Thinks Grendel's Den Case Will Stop the Texas Abortion Law. Is He Right? O, owner of Blackacre, wanted to get rid of his land, Blackacre, and move somewhere warmer, but he still wanted to preserve the landscaping of the beautiful property. O drew up a deed of conveyance to her daughter, A. The deed stated the following: "To A, for so long as she does not remove any of the trees or flowers, and if she does then I have the right to reenter." A gladly accepted the deed and managed O's property nicely for a few years, but she soon decided she didn't like the large oak tree in the front yard. She decided to have it removed. At the moment the tree was removed, what interests did O and A have in Blackacre, respectively? A. O right of entry; A - Fee Simple Determinable. B. O Fee simple; A - Nothing. C. O right of entry; A - Nothing. D. O right of entry; A - Fee simple subject to a condition subsequent. Question #245. OA526: Updates from Five Cases on the OA Docket A plaintiff filed an action against a defendant in federal court, seeking damages for negligence after the defendant lost control of his car on a snowy day and collided with the plaintiffs car. The defendant has an insurance policy that will pay for the defense of the defendant's claim as well as most or all of the damages that the defendant is deemed liable for. Is the defendant's insurance policy subject to discovery by the plaintiff? A. Yes, the defendant must provide the insurance policy to the plaintiff if the plaintiff makes a proper request for production of documents that includes the insurance policy. B. Yes, the defendant must provide the insurance policy to the plaintiff even if the plaintiff does not request it. C. No, the defendant need not disclose the policy because it is not relevant to the plaintiff's claim or the defendant's defense. D. No, because insurance policies are precluded from being admitted into evidence if agency, control, or impeachment is not at issue as admission of an insurance policy into evidence is considered to violate public policy. Question #246. OA528: Disgraced Ex-President Sues Niece Mary Trump The local police department was trying to solve a recent series of murders. Possessing reasonable suspicion that the perpetrator resided in a neighborhood on the west side of the city, the police decided to search through the garbage of all the residents on garbage day. One of the houses the police visited belonged to the local tailor. The tailor had recently left two bags of garbage at the end of the driveway. They were both tied tightly and sealed well. While going through the bags, the police found evidence that placed the tailor at the scene of many of the murders. The tailor was, subsequently charged, and the evidence found in the garbage bags was offered at trial. The tailor moved to suppress the evidence, arguing that the search violated the Fourth Amendment's protection against unreasonable searches and seizures. Should the evidence be suppressed? A. Yes, because the search does not fall into any of the exceptions to the warrant requirement. B. Yes, because the officers were basing their search on a mere reasonable suspicion. C. No, because the plain view exception justifies this search. D. No, because the 4th Amendment does not apply to this search. Question #247. OA530: Andrew Testifies to Congress on Christian Healthshare Ministries A man entered into a valid contract with Painter A to touch up the paint on his house. The man agreed to pay Painter A $400, as well as supply the paint for the job. Payment was due after the job was completed. The paint would have the cost the man $50. Before the man purchased the paint, Painter A breached the contract and refused to paint the man's house. The man subsequently entered into a second valid contract with Painter B to complete the same work that Painter A was going to do. The man agreed to pay Painter B $600 and Painter B provided her own paint. Painter B completed the job and the main promptly paid the $600. The man then timely filed suit against Painter A for breach of contract. How much will the man recover from Painter A? A. $150. B. $200. C. $350. D. $400. Question #248. OA532: OK I'll Bite What is the Debt Ceiling A man lives in a remote area in the Upper Peninsula of State A. Since he lives so far from town, he prefers to make one large trip to the local convenience store each month to purchase all of his supplies. During the winter, the area gets so much snow that the roads are often un-passable by vehicle. Thus, the man decided he wanted to purchase a state-of-the-art snowmobile that would allow him to transport his supplies back and forth from town. Since the best deals are during the summer, the man went to purchase his snowmobile in July. He asked the salesperson to show him "the best snowmobile you've got." The salesperson took him over to a large snowmobile with red and white stripes. The salesperson said, "I think this is exactly what you're looking for!" The box advertised that the snowmobile could carry up to 200 pounds. Impressed, the man replied "I'll take one! But can I get it one with blue and black stripes?" The salesperson readily agreed and said it would be delivered in one week. The correct model snowmobile with blue and black stripes was indeed delivered on time the next week. The man was very happy and accepted the snowmobile. When the first snowfall hit in November, the man tried to take it to the store to purchase his supplies. On his way back, with 100 pounds of supplies on the snowmobile, the snowmobile kept stalling due to the weight of the supplies. He had to call for help because the snowmobile could not get him home. When he called the store, he was told that there was a problem with the bearings in that model that prevented it from operating correctly when carrying heavy loads. However, it still works properly in every other scenario. The man asked for his money back. A. Yes, because the man can sue under the theory of implied warranty of fitness for a particular purpose. B. Yes, because the man properly revoked his acceptance of the snowmobile. C. No, because a perfect tender delivery was made and the man did not properly reject the snowmobile. D. No, because the man is still able to use the snowmobile; he simply cannot carry supplies on it. Question #249. OA534: State and Federal Agencies Now Fighting EACH OTHER in Blizzard Activision Case A recent deluge of rain in state a left the ground very unstable. In an effort to protect against infrastructure problems. Congress passed a law imposing a strict maximum weight for all shipping trucks attempting to enter State A on its toll highway. The weight was lower than the weight limit of any of the surrounding states. Companies that operated solely in state A were unburdened as they did not need to pass through the laying checkpoints. However, companies that utilize shipping trucks that drove through state A were forced to either drive around state A or lessen their loads in order to pass through the state. A business in state B decided to challenge this law. Which are the following arguments would be the businesses best argument that the law is unconstitutional? A. the law imposes an unreasonable burden on interstate commerce. B. The law discriminates against out-of-state citizens in violation of Article 4. C. the law is not necessarily and narrowly tailored to serve a compelling government interest. D. the law is not rationally related to a legitimate government interest. Question #250. OA538: The Rittenhouse Trial Instructions, Explained A woman hired a driver to drive her across town to her sister's house. The driver negligently failed to pay attention to where he was going and drove in the opposite direction of the sister's house. By the time the driver realized his error, it was too late to drive the woman back home. Thus, the driver paid for the woman to stay in a hotel room for the night. While the woman was dining at the hotel that evening, a man pickpocketed the woman's expensive pair of glasses out of her purse. The woman sued the driver for the value of the glasses. Should she prevail? A. Yes, since the driver negligently failed to pay attention to where he was going. B. Yes, so long as the woman was not negligent in failing to notice the driver was going the wrong way. C. No, since larceny is not a foreseeable result of the breach by the driver. D. No, since the act of larceny was committed by a third person, not the driver. Question #251. OA540: The Bizarre Case of Steven Donziger A defendant is on trial for a robbery that recently occurred at a department store. Witnesses saw two men fleeing the scene, but no one got a good look at them. Two days after the robbery, a co-conspirator went to the police station, no longer able to keep his secret. He said he needed to tell the officers something. The lead investigator took the co-conspirator into an office. The co-conspirator said, "I can't take it anymore, [the defendant] and I committed the robbery." The defendant was subsequently arrested. The co-conspirator has not said a word since making that statement and refuses to testify at any hearing or trial. The prosecutor asks the lead investigator to testify regarding the statement made by the co-conspirator. The defendant moves to suppress the statement Is the co-conspirator’s statement likely to be admitted? A. Yes as a statement by a co-conspirator B. Yes as a statement against interest C. No, because it would violate the defendant's constitutional rights D. No, because the statement is hearsay and does not fall under any exception. Question #252. OA542: Oklahoma Supreme Court Overturns Verdict Against Opioid Companies On January 1, a man offered to sell his neighbor his lawn mower for $400. The neighbor was considering whether to purchase the lawn mower. The neighbor said to the man, "I cannot decide whether I want to buy your lawn mower. If I pay you $10.00 will you leave the offer open until January 5?" The man agreed to leave the offer open until January 5, and the neighbor paid the man $10.00. On January 2, the neighbor told the man, "I reject your offer." The man did not take any steps in reliance on the neighbor's rejection. On January 3, the neighbor told the man, "I accept your offer." Is there a contract between the neighbor and the man? A. No, because the January 3 statement constituted a counteroffer. B. No, because $10.00 is not considered to be adequate consideration to hold open an offer. C. Yes, because the man did not take any steps in reliance on the neighbor's January 2 rejection and the neighbor accepted the offer prior to the January 5 deadline. D. Yes, because an option contract can always be accepted during the time period that the offer is held open. Question #253. OA544: Debunking the Left's Rittenhouse Trial Myths One winter, a man purchases swimming goggles from a nearby dive shop. The man does not use the swimming goggles until a couple months later when the weather gets warmer. Upon using the swimming goggles, he discovers that there is a defect in the lens which causes water to flood the goggles, rendering them effectively useless. The next day, the man brings the goggles back to the dive shop and demands his money back. What is the man’s best argument to receive his money back? A. The swimming goggles are defective so he is rejecting them. B. The swimming goggles are defective so he is revoking his acceptance of them. C. The dive shop breached the implied warranty of fitness for a particular purpose. D. The harm suffered by the man was a foreseeable result of the dive shop's negligence. Question #254. OA546: Three Racist Murderers Found Guilty of Murdering Ahmaud Arbery Owner granted his real estate "to museum so long as the property is always used for museum purposes." Owner then died. Owner left a will that gave "all of my estate" to a friend. Owner's only heir at law was a daughter. The museum ended up closing after a downturn in business. The museum sold its property to a developer by quitclaim deed. The developer wanted to use the land to develop a retail mall. The developer brings an appropriate action to quiet title against the museum, the daughter, and the friend. The court should find title is now in which party? A. The developer. B. The friend. C. The daughter. D. The museum. Question #255. OA548: Supreme Court Signals Roe v. Wade's Demise A man was building a home on a vacant piece of land. The man wanted to remove tree roots that stretched from a tree stump in the neighbor's yard to his own yard. The man decided to dig beneath the neighbor's yard and remove the tree roots. The digging caused no Impact on the neighbor's land, and it did not interfere with the neighbor's use or enjoyment of his land. Nonetheless, the neighbor sued the man for trespass. Which party is likely to prevail? A. The man, because his decision to remove tree stump roots was reasonable. B. The man, because he did not interfere with the use and enjoyment of the neighbor's property. C. The neighbor, because the man intentionally entered the neighbor's property without permission. D. The neighbor, because the man did not give reasonable notice to the neighbor of his intent to dig up the tree roots. Question #256. OA550: Oxford Shooter's Parents Charged A man owned 3 houses in a suburban neighborhood. One day he discovered that the second step of his front porch was very wobbly. Without the time to fix the step this month, he created a sign to hang on the handrail that leads up to the stairs. The sign said, "USE CAUTION - WOBBLY STEP - COULD CAUSE INJURIES." The sign was properly placed and easily visible. A few days later, the man invited one of his new neighbors, an elderly woman, over for lunch. The neighbor approached the stairs, saw the sign, but decided to use the staircase regardless. When she stepped on the wobbly stair, it caused her to slip. The neighbor was still healing from a recently broken hip, and the resulting fall from this slip caused her hip to re-break. If the neighbor were to sue the man for negligence, who Is most likely to win? A. The neighbor, because the man breached his duty to the neighbor to make the step as safe as soon as he learned of the dangers. B. The neighbor, because the step was the the actual and proximate cause of the injury. C. The man, because he did not breach a duty that he owed to the neighbor. D. The man, because the neighbor assumed the risk that she may fall. Question #257. OA552: Will Joe Manchin Save Democracy? No, Really! ... Stop Laughing... A man sued his boss in state court, alleging claims for nonpayment of wages under both state and federal law. The man’s friend, who was an attorney, advised the man to remove the case to federal court on the basis of federal question jurisdiction because he thought the federal court would be more sympathetic to the man’s claim. The man thereafter promptly removed the case to federal court. The man’s boss made a timely motion to remand the case to state court How will the federal court proceed? A. It will hear the federal claims but not the state claims B. It will hear the entire case because it was timely removed and involves a federal question C. It will remand the case unless the amount in controversy exceeds $75,000 D. It will remand the case to state court Question #258. OA554: 6th Circuit Overrules Atrocious 5th Circuit Ruling on Vax Mandates A defendant is prosecuted for battery after being involved in a fight at a sporting event. We'll say hockey. The defendant claims that the victim started the fight and that the defendant was acting in self-defense. At trial, the defendant called a witness that testified that the victim had a reputation for being belligerent and occasionally violent. The prosecution wishes to respond by calling its own witness that will testify that the defendant is known for his own hot temper and is feared in the community for getting violent over the smallest things. Is the prosecution's action proper? A. Yes, because when a victim's character is attacked, the prosecution can rebut with evidence of the victim's same character trait. B. Yes, because when a defendant claims self-defense, he automatically places his or her character for violence in issue. C. No, because the defendant never opened the door by placing his character for violence in the show. Or D. No, because the prosecutor is limited to introducing evidence regarding the victim's character that contradicts the defendant's evidence. Question #259. OA556: Maxwell Convicted; BBC Invites Dershowitz to Comment Despite Massive Conflict of Interest A man was driving home from work one cold December day and noticed a large, heavily decorated Christmas tree sitting outside of the city hall. The Christmas tree stood alone, was brightly lit with Christmas lights, and had several red and green ornaments decorating it. There were no other holiday or religious symbols on the city grounds. The man, who practices Judaism and who is a taxpayer in the city, was offended. He sued the city, arguing that the city exceeded its constitutional limits in erecting the Christmas tree Assuming the man has standing, will the man prevail in his lawsuit? A. Yes, because the city's display violates the Free Exercise Clause of the First Amendment. B. Yes, because the city's display violates the Establishment Clause of the First Amendment. C No, because the city may engage in free speech and spread whatever message it desires to so long as it does not prohibit citizens from engaging in the same free speech. D. No, because the city's display does not violate the First Amendment. Question #260. OA558: One Year Since 1/6 - What Merrick Garland Said (and Didn't Say) A man got divorced from his wife. The judgment of divorce granted their marital home to the man's ex-wife. The man rented an apartment nearby. The man was upset that his ex-wife had kept their dog. The man believed that the dog belonged to him as he had paid for the dog and it was his before the man and the ex-wife were married. The ex-wife refused to give the dog to the man. One night, the man was looking at pictures of his dog and got very upset that his ex-wife would not return the dog to him. The man drove to his ex-wife's house and knocked on the door. Nobody answered the door. The man could hear the dog barking. He decided to take the dog back to his house. To his surprise, the door was unlocked. He opened the door, and the dog greeted him. At that moment, he felt bad for his ex-wife and decided to leave without taking the dog. The man was later charged with burglary. What is the best argument that the man did not commit burglary? A - He did not have the requisite intent to commit burglary. B - He was not armed with a dangerous weapon. C - He did not break into the dwelling. D - He abandoned the crime. Question #261. OA560: Supreme Court Just Plain Making Sh*t Up In Vaccine Ruling An attorney was representing a nonprofit organization in merger negotiations with another nonprofit entity. During a review of the client organization's financial records, the attorney discovered that the organization had engaged in activities that jeopardized its nonprofit status. The attorney reasonably concluded that the organization’s failure to report these activities to the other nonprofit entity would constitute a fraud. The attorney had not represented the organization at the time of the activities and had not made any false representations to the other nonprofit entity during the negotiations. The attorney met with the organization’s board of directors and informed it that he would withdraw from representing the organization in the merger negotiations unless it disclosed to the other nonprofit entity the activities that jeopardized the organization’s nonprofit status. The board refused to disclose the activities and insisted that the attorney continue to represent the organization. The attorney declined and promptly withdrew from the representation without making any disclosures to the other nonprofit entity or to the state tax authorities. Is the attorney subject to discipline for his conduct in this representation? A. Yes, because the attorney did not inform the other nonprofit entity that the organization had failed to report the activities that jeopardized the organization's nonprofit status. B. Yes, because the attorney withdrew from representing the organization in the merger negotiations without the board’s permission. C. No, because the attorney withdrew from the representation after informing the board of directors about the need to disclose the activities to the other nonprofit entity. D. No, because the attorney was permitted to withdraw regardless of whether he informed the board of directors of the need to disclose the activities to the other nonprofit entity. Question #262. OA562: Supreme Court Rules Against Trump; Interview with Rep. Raja Krishnamoorthi A state prosecutor was assigned to a murder case in which the defendant had confessed to committing the crime. In the same confession, the defendant had also confessed to committing many other crimes over many years, including the murder of a young woman in another jurisdiction. In investigating the defendant's confession, which the prosecutor knew to be credible, the prosecutor discovered that another individual had been convicted and sentenced to prison for the murder of the young woman. Based on the defendant's confession, for which there was corroboration, the prosecutor knew that it was reasonably likely that the individual convicted of murdering the young woman was innocent. The prosecutor did not tell anyone about the defendant's confession to the murder of the young woman. Was the prosecutor's conduct proper? A. No, because the prosecutor failed to promptly disclose the information regarding the defendant's confession to a court or the chief prosecutor in the other jurisdiction. B. No, because the prosecutor failed to investigate to determine whether the individual convicted of murdering the young woman was innocent. C. Yes, because the defendant's confession was information relating to the prosecutor's representation of the state. D. Yes, because the prosecution of the other individual for murdering the young woman was final. Question #263. OA564: The Reteyerment; At-Will Employees Forced to Stay? and Avenatti vs Stormy Daniels! An attorney represented a company that produces chemical products. Some of the waste products of the company’s manufacturing processes are highly toxic and are reasonably certain to cause substantial bodily harm if disposed of improperly. The president of the company recently informed the attorney that a new employee had mistakenly disposed of the waste products in the ground behind the company plant, an area that is part of the source of the city’s water supply. The attorney advised the president that the company could be civilly and criminally liable for negligence in lawsuits brought by any persons harmed by the waste products. The attorney advised the president to immediately report the problem to city authorities. Fearful of adverse publicity, the president declined to do so. The attorney further advised the president that she believed the president's decision was immoral. The president continued to decline to report the matter. The attorney then informed the president that she was withdrawing from the representation and would inform the authorities herself. Immediately after withdrawing, the attorney reported the company’s conduct to the authorities. Is the attorney subject to discipline? A. Yes, because the attorney received the information in confidence and was not permitted to reveal the information without the company’s consent. B. Yes, because the company’s conduct was not intentional. C. No, because the attorney reasonably believed that the president was pursuing an imprudent and immoral course of conduct. D. No, because the attorney reasonably believed that the company’s disposal of the waste products was reasonably certain to cause substantial bodily harm. Question #264. OA566: Explosive Lawsuit Against the NFL Details Racial Bias A state trial court judge was assigned an action that sought to invalidate a testamentary trust as violating the rule against perpetuities. The judge had no experience in estate matters and did not find the briefs submitted by the parties to be helpful. Without prior notice to the parties, the judge consulted another trial judge with extensive experience in estate planning concerning the essential aspects of the rule. The judge was careful during the consultation to avoid identifying the parties or receiving any factual information that was not part of the record. The judge then personally decided the matter. She neither informed the parties about her consultation with the other judge nor gave them the opportunity to object or respond. Was the judge’s conduct proper? A. Yes, because the judge did not identify the parties. B. Yes, because the judge personally decided the matter. C. No, because the judge did not give the parties prior notice that she planned to consult ex parte with the other judge concerning the matter. D. No, because the judge neither informed the parties about her consultation with the other judge nor gave them the opportunity to object or respond. Question #265. OA568: How Christian Nationalism Fueled the Insurrection An attorney represented a mother in a child custody dispute. The father of the child was not represented by a lawyer. The mother told the attorney that she wanted to resolve the custody dispute amicably and asked the attorney to meet with the father to see if an agreement could be reached before the upcoming court date. The attorney met with the father, presented the mother's proposed custody arrangement, and asked the father to agree to it. The father asked the attorney whether the custody arrangement was fair. The attorney advised the father to hire his own lawyer if he wanted advice regarding the fairness of the arrangement. The attorney did not offer any other advice. The father hired a lawyer, and the matter turned into an antagonistic negotiation that had to be resolved in court. Is the attorney subject to discipline? A. Yes, because the attorney asked the father to agree to the proposed custody arrangement. B. Yes, because the attorney advised the father to hire his own lawyer. C. No, because the attorney offered the father no advice other than to hire his own lawyer. D. No, because a lawyer may give legal advice to an unrepresented opposing party. Question #266. OA570: Sandy Hook Families Win Major Settlement An attorney represented a client who was the plaintiff in a personal injury action. After the action had settled, the attorney received a check in the amount of $10,000 payable to the attorney, which she deposited in her client trust account. The next day, the attorney received a letter from a bank. The bank informed the attorney that the client had failed to make mortgage payments on a residential building for the last three months and demanded that the attorney immediately release $3,000 of the settlement proceeds to the bank to avert immediate foreclosure proceedings against the client. When the attorney, who did not represent the client in the mortgage matter, informed the client of the bank’s letter, the client did not dispute the $3,000 debt but expressed no concern for what the bank might do. The client believed that the building was essentially worthless, so he assumed that the bank would never foreclose. The client instructed the attorney to take her legal fees from the settlement and turn the rest of the proceeds over to him. The attorney followed the client's instructions. Is the attorney subject to discipline? A. Yes, because the client did not dispute the $3,000 debt to the bank. B. Yes, because the attorney knew that the client was planning to force the bank to foreclose the mortgage. C. No, because the attorney did not represent the client in the mortgage matter. D. No, because the bank had no established right to the specific proceeds of the client's personal injury settlement. Question #267. OA572: Ukraine and International Law; Gov. Abbott's Despicable Attack on Trans Kids An attorney in a five-lawyer firm learned that one of her partners had charged personal expenses to a client and had fraudulently represented that the expenses were related to the client's representation. After being confronted with evidence of the fraud, the partner resigned. The attorney disclosed the fraud to the client and directed the law firm’s billing department to refund the fraudulent charges to the client. The attorney asked the client whether the client wanted to disclose the partner’s conduct to the disciplinary authority. The client did not object to the disclosure but left the decision to the attorney. Is the attorney required to report the partner’s conduct to the disciplinary authority? A. Yes, because lawyers must report all violations of the rules of professional conduct to disciplinary authorities. B. Yes, because the conduct raised a substantial question as to the partner's honesty, trustworthiness, or fitness as a lawyer. C. No, because the attorney acted promptly to rectify any damage to the client caused by the partner’s conduct. D. No, because the attorney owes a continuing fiduciary duty to the partner with respect to matters that occurred during the course of their partnership. Question #268. OA576: Election Laws and Lies, with Lawrence Lessig An attorney represented the wife in an acrimonious divorce proceeding involving issues of property division and child custody. After one day of trial, the husband, through his lawyer, made a settlement offer. The proposed settlement required that the wife’s attorney agree not to represent the wife in any subsequent proceeding, brought by either party, to modify or enforce the provisions of the decree. The wife wanted to accept the offer, and her attorney reasonably believed that it was in the wife’s best interest to do so because the settlement offer was better than any potential award to the wife resulting from the case going to judgment. Consequently, the attorney recommended to the wife that she accept the offer. Was it proper for the wife’s attorney to recommend that the wife accept the settlement offer? A. No, because the attorney did not obtain the wife’s informed consent to the conflict of interest created by the proposed settlement. B. No, because the proposed settlement restricted the attorney’s right to represent the wife in the future. C. Yes, because the restriction on the attorney was limited to subsequent proceedings in the same matter. D. Yes, because the attorney reasonably believed that it was in the wife’s best interest to accept the proposed settlement. Question #268p OA578: 1/6 Prosecutors Admit Violating Suspects Rights. What Happened? A man asked an attorney to represent him pro bono in an appeal from the denial of his unemployment benefits. The attorney agreed, but missed the deadline to file the appeal. When the attorney discovered this error, he met with the man and presented a proposed settlement of the man's potential malpractice claim against himself, against the attorney. The attorney proposed to pay the man an amount equal to the unemployment benefits he would have received had his appeal been successful. In exchange, the man would agree to waive any malpractice claim. The attorney memorialized this proposed settlement in a written document that included a statement advising the man to seek independent legal counsel before accepting the settlement and stating that the man could take as long as he needed to do this. However, without consulting independent legal counsel, the man agreed to and signed the proposed settlement the next week. The attorney subsequently paid the man the money he would have received from a successful appeal. Is the attorney subject to discipline? A. Yes, because the attorney made an agreement with the man to waive a potential claim for malpractice. B. Yes, because the man did not consult independent counsel before he agreed to the proposed settlement. C. No, because the attorney advised the man in writing to seek independent legal counsel before agreeing to the proposed settlement. D. No, because the attorney agreed to fully compensate the man for his losses. Question #269. OA580: Google Caught Pulling Privilege Shenanigans An attorney represented a client in a breach of contract action against the client's former business partner. During the presentation, the client presented the attorney with incontrovertible proof that the former partner had committed perjury in unrelated litigation several years earlier. The prior litigation had been resolved in the former partner's favor. Neither the attorney nor the client had been involved in any way in the prior litigation. However, at the time of the prior litigation, the client and the former partner had had a close personal and business relationship. The attorney believed that it would be detrimental to the client's best interests to reveal the former partner's perjury because the court might draw negative inferences about the client from the former partner's behavior. As a result, the attorney did not disclose the former partner's prior perjury to the court. Was the attorney's conduct proper? A. Yes, because the attorney believed that the disclosure would be detrimental to the client's best interests. B. Yes, because neither the client nor the attorney had been involved in any way in the prior litigation. C. No, because the attorney possessed knowledge that the former partner had perpetrated a fraud on a tribunal. D. No, because the information regarding the prior perjury was not privileged. Question #270. OA582: Alex Jones Is In Big Trouble A buyer mailed a signed order to a seller that stated, "Please send me 1,000 pens at whatever price you are currently selling them for." The seller received the order on June 1. Later that afternoon, the seller mailed the buyer a signed letter stating that he accepted the buyer's offer at the current price of $2.00 per pen. The seller properly addressed, stamped, and mailed the letter to the buyer. On June 2, the buyer called the seller and left a voicemail message saying that he revoked his offer to purchase pens. On June 3, the seller listened to the buyer's voicemail message. On June 4, the buyer received the seller's letter. Is there a contract between the buyer and the seller on June 4? A. No, because the buyer effectively revoked the offer prior to receiving the seller's written acceptance B. No, because the offer was too indefinite since it did not contain a price term. C. Yes, because the offer was irrevocable for a reasonable time. D. Yes, because the seller accepted the buyer's offer before the buyer attempted to revoke it. Question #271. OA584: Amazon Workers Unionize! A state passed a statute that conditioned the holding of a classroom teacher position in all K-12 public schools in the state on being a citizen of the United States. A group of teachers that were denied employment on the basis of their citizenship challenged the constitutionality of this statute in state court Assuming there are no issues in terms of standing, what is the most likely outcome of their challenge? A. The statute will be deemed constitutional as it is rationally related to a legitimate government interest. B. The statute will be deemed constitutional as it is substantially related to an important government interest. C. The statute will be deemed unconstitutional as it is not narrowly tailored to achieve a compelling government interest. D. The statute will be deemed unconstitutional as it infringes on a legislative area that is exclusively reserved for the United States Congress. Question #272. OA586: CA Uses TX Abortion Ban Trick but for Gun Control A city allows concerts to be held in its city park stadium. However, after hearing complaints about the noise level, the city passes an ordinance that requires groups that rent out the city park stadium to use the city's audio equipment found in the city park stadium. A particular band of a unique genre states that the city’s audio equipment found in the city park stadium is insufficient for the various songs it plans on performing. The city nonetheless insists that the band use the city's equipment. The band sues for a violation of its First Amendment rights. Will the band be successful? A. Yes, because the ordinance is not narrowly tailored to serve an important government interest. B. Yes, because although the ordinance is constitutional on its face, it is unconstitutional as applied to the band. C. No, because the ordinance serves an important government interest and there are no less restrictive means by which it can satisfy that interest. D. No, because the ordinance is rationally related to a legitimate government interest. Question #273. OA588: Wildly Ignorant Trump Judge Upends National Mask Policy A sister and brother own property as joint tenants. A victim of a car accident has a judgment against the sister after a court found she was negligent in causing a car accident that injured the victim. The victim properly filed the judgment and has a lien on the sister's real property. The statute in the jurisdiction states: "Any judgment properly filed shall, for ten years from the date of filing, be a lien on the real property then owned or subsequently acquired by any person against whom the judgment is rendered." Before the victim took any additional action, the sister died. Who has an interest in the land? A. The brother. B. The brother and the sister's heirs. C. The brother and the victim. D. The brother, subject to the victim's judgment. Question #274. OA591: Musk Buys Twitter, Part 2! A mother, who was the owner in fee simple of Blackacre, validly executed and conveyed the land to her daughter via a quitclaim deed. Her daughter paid no consideration and failed to record the deed. Six months later, the mother asked the daughter to rip up the deed and stated that she instead wished to keep the land. The daughter voluntarily destroyed the deed. One week later, the mother and the daughter were killed in an accident. They both died intestate. The heirs of the mother and the heirs of the daughter both claim title to Blackacre. Who has title to Blackacre? A. The mother's heirs, because the daughter never recorded her interest in the land. B. The mother's heirs, because the daughter ripped up the deed. C. The mother's heirs, because the daughter did not give valid consideration in exchange for the land. D. The daughter's heirs, because the daughter never effectively passed title of the land back to the mother. Question #275. OA593: Roe Was Never About a 'Right to Abortion' A defendant is charged with homicide after the defendant was involved in a car accident that killed a pedestrian. At the defendant's trial, the prosecution called to the stand a shop-owner who testified that immediately after the accident, a witness ran into his store and said, "Whoa! Can I use your phone? That blue car just ran a red light and hit someone!" The witness is not testifying at trial and is not present in court. With the sole objective of discrediting this testimony, the defense sought to introduce an audio recording from when a police officer interviewed the witness at the station the next day. The recording consists of the following exchange: Officer: "Did you see a blue car run a red light?" Witness: "No. The blue car did not run a red light. The light was green when it drove through the intersection." The prosecution objected to the introduction of the recording. Assume that the recording and the voices are properly authenticated and satisfy the best evidence rule. Should the court sustain the objection? A. Yes, because the recording constitutes inadmissible hearsay. B. Yes, because extrinsic evidence is not admissible in this situation C. No, because the recording constitutes admissible hearsay under the recorded recollection exception. D. No, because extrinsic evidence is admissible in this situation. Question #276. OA595: Shareholders Sue Twitter, Alleged Facts Jeopardize Musk Deal A driver ran a stop sign and collided with a vehicle driven by a man. The man was injured. The man sued the driver and the city for negligence. The man alleged that the driver was negligent in not paying attention and the city was negligent for not properly trimming the brush that partially covered the stop sign. The jury found that the amount of damages is $100,000. The jury found that the man was 10% negligent. It also found that the driver was 85% negligent and the city was 5% negligent. How much can the man recover from the city? A. $5,000. B. $90,000. C. $100,000. D. Nothing. Question #277. OA597: 5th Circuit Tells SCOTUS Hold My Beer A 16-year-old boy was operating a motor boat. While operating the motor boat, the boy got distracted by text messages from his friend, and the boy inadvertently struck a woman sitting in a canoe in his path. The boy did not know the woman was there. The woman suffered serious injuries and sued the boy for battery. Will the woman succeed? A. Yes, if the trier of fact finds that a boy of his age, intelligence, and experience would have been able to avoid the collision. B. Yes, because the boy intended to drive the boat, which caused her harm. C. No, because the boy did not intend to strike the woman. D. No, because the boy did not make direct physical contact with the woman. Question #278. OA599: Our Massive Gun Problem - What Can We Do? A man walked into a convenience store and immediately approached the cashier. He yelled, "Give me all your money or I will shoot you!" The man had his hand in his pocket with his finger pointed like a gun and shook it at the cashier. However, there was no gun in his pocket. Feeling scared, the cashier tripped the alarm and when the man heard it, he ran out of the store, empty-handed. The man's friend was waiting for the man outside the store to serve as the getaway driver. As soon as the man ran out of the store, he quickly got into his friend's car, and the friend drove away from the scene in order to avoid being caught. What crime is the man’s friend liable for? A. Robbery. B. Attempted Robbery. C. Larceny. D. No Crime. Question #279. OA601: SCOTUS Ruling Guarantees More Innocent Will Be Put to Death A restaurant headquartered in state a was the subject of multiple premises, liability suits, stemming from a spill on the floor that caused multiple injuries. Plaintiff 1 sued the restaurant in state court located in state A for negligence. In that suit, it was determined after a full litigation that the employees of the corporation were aware of the spill on the floor and did have a duty to warn the plaintiff of the spill. Following a full trial, plaintiff 1 succeeded in her suit against the restaurant. The restaurant filed an appeal and that appeal is still pending. Plaintiff 2 filed suit against the restaurant in federal court located in state B. Plaintiff 2 sought to preclude the restaurant from asserting that it did not owe a duty to warn of the spill under collateral estoppel. The law of State A allows for the use of offensive non-mutual collateral estoppel. The law of State B prohibits the use of offensive non-mutual collateral estoppel. Federal law allows for the use of collateral estoppel as justice permits. The law in all three jurisdictions states that a judgment is final until reversed or otherwise modified assumed jurisdiction and venue are proper. What should the court do? A. Allow the restaurant to litigate the issue of its duty to warn, because the federal court will apply the law of the state in which it sits. B. Evaluate whether justice would allow the use of non-mutual collateral estoppel because the federal law will apply its own law on procedural issues. C. Prohibit the restaurant from litigating the issue of its duty to warn because the federal court will apply the law of state A. D. Delay the case until the appeal is decided. Question #280. OA603: California Court Rules That... Bees are Fish? A State A resident man and a State B resident woman were in a car accident in State B. The man filed a lawsuit against the woman in federal court in State A alleging the woman was negligent. The man decided to serve the woman at her parents’ house, which is a few miles from the woman's house but located just across the border in State A. The man knew the woman visited her parents occasionally, but she was not present when service was made. The woman's parents accepted service of process and notified her of the lawsuit. The woman timely filed an answer denying the allegations against her. A few weeks later, the woman filed a motion to dismiss the case. State A's rules of civil procedure are identical to the Federal Rules of Civil Procedure. If the motion to dismiss is properly granted, what would be the most likely reason why? A. The method of service was improper. B. The man was asking for $75,000 in damages. C. The woman not have minimum contacts with State A D. The car accident did not occur in State A Question #281. OA605: GEICO to Pay $5.2m for Car Sex?! and MORE Jan 6 Bombshells! A soccer player brought a battery action against a player on the opposing soccer team. The soccer player presented evidence that the opposing player kicked the soccer player in the shin when the soccer player was trying to score a goal. The opposing player presented evidence that he was merely trying to get the ball from the soccer player and the kick to the shin was an accident. Further, the opposing player testified that the game was "played rough" in their city, and the soccer player consented to being kicked regardless. The opposing player also presented a witness who testified that several players are injured over the course of a typical soccer game in that city and that games have been played that way for several years in that particular area. The soccer player did not cross-examine the witness. After resting its case, the opposing player moved for judgment as a matter of law. Should the court grant the motion? A. Yes, because the soccer player did not cross-examine the witness who testified regarding custom in the city. B. Yes, because the judge can, as a matter of law, decide cases when the judge believes the evidence substantially favors one party. C. No, because whether the kick was an accident and whether the soccer player consented to it are questions of fact for the jury. D. No, because a motion for judgment as a matter of law must first be made at the close of the plaintiff's case-in-chief rather than for the first time at the end of trial. Question #282. OA607: Will There Be Justice for Shaye Moss and Ruby Freeman? and Bye Bye Miranda Rights! State A requires all state park rangers to pass a fitness test in order to be hired for a job as a state park ranger. The justification is that the job requires intense physical activity. Primarily men apply for the job. Last year, 50 men applied for the job as a state park ranger, and 10 women applied. 30 people passed the physical fitness test. All were men. The women brought a lawsuit claiming that the physical fitness test violated the Equal Protection Clause of the United States Constitution Will the women prevail in their lawsuit? A. No, because the state can show that the fitness test is substantially related to an important government interest. B. No, because there is a rational basis for the physical fitness test. C. Yes, because the state cannot show that the fitness test is substantially related to an important government interest. D. Yes, because the woman can demonstrate a discriminatory impact. Question #283. OA609: Surprise Jan 6 Hearing Bombshells Within Smocking Guns Within More Bombshells A city enacted an ordinance banning all flyers consisting of wholly commercial advertisements on utility poles on public sidewalks. The reason for the ordinance was to prevent litter on the public streets and sidewalks. However, the city continued to allow other kinds of flyers to be displayed on utility poles on public sidewalks. As a result of the ordinance, approximately 50 of the 500 flyers were removed from the utility poles since they consisted of wholly commercial advertisements. Is the ordinance constitutional? A. Yes, because it is commercial speech and subject to a lesser scrutiny than other kinds of speech. B. Yes, because prohibiting litter is a substantial government interest and the ordinance is necessary to effectuate this interest. C. No, because prohibiting litter is not a compelling government interest. D. No, because there is not a reasonable fit between the law and the outcome desired by the city. Question #284. OA611: Cipollone to Testify! Andrew Called It! Here's What To Expect. A seller entered into a contract to convey Blackacre to a buyer. The contract stated that the closing was to occur "on May 1." The buyer was prepared to perform on May 1, but the seller was not. On June 1, the seller was ready to close on Blackacre. The buyer does not want to follow through with the closing and claims the seller has breached the contract. Must the buyer follow through with the closing? A. Yes, unless the buyer can show the delay of closing caused the buyer to suffer undue harm B. Yes, unless the contract or the circumstances indicate that time is of the essence C. No, because the closing date is an express term of the contract that must be complied with exactly. D. No, because one month is not a reasonable time after the initial closing date. Question #285. OA613: Cipollone Prediction: Bullseye. Here's the Next Potential Jan 6 Domino to Fall Testator devised his home "to my son for life, then to my son’s children and their heirs." Testator's son, a 70-year-old widower, had one unmarried adult son and one unmarried adult daughter. In an appropriate action to construe the terms of the will, the court will determine that the remainder to the children is: A. Indefeasibly vested. B. Contingent. C. Vested, subject to partial divestment. D. Vested, subject to total divestment. Question #286. OA615: Rep. Jared Huffman on the Health Share Transparency Act A man and a woman own neighboring pieces of land. The man wants to build an in-ground pool in his backyard. Fearing that if not properly maintained the pool could cause damage to her yard and attract unwanted plants and animals, the woman makes the man promise to have the pool cleaned and maintained by someone qualified once every two weeks while in use. This promise was reduced in writing and contained the following clause: "It is our intent that this agreement will bind our successors unless an agreement is reached between the owners at that time." The man and the woman had this written agreement added into the deeds of both pieces of property One year later, the man granted his land to his son, and the woman granted her land to her daughter. There were no discussions about the pool maintenance promise. Two months later, the daughter noticed that the pool was in disrepair as no one had cleaned it since the son moved in, although he had been using it. The daughter filed for an injunction demanding that the son resume cleaning his pool. A. The son, because there was no horizontal privity between the man and the woman. B. The son, because there was no horizontal privity between the son and the daughter. C. The daughter, because she is an original party to the contract and can enforce it against the other original party or any party in privity. D. The daughter, because she sought an injunction. Question #287. OA617: Alex Jones' Lawyers Have No F*cking Idea What They're Doing In a civil battery case where the plaintiff was suing the defendant after the defendant allegedly tripped her while she was walking down the stairs, the plaintiff calls a witness to testify on her behalf. The witness corroborates the plaintiff's story. Then, on cross-examination of the witness, counsel for the defendant asks, "Isn't it true that you cheated on your CPA exam last year by attempting to look at your notes during the test?" The plaintiff's counsel objects to the question. How should the court rule on the objection? A. It should be sustained because character cannot be proved by specific instances of conduct B. It should be sustained because the question is irrelevant to the battery case C. It should be overruled because the question regards the credibility of the witness D. It should be overruled because it is permissible character evidence Question #288. OA619: OK, Alex Jones's Lawyers REALLY Don't Know WTF They're Doing In a civil action for conversion of her automobile, a plaintiff seeks to offer testimony that after her automobile went missing, the defendant approached the plaintiff at her place of business and said, "I just stole your car. How does it feel to be stranded?" The business had security cameras that recorded the entire conversation. The defendant objects to the plaintiff's testimony regarding his statement. Is the plaintiff's testimony regarding the defendant's statement admissible? A. Yes, as a declaration against interest B. Yes, because it is not hearsay C. No, because the tape recording is the best evidence D. No, because it is hearsay not within any exception Question #289. OA621: The FBI Goes to Mar-a-Lago! This Is Big... Knowing that the owner of a house owned a large collection of valuable jewelry, a man decided to break into the house to steal some. During the day when he thought the owner was at work, the man broke the lock on the door and entered. The owner was actually upstairs at the time, and the man could hear footsteps. Getting nervous, the man grabbed a necklace he saw laying on a coffee table. When he started to hear the owner coming down the stairs, the man panicked and smashed a window to escape quicker. The owner had actually started coming downstairs just after the man entered the house, witnessing both his grabbing the necklace and smashing the window. This latter action scared the owner immensely, and the police were called. In a common law jurisdiction, what crime(s) could the man be guilty of? A. Larceny. B. Robbery. C. Larceny and burglary. D. Robbery and burglary. Question #290. OA623: This Week in High Treason A man was taken to a police station for questioning after his girlfriend disappeared. The man was not given Miranda warnings. After a few minutes of questioning by a police officer, the man confessed to murdering his girlfriend and told the police officer that he threw the gun that he used to shoot his girlfriend in a swamp, which was located several miles from the city. After an intensive search of the swamp over a period of several weeks, the police found the gun. The man is charged with first-degree murder. The prosecutor plans to use the gun as evidence against the man at trial. The man argues that the gun should be suppressed at trial. Will he prevail? A. Yes, because the man was not given Miranda warnings. B. Yes, because the interrogation violated the man's due process rights. C. No, because the exclusionary rule will not serve to exclude the gun from evidence. D. No, pursuant to the "inevitable discovery" exception to the exclusionary rule. Question #291. OA625: Student Loan Forgiveness! A woman and a piano teacher entered into a contract whereby the teacher would give the woman piano lessons once a week for three months, and the woman would pay the teacher an hourly rate. The contract contained the following provision: "the parties agree that neither will assign the contract." The piano teacher's sister was in need of money. The piano teacher reached a written agreement with the sister to assign the piano teacher's right to collect the money from the woman to the sister. Which of the following is a true statement? A. The piano teacher's assignment is invalid and the piano teacher breached the contract with the woman. B. The piano teacher breached the contract with the woman, but the assignment will still be effective. C. The piano teacher's assignment is valid so long as the woman gave permission for the piano teacher to do so. D. The piano teacher's assignment is valid and the sister will be able to sue the woman if she does not pay. Question #292. OA627: How To Convict Donald Trump A local resident arrived at his favorite fast-food restaurant intending to purchase lunch. When the resident entered the building he accidentally bumped into another patron, causing some of the patron's drink to spill on the floor. The collision was not observed by any restaurant employees. The resident proceeded to order his food. About one minute later, another customer entered, slipped on the wet floor, and sustained a broken wrist from the fall, which necessitated a surgical operation. Arguing that the restaurant was negligent in protecting patrons from dangers, the customer sued the restaurant in tort. Will the customer succeed in his cause of action? A. Yes, because the restaurant breached its duty to inspect the premises and warn of the dangerous condition or make it safe. B. Yes, because the restaurant is vicariously liable for the torts of its patrons. C. No, because the customer's true claim is against the resident that caused the drink to spill. D. No, because the restaurant did not breach any duty owed to the customer. Question #293. OA629: Hack Judge Hands Trump a Win. What Comes Next? A man was at his neighbor's house for a neighborhood gathering. The man noticed that the neighbor had a new garden hose. Upon seeing the garden hose, the man remembered that he needed to water his plants. The man decided to use the neighbors garden hose while the neighbor was entertaining guests. The man stretched the garden hose to his backyard and began watering the plants. However, the garden hose was stretched too far, broke in half, and could not be repaired. In a suit for damages by the neighbor, will the neighbor recover? A. Yes, for the actual damage to the garden hose. B. Yes, for the value of the garden hose before the neighbor broke it. C. No, because the neighbor did not intend to keep the garden hose. D. No, because the neighbor did not intend to damage the garden hose. Question #294. OA631: Updates on Trump Treason! A man is riding a bicycle too small for his size. He rides carefully around a bend in the road but suddenly smashes into a parked car. The owner of the parked car sues for damages. The owner will win if: A. The bicycle rider rounded the bend in violation of a law. B. The bicycle rider rounded the bend in violation of a safety law. C. The bicycle rider rounded the bend in violation of a cycling law. D. The bicycle rider rounded the bend in violation of state law. Question #295. OA633: Why Adnan Was Released During the course of three days, oral testimony is admitted on the question of causation in a voluntary manslaughter trial. The wife of the accused seeks to testify. The accused objects to the admission of her testimony. Under the federal rules of evidence. A. The wife can decide to testify against her husband. B. The husband can have his wife's testimony against him rejected. C. The prosecutor can impeach the wife's testimony. D. The other state witnesses must corroborate the wife's testimony. Question #296. OA635: Trump Gets His Special Master, Immediately Regrets It Golf corporation paid the world’s #1 golfer $330,000 to promote its golf equipment during a White House dinner to which the world’s #1 golfer was invited. On the night of the dinner the golfer fell ill and did not attend. He may escape liability for damages by A. Offering a more valuable substitute performance. B. Offering an equivalent substitute performance. C. Offering to promote golf throughout the country. D. Suing for specific performance of the contract instead of damages. Question #297. OA637: Trump's Appeal Is On Clarence Thomas's Desk, But That's Ok A woman left a will which devised the residue of her estate to her two doctors in equal shares. The will has been probated. The woman's evening doctor sold his share to the morning doctor. After the sale: A. Evening doctor holds the property in fee simple. B. The morning doctor holds the property in fee tail. C. The two doctors hold as joint tenants with right of survivorship. D. The morning doctor holds the property in fee simple. Question #298. OA639: Alex Jones Owes a Literal Billion Dollars While driving his patrol car, officer Jerry saw a man reach into a woman’s bag and remove a purse. Officer Jerry stopped his patrol car, got out and walked quickly after the man, and caught up with the man on a corner. Did officer Jerry have a right to seize an expensive watch from the man’s wrist after officer Jerry arrested the man and took the purse out of the man’s pocket? A. Yes, it was in plain view. B. Yes, he had reason to suspect that it was stolen. C. He lacked reason to suspect that it was stolen, no. D. Yes, because the man in fact stole it. Question #299. OA641: Massive Smocking Gun in Newly Revealed Eastman Docs! After school hours a school teacher asked a student to carry the school teacher’s books to the teacher’s car. The student demanded payment but the teacher threatened to report the student to the principal for bad behavior. As a result the student carried the teacher's books to her car for no payment. Is the student entitled to damages for breach of contract? A. No the contract was not breached. B. Yes assuming an implied contract existed. C. Yes because no contract existed. D. No because the teacher coerced the student’s unwilling performance. Question #300. Whoops! Here's OA643 T3BE (Meant to be part of OA643: Magnus Carlsen Accused Hans Niemann of Cheating at Chess. The Chess World Exploded. Now Hans Has Sued) Because her wedding was in three weeks a future bride requested to use a hotel’s banquet hall for her wedding reception. The future bride expected about seventy guests. When she tried to pre-pay for the hall, the hotel said that pre-paying was not allowed but gave no reason. On the future bride's wedding day, another wedding party used the hall. For what tort or torts could the future bride bring an action against the hotel? A. Negligent or intentional misrepresentation and she will succeed B. Fraud and deceit and she will succeed C. Conversion of the hall and she will succeed at common law D. None of the above Question #301. OA645: We Badly Underestimated Just How Terrible Elon Musk Is At Business A fifteen year-old girl wrote an entry on her Internet networking page claiming that her former boyfriend was a murderer who once cut another boy into pieces and buried him in a land fill. After reading this, the former boyfriend’s new girlfriend threatened to leave the former boyfriend. If the former boyfriend says, "I’ll kill that bitch!" A. The statement does not make the former boyfriend guilty of a crime. B. The statement makes the former boyfriend guilty of attempted murder. C. The statement may make the former boyfriend guilty of second degree murder. D. The statement has not been proved beyond a reasonable doubt. Question #302. OA647: Senate AND House Both Very Much In Play for Democrats! A farmer asked a grocery distributor for an application to supply onions to the distributor and filled it out. The farmer addressed it correctly then mailed it first class to the address provided on the form. Two months have passed and the farmer has heard nothing further from the distributor. Has the farmer’s offer lapsed? A. If the farmer indicates that it has lapsed, yes. B. If the distributor indicates that it has lapsed, yes. C. If the farmer accepts the offer, yes. D. If the distributor determines that the first class mailing was inadequate, yes. Question #303. OA649: Respect for Marriage Act Advances in the Senate! A man was charged with larceny of a coat from a bar stool. The state produced a witness who testified that the man said before he took the coat, "Gosh I'm so forgetful. I must have left this here hours ago. It could easily have gone missing." The judge declared a mistrial rather than acquittal. Can the man be retried? A. Yes, this retrial will not offend the federal constitution. B. No, under the federal constitution a retrial requires new evidence. C. Yes, if the man can be proved guilty. D. Yes, under the Erie doctrine. Question #304. OA652: Another REALLY Bad Week for Alex Jones... Cable Corp paid for an ad in the newspaper promising to include the FDO channel in its program line-up for subscribers who signed up within two weeks. A family signed up on the twelfth day and received the FDO channel. On the sixteenth day the FDO channel stopped showing on the family’s television. What are Cable Corp’s rights? A. Cable corp is entitled to time to restore the FDO channel B. Cable corp delivered on its promise and has no liability C. The family is liable for damages for misuse of the FDO channel D. The family did not inform cable corp of the problem with the FDO channel Question #305. OA656: Oath Keepers Found Guilty of Sedition! Peyote Church plans to hold a lottery to raise funds for a parsonage but lotteries are illegal in the state. Has Peyote Church’s freedom to practice its religion been unduly burdened? A. No because lotteries are illegal for non religious reasons B. No because lotteries are also illegal for non religious organizations C. Yes because the federal government should not be involved D. Yes because of the tenth amendment Question #306. OA660: Alex Jones... Declares... (personal) BANKRUPTCYYYYYY Training Co conveyed premium shopping center space to a man by quitclaim deed and received $210,000. Four years later another business, Diet Foods Co, filed an action to quiet title to the property as an adverse possessor. Can Diet Foods Co introduce evidence that it always kept the property clean? A. Yes, it tends to show possession B. Yes, it tends to show actual possession C. Yes it tends to show cleanliness D. Yes it tends to show that Diet Foods Co employs professional cleaners Question #307. OA664: Trustees Call BS on Alex Jones's Bankruptcy Shenanigans If the seller of real property under a general warranty hears from the buyer that a party is attempting to evict the grantee what must the seller do? A. Nothing - the grantee now holds title. B. Perform under one of the future real covenants. C. Nothing until the person attempting to evict succeeds. D. Nothing if the person attempting to evict succeeds. Question #308. OA667: The Referrals Have Been Made. Now What Will It Take To Convict? While talking with B about baseball A held his bike to keep it from falling over. A made a joke. While laughing excessively at the joke, B blundered into A’s bike, knocking it to the ground. The bike is now no longer functional. Is B liable for conversion? A. No, B had no volition to touch the bike B. No, A had no volition to touch the bike C. Yes, because conversion has occurred D. Yes, because the bike is now no longer functional Question #309. OA669: The Witnesses Against Donald Trump A fast food company employed a student as a door greeter, to be paid weekly at the rate of $7 an hour. The student fell asleep on his feet on her second day on the job and the fast food company terminated her employment. The student later won damages of two weeks’ pay against the fast food company. This was probably because A. The fast food company breached the employment contract B. The student had an at-will contract with the fast food company C. A condition precedent to the fast food company's performance failed D. An employee does not give up all personal rights Question #310. OA672: How Conservative Courts Rewrite History City college is funded by the county government of state X. City college issues an email directive that all employees disclose all social club memberships at the start of every school year. The purpose of the directive is unclear. Most employees make the disclosure for fear of losing their jobs, but a college accountant challenges the directive as unconstitutional. Is the college accountant correct? A. The college accountant is probably wrong - such memberships exist B. The directive is probably wrong - social club memberships are a private right. C. No. The directive is probably constitutional as long as there is a connection to job performance. D. Yes. The directive is probably constitutional if there is a proven state action and it is wrongful. Question #311. OA676: Kel McClanahan on the Biden Docs City college is funded by the county government of state X. City college issues an email directive that all employees disclose all social club memberships at the start of every school year. The purpose of the directive is unclear. Most employees make the disclosure for fear of losing their jobs, but a college accountant challenges the directive as non-mandatory. Can the college accountant call one of her fellow employees to testify that the directive was distributed only by email? A. No because email is not a proper forum for college instruction. B. Yes because it is relevant. C. Yes because it is relevant to the issue of state action. D. Yes because it is relevant to whether it was advisory or mandatory. Question #312. OA680: The Pathetic SCOTUS Leak Investigation Fizzles During examination-in-chief, a plaintiff's lawyer seeks to show the plaintiff's witness eight text messages the witness sent to a relation in another state an hour after a hurricane struck the witness's town. May the lawyer do so? A. Yes, because the witness is dodging the lawyer's questions. B. Yes, because the witness cannot remember what she texted. C. Yes, because it is relevant for cross-examination. D. Yes, because the witness is considered hostile. Question #313. OA684: Why We Haven't Seen the Georgia Grand Jury Report Yet During cross-examination, a plaintiff's lawyer seeks to show a witness copies of text messages the witness allegedly sent to a relation in another state, after a hurricane struck the witness' town. If the witness asserts that she did not send the texts, which of these actions may the plaintiff's lawyer take? A. Request a sidebar conference with the witness and judge. B. Tell the court the truth as he knows it. C. Disprove the witness' assertion by other testimony. D. Raise a timely objection. (Thomas' answer: C. Correct answer: C) Question #R001 Thomas Takes the Bar Exam Returns! T3BE Week 1 A man who believed that his wife was cheating on him with her gym trainer decided to kill the trainer. He loaded his handgun and set off for the trainer’s house. Because he was anxious about committing the crime, the man first stopped at a bar, drank eight shots of hard liquor, and became intoxicated. He then left the bar and went to the trainer's house. When the trainer answered the door, the man shot and killed him, The man then passed out on the trainer's porch. The man has been charged with murder in a jurisdiction that follows the common law. Can the man raise an intoxication defense? A. No, because drinking at the bar was the proximate cause of the killing. B. No, because the man intended to commit the murder and drank to strengthen his nerve. C. Yes, because drinking at the bar was a foreseeable intervening cause of the killing. D. Yes, because the man’s intoxication negated the specific intent required for murder. Question #R002 Thomas Takes the Bar Exam Returns! T3BE Week 1 Without a warrant, police officers searched the garbage cans in the alley behind a man's house and discovered chemicals used to make methamphetamine, as well as cooking utensils and containers with the man's fingerprints on them. The alley was a public thoroughfare maintained by the city, and the garbage was picked up once a week by a private sanitation company. The items were found inside the garbage cans in plastic bags that had been tied closed and further secured with tape. The man was charged in federal court with the manufacture of methamphetamine. Did the search of the garbage cans violate the Fourth Amendment? A. No, because the man had no reasonable expectation of privacy in garbage left in the alley. B. No, because the probative value of the evidence outweighs the man's modest privacy claims in his garbage. C. Yes, because the alley was within the curtilage of the man's home and entry without a warrant was unconstitutional. D. Yes, because there is a reasonable expectation of privacy in one's secured garbage containers. Question #R003 Thomas Takes the Bar Exam Returns! T3BE Week 1 A doctor who was licensed to practice medicine in a particular state was convicted in state court of improperly distributing specified drugs by writing prescriptions for fictitious persons. Under state law, such an abuse of the prescription-writing privilege requires revocation of a doctor's license. After it received an official notification of the doctor's conviction, the state medical board revoked the doctor's license without affording the doctor any opportunity for a hearing. The doctor has sued the board in state court to set aside the revocation, alleging deprivation of property without due process of law because the board did not provide an opportunity for a trial-type hearing before revoking the license. The doctor does not deny the conviction or the factual basis for it. Which of the following is the strongest argument in support of the state medical board? A. A doctor’s license to practice is a privilege, not a right, and therefore is not property within the meaning of the due process clause of the Fourteenth Amendment. B. Due process requires a balancing of interests, and the state’s interest in preventing drug abuse outweighs the doctor’s interest in the particular procedure followed in the disciplinary proceeding. C. The adjudicative facts necessary to revoke the doctor's license were determined in the criminal trial, and therefore due process does not require any further trial-type hearing. D. The licensing board was required to summarily revoke the doctor's license because Article IV, Section 1, of the Constitution requires the licensing board to give full faith and credit to the doctor's criminal conviction. Question #R004 T3BE Week 2! A husband and wife took their 12-year-old son to a political rally in an auditorium to hear a controversial United States senator speak. The speaker was late, and the wife stepped outside the auditorium to smoke a cigarette. While there, she saw a man placing what she believed to be a bomb against the back wall of the auditorium. She went back inside and told her husband what she had seen. Without alerting anyone, they took their son and left. Some 20 minutes later, the bomb exploded, killing 8 persons and injuring 50. In the jurisdiction, murder in the first degree is defined as an intentional homicide committed with premeditation and deliberation; murder in the second degree is defined as all other murder at common law; and manslaughter is defined as either a homicide in the heat of passion arising from adequate provocation or a homicide caused by gross negligence or reckless indifference to consequence. As to the deaths of the eight persons, what crime, if any, did the wife commit? A. Manslaughter. B. Murder in the first degree. C. Murder in the second degree. D. No crime. Question #R005 T3BE Week 2! A nightclub owner applied for a required zoning permit to open a nude-dancing nightclub in the theater district of a city. An organization of influential city residents began an intensive lobbying effort to persuade the city council to deny the owner a permit to operate any type of nude-dancing facility at any time or in any place in the city. The owner has sued the city in an appropriate federal court, seeking an injunction that would prohibit the city council from considering the organization’s views, on the ground that if the organization is successful in its lobbying efforts, the owner's First and Fourteenth Amendment rights would be violated. The city has moved to dismiss the action. Should the court dismiss the owner’s action? A. No, because nude dancing is symbolic speech and is therefore protected by the First and Fourteenth Amendments. B. No, because the organization does not seek a reasonable time, place, and manner regulation of nude dancing, but instead seeks a total ban on the owner’s opening any type of nude-dancing facility at any time or in any place in the city. C. Yes, because the action is not ripe. D. Yes, because the First and Fourteenth Amendments do not protect obscenity, and nude dancing is obscene. Question #R006 T3BE Week 3! Drugs and Hearsay Police officers had probable cause to believe that drug dealing was routinely taking place in a particular room at a local motel. The motel manager authorized the officers to enter the room and provided them with a passkey. Without obtaining a warrant, the officers knocked on the room's door, announced their presence, and told the occupants that they would like to speak with them. The officers then heard yelling and repeated flushing of the toilet. They then used passkey and entered the rooms. Where they saw the occupants dumping drugs into the toilet. The occupants of the rooms were charged with drug dealing and have moved to suppress the drugs. Should the court grant the motion to suppress? A. No, because exigent circumstances justified the officers' entry. B. No, because the motel manager consented to the officers' entry. C. Yes, because exigent circumstances cannot excuse the lack of a warrant. D. Yes, because the officers cannot benefit from exigent circumstances that they created. Question #R007 T3BE Week 3! Drugs and Hearsay A defendant was charged with aggravated assault. At trial, the victim testified that the defendant beat her savagely, but she was not asked about anything said during the incident. The prosecutor then called a witness to testify that when the beating stopped, the victim screamed: "I'm dying. Don't let [the defendant] get away with it!" Is the testimony of the witness concerning the victim's statement admissible? A. No, because it is hearsay not within any exception. B. No, because the victim was not asked about the statement. C. Yes, as a statement under belief of imminent death, even though the victim did not die. D. Yes, as an excited utterance. Question #R008 T3BE Week 4! Firefighters' Rule and Interstate Commerce A driver, returning home from a long work shift at a factory, fell asleep at the wheel and lost control of his car. As a result, his car collided with a police car driven by an officer who was returning to the station after having responded to an emergency. The officer was injured in the accident and later sued the driver in negligence for her injuries. The driver has moved for summary judgment, arguing that the common law firefighters' rule bars the suit. Should the court grant the motion? A. No, because the firefighters' rule does not apply to police officers. B. No, because the police officer's injuries were not related to any special dangers of her job. C. Yes, because the accident would not have occurred but for the emergency. D. Yes, because the police officer was injured on the job. Question #R009 T3BE Week 4! Firefighters' Rule and Interstate Commerce A federal statute extends federal minimum wage requirements to all dry cleaning stores. The statute contains express findings that, when combined, the wages received by dry cleaning workers have a substantial impact on the national economy and on the flow of goods and services in interstate commerce. These findings are supported by information presented to Congress during committee hearings on the legislation. A small dry cleaning store operates exclusively within a community in the center of a geographically large state. It has no customers from outside the state. It employs three workers, each of whom is paid less than the federal minimum wage. Must this dry cleaning store comply with the statute imposing the federal minimum wage requirements on all dry cleaning stores? A. No, because the store does no business in interstate commerce. B. No, because the wages of the store’s three workers do not have a substantial impact on interstate commerce. C. Yes, because the commerce clause vests Congress with plenary legislative authority over labor relations. D. Yes, because the wages paid by dry cleaning stores have a substantial impact on interstate commerce. Question #R010 T3BE Week 5! Personal Injury and Comparative Negligence In a personal injury case, the plaintiff sued a retail store for injuries she sustained from a fall in the store. The plaintiff alleged that the store had negligently allowed its entryway to become slippery from snow tracked in from the sidewalk. Before the lawsuit was filed, when the plaintiff first threatened to sue, the store's manager said, "I know that there was slush on that marble entryway, but I think your four-inch-high heels were the real cause of your fall. So let's agree that we'll pay your medical bills, and you release us from any claims you might have." The plaintiff refused the offer. At trial, the plaintiff seeks to testify to the manager's statement that "there was slush on that marble entryway." Is the statement about the slush in the entryway admissible? A. No, because it is a statement made in the course of compromise negotiations. B. No, because the manager denied that the slippery condition was the cause of the plaintiff's fall. C. Yes, as a statement by an agent about a matter within the scope of his authority. D. Yes, because the rule excluding offers of compromise does not protect statements of fact made during compromise negotiations. Question #R011 T3BE Week 5! Personal Injury and Comparative Negligence A bright 12-year-old child attended a day-care center after school. The center was located near a man-made duck pond on the property of a corporation. During the winter, the pond was used for ice-skating when conditions were suitable. At a time when the pond was obviously only partially frozen, the child sneaked away from the center’s property and walked out onto the ice over the pond. The ice gave way, and the child fell into the cold water. He suffered shock and would have drowned had he not been rescued by a passerby. At the time of the incident, the pond was clearly marked with numerous signs that stated, "THIN ICE-KEEP OFF." When the child sneaked away from the day-care center, the center was staffed with a reasonable number of qualified employees, and the employees were exercising reasonable care to ensure that the children in their charge did not leave the premises. There had not been a previous instance of a child coming onto the corporation’s property from the day-care center. The jurisdiction follows a rule of pure comparative negligence. In a suit brought on the child’s behalf against the day-care center and based only on the facts above, who is likely to prevail? A. The child, because he left the center while he was under the center’s care. B. The child, because the day-care center is located near a pond. C. The day-care center, because it was not negligent. D. The day-care center, because the child was a trespasser. Question #R012 T3BE Week 6 - ROCKED by Scandal #T3BEgate2 A state constitution provides that in every criminal trial "the accused shall have the right to confront all witnesses against him face to face." A defendant was convicted in state court of child abuse based on testimony from a six-year-old child. The child testified while she was seated behind one-way glass, which allowed the defendant to see the child but did not allow the child to see the defendant. The defendant appealed to the state's highest court, claiming that the inability of the child to see the defendant while she testified violated both the United States Constitution and the state constitution. Without addressing the federal constitutional issue, the state's highest court reversed the defendant's conviction and ordered a new trial. The court held that "the constitution of this state is clear, and it requires that while testifying in a criminal trial, a witness must be able to see the defendant." The state petitioned the United States Supreme Court for a writ of certiorari. On which ground should the United States Supreme Court DENY the state's petition? A. A state may not seek appellate review in the United States Supreme Court of the reversal of a criminal conviction by its highest court. B. The decision of the state's highest court was based on adequate and independent state ground. C. The Sixth Amendment to the United States Constitution does not require that a witness against a criminal defendant be able to see the defendant while the witness testifies. D. The decision of the state's highest court requires a new trial, and therefore it is not a final judgment. Question #R013 T3BE Week 6 - ROCKED by Scandal #T3BEgate2 A man and a woman agreed that the woman would rob a bank and that the man would steal a car beforehand for the woman to use as a gateway vehicle. The man stole a car and parked it two blocks from the bank. He left the car key under the floor mat with a note saying that he wanted nothing more to do with the scene. The next day, the woman robbed the bank and ran to the spot where the man had said he would leave the stolen car. She then escaped in the car and disappeared. She never shared any of the money with the man. In a jurisdiction that has adopted the bilateral requirement for conspiracy, can the man properly be convicted of conspiring with the woman to rob the bank? A. No, because the man received no benefit from the robbery. B. No, because the man withdrew from the conspiracy. C. Yes, because the robbery was successful due in part to the man's actions. D. Yes, because there was an agreement to rob the bank and an overt act in furtherance of the agreement. Question #R014 T3BE Week 7! Best Evidence and Contract Law A defendant is being prosecuted for conspiracy to possess methamphetamine with intent to distribute. At trial, the government seeks to have its agent testify to a conversation that he overheard between the defendant and a coconspirator regarding the incoming shipment of a large quantity of methamphetamine. That conversation was also audiotaped, though critical portions of it are inaudible. The defendant objects to the testimony of the agent on the ground that it is not the best evidence of the conversation. Is the testimony of the agent admissible? A. No, because the testimony of the agent is not the best evidence of the conversation. B. No, because the testimony of the agent recounts hearsay not within any exception. C. Yes, because the best evidence rule does not require proof of the conversation through the audiotape. D. Yes, because the audiotape is partly inaudible. Question #R015 T3BE Week 7! Best Evidence and Contract Law Your client owns a carpet cleaning company. The company recently contracted with a customer to clean the carpets throughout the customer's ten-story building for $15,000, with payment due in 30 days. Before payment was due, the customer sent your client a check for $12,000 along with a note specifying that several of the carpets still had stains and included pictures of the stained carpets. The customer's note also said that if your client cashes the check, the remainder of the customer's debt will be discharged. Your client now seeks your advice as to whether the company can cash the check without relinquishing its right to recover the remainder of the customer's debt. Which of the following search term(s) would be the most likely to produce resources that will answer the client's question? A. Accord and satisfaction. B. Consideration. C. Novation. D. Offer and Acceptance. Question #R016 T3BE Week 8! Elaborate Crimes and Psychedelics Milo, a well-known musician, has battled with severe depression for many years. One evening, while under extreme distress, Milo ingests an excessive amount of psychedelic mushrooms, hoping that the experience would provide him with some clarity. During his hallucinogenic trip, Milo believes he is being attacked by a giant insect monster and, in an attempt to defend himself, he breaks a window and severely injures a bystander outside his home. Upon being charged with aggravated assault, Milo pleads not guilty by reason of insanity, arguing that his mental illness combined with the hallucinations made him incapable of understanding the nature of his act. Which of the following is the most accurate? A. Milo should be acquitted because his voluntary consumption of hallucinogens was an extension of his mental illness. B. Milo's plea is valid, as he was hallucinating during the incident, which hindered his understanding of reality. C. Milo's plea is invalid, as voluntary intoxication cannot be the basis for an insanity defense. D. Milo should be convicted, as the insanity defense requires a continuous, persistent state of psychosis. Question #R017 T3BE Week 8! Elaborate Crimes and Psychedelics Claire, a disgruntled employee, was determined to disrupt her company's annual charity gala. She believed her company was using the charity event to launder money. So, she decided to protest by tampering with the company's new software system, which was set to be unveiled at the gala. Claire had been involved in the development of the system and knew that an excessive amount of data could cause the system to crash. So, she started feeding irrelevant and bogus data into the system. At the gala, when the system was unveiled and the influx of information started to pour in, the system crashed as Claire had planned. The gala came to a halt and the incident resulted in significant reputational damage for the company. However, unbeknownst to Claire, the software system was also utilized in the operation of local emergency services, which experienced a temporary shutdown when the system crashed. This resulted in delays in dispatching emergency vehicles, causing a severe injury to be fatal due to the delay in getting the victim to the hospital. The state has a strict liability statute that penalizes anyone who disrupts emergency services. Claire was charged under this statute. She defended herself by claiming she didn't know the software sys tem was used for emergency services, and her intent was solely to disrupt the company's gala. Given the nature of the strict liability statute, how should the court rule? A. Claire is guilty because she disrupted the emergency services, regardless of her knowledge or intent. B. Claire is not guilty because she did not intend to disrupt emergency services. C. Claire is guilty because she intentionally disrupted the gala, and the disruption of emergency services was a consequence of that. D. Claire is not guilty because the company didn't inform her that the software system was used for emergency services. Question #R018 T3BE Question 9! Prescription Negligence and Breach of Contract Olivia, a pharmacist, accidentally gives a customer, Patrick, the wrong prescription medication. Patrick suffers an adverse reaction to the medication and is hospitalized. Patrick sues Olivia for negligence. At trial, expert witnesses testify that the medication Olivia gave Patrick was in a similar-looking container to the correct medication, and that other pharmacists have made similar mistakes in the past. What is the most likely outcome of this case? A. Olivia will be found negligent because she failed to exercise the standard of care required of a pharmacist. B. Olivia will not be found negligent because the medication containers looked similar. C. Olivia will be found negligent only if Patrick can prove that she intentionally gave him the wrong medication. D. Olivia will not be found negligent if other pharmacists have made similar mistakes in the past. Question #R019 T3BE Question 9! Prescription Negligence and Breach of Contract Lucy and Brandon, two aspiring musicians, signed an agreement where Lucy, a songwriter, would write an original song for Brandon, a popular singer. The contract stipulated that the song would remain exclusive to Brandon, and Lucy would not share it with any other artist. However, due to some personal reasons, Lucy decided to share the song with another singer, contrary to the terms of the contract. Brandon, upon learning this, filed a lawsuit against Lucy, seeking an injunction against the breach. A. Brandon will prevail because Lucy breached the contract. B. Lucy will prevail because an injunction cannot be used in cases involving creative works. C. Brandon will prevail because an injunction is an appropriate remedy in cases involving exclusive rights. D. Lucy will prevail because she shared the song due to personal reasons. Question #R020 T3BE Week 10! Disappearing Wallets and Fake Philanthropy Clyde, a local street performer, is known for his mesmerizing magic tricks. On this particular day, Clyde announces that he will be performing a new trick involving a wallet and a volunteer from the audience. Charles, an unsuspecting tourist, agrees to participate, handing over his wallet to Clyde. Clyde promises Charles and the audience that the wallet will disappear and then reappear in Charles' pocket. However, once the wallet 'disappears', it never reappears. Clyde later confesses that there was no magic trick, he just wanted to steal Charles wallet. In this case: A. Clyde cannot be charged with robbery as he did not use force or threat of force. B. Clyde can be charged with robbery as he deceived Charles and the audience. C. Clyde cannot be charged with robbery as Charles voluntarily handed over his wallet. D. Clyde can be charged with robbery as he intended to steal Charles' wallet. Question #R021 T3BE Week 10! Disappearing Wallets and Fake Philanthropy Peter, a philanthropist, publicly promised to donate $10 million to support an upcoming medical research project at a university. Relying on this pledge, the university invested heavily in the project's infrastructure. Peter, however, decided not to make the donation. Can the university enforce Peter's promise? A. No, because Peter's promise was a gift without consideration. B. Yes, because the university detrimentally relied on Peter's promise. C. No, because there was no signed agreement between Peter and the university. D. Yes, because Peter's promise created a moral obligation. Question #R022 The Trump Trial Is On, and Lordy There Are Transcripts! Jenny, a disgruntled tenant, had been living in a rundown apartment building for years. She was tired of the landlord's consistent failure to make necessary repairs. To teach the landlord a lesson, Jenny decided to set her apartment on fire, hoping that the fire would destroy the entire building and that she could start anew somewhere else. One night, she doused her old couch in her living room with kerosene and set it on fire. To her shock, the fire spread rapidly and got out of control. Panicked she ran outside the building, screaming and warning other tenants about the fire. Most of the tenants were able to escape the fire, but two tenants, an elderly couple living directly above Jenny, were severely injured while escaping. Once the fire was put out by the fire department, the police investigated and found out from other tenants that Jenny had warned them about the fire. On questioning, Jenny confessed to setting her apartment on fire. What is the most serious crime Jenny could be charged with? A. Arson only. B. Arson and assault. C. Arson and attempted murder. D. Arson, assault, and attempted murder. Question #R023 Cannabis Rescheduling; Judge Cannon Stops Trump Trial Rebecca, a famous violinist, signed a contract with "The Grand Symphony," an esteemed music company, to perform exclusively at their annual concerts for the next three years. Due to a sudden illness, Rebecca was unable to perform and thus delegated her performance duties to her protégé, Lisa, a violinist of equal skill and reputation. The Grand Symphony refused to accept Lisa's performance. Lisa sued The Grand Symphony for breach of contract. Is Lisa likely to succeed in her claim? A. Yes, because Rebecca was legitimately unable to perform. B. Yes, because Lisa has equal skill and reputation. C. No, because a contract for personal services cannot be delegated. D. No, because Rebecca did not fulfill her contractual obligation. Question #R024 Liz Warren's CFPB Saved By... Originalism? Jack owns a large fish farm and keeps several difference species, including a type of fish known for its aggressive behavior. One day, a group of divers enters his property without permission and is attacked by the aggressive fish, resulting in injuries. The divers sue Jack under strict liability for their injuries. How will a court likely rule? A. In favor of Jack, because the divers were trespassing on his property. B. In favor of the divers, because Jack is strictly liable for injuries caused by his dangerous animals, regardless of the divers' trespassing. C. In favor of Jack, if he can prove that he had posted adequate warning signs about the aggressive fish. D. In favor of the divers, but only if they can prove that Jack was negligent in securing the area where the aggressive fish were kept. Question #R025 Law School Doesn't Have to Suck A plaintiff was on a crowded BART train during commute hours on the way home. The BART line was undergoing significant renovations, resulting in frequent, sudden stops by the BART trains. The plaintiff was standing in the middle of one of the BART cars and holding onto a pole for stability. The defendant, also standing in the same BART car, was texting on his cell phone and not holding onto anything. The BART train came to a sudden stop causing the defendant to fall toward the plaintiff. The defendant lightly grabbed the plaintiff's arm to stop himself from falling completely over. The plaintiff did not like being touched by anyone. Although she was not injured by the defendant's conduct, the plaintiff subsequently brought an action for battery against the defendant. Will the plaintiff prevail? A. No, because the plaintiff consented to the defendant's contact. B. No, because the plaintiff did not suffer any actual harm. C. Yes, because the defendant failed to exercise reasonable care. D. Yes, because the defendant intentionally grabbed her arm. Question #R026 OA Bar Prep with Heather! T3BE26 The owner of a building leased it to a manufacturer for 10 years. Among the terms of the lease was a provision that prohibited anyone from assigning any rights under the lease without the express written consent of the owner. Three years later, the manufacturer, facing a contraction of its business, entered into an agreement with a retailer to assume the manufacturer's obligations under the lease for the remaining seven years. The manufacturer did not seek the approval of the owner to this agreement, but the owner was aware of it and accepted the retailer's payment of the rent. With five years remaining on the lease, the retailer entered into an agreement with a distributor for the distributor to lease the building for two years. The retailer sought the owner's permission for this transfer. The owner, because of personal animus toward the distributor, has refused to grant his permission. Which of the following is an argument that is most likely to compel the owner to accept the distributor as the tenant of the building? A. The lease provision does not require the owner's approval of the agreement between the retailer and the distributor. B. The owner waived his rights to object under the lease by accepting the retailer as a tenant. C. A non-assignment provision constitutes an unreasonable restraining on alienation. D. The owner does not have a commercially reasonable objection to the distributor as a tenant in the building. Question #R027 Congress passes a law regulating the whole-sale and retail prices of "every purchase of an automobile in the United States." The strongest argument in support of the constitutionality of such a statute is that: A. Taken as a whole, the domestic purchases and sales of such products affect interstate commerce. B. The United States Constitution expressly authorizes Congress to pass laws for the general welfare. C. Congress has the authority to regulate the prices of products purchased and sold because commerce includes buying and selling D. Congress has the right to regulate interstate transportation and the importation of products from abroad. Question #R028 A woman brought suit in State A federal district court against the company she worked for, claiming that it had failed to promote her on account of her gender, in violation of a federal employment-discrimination statute. The woman is a citizen of State A; the company is a corporation incorporated in State B, with its headquarters in State C and with most of its employees working at the office in State A where the woman works. The relief sought by the suit consisted solely of $46,000 in back pay. Two months after the company timely filed its answer, and while discovery was still pending, the company made a motion to dismiss the suit for lack of subject-matter jurisdiction. Will the federal court grant the motion? A. Yes, because the company is a citizen of several states, one of which is the same as the woman's state of citizenship. B. Yes, because although there is diversity of citizenship, the amount in controversy requirement is not met. C. No, because the woman's claim arises under federal law. D. No, because the company waived its objection by failing to assert it either in its answer or in a motion made before it served its answer. Question #R029 OA Bar Prep with Heather! T3BE29 The vaccination of children against childhood contagious diseases (such as measles, diphtheria, and whooping cough) has traditionally been a function of private doctors and local and state health departments. Because vaccination rates have declined in recent years, the President proposes to appoint a Presidential Advisory Commission on Vaccination which would be charged with conducting a national publicity campaign to encourage vaccination as a public health measure. No federal statute authorizes or prohibits this action by the president. The activities of the Commission would be financed entirely from funds appropriated by Congress to the Office of the President for "such other purposes as the President may think appropriate." Is the creation of the Commission by the President a constitutional exercise of authority? A. Yes, because the President has plenary authority to provide for the health, safety, and welfare of the people of the United States. B. Yes, because this action is within the scope of executive authority vested in the President by the Constitution, and no federal statute prohibits it. C. No, because the protection of children against common diseases by vaccination is a traditional state function, and therefore, is reserved to the states by the Tenth Amendment. D. No, because Congress has not specifically authorized the creation and support of such a new federal agency.